ORTHOPEDIC MCQS ONLINE OITE 1213 1A

 ORTHOPEDIC MCQS ONLINE BANK OITE 1213

 

 

2013 & 2012 OITE

Study Guide

© 2012 American Academy of Orthopaedic Surgeons 2012 Orthopaedic In-Training Examination

 

 

-Training Examination

 

2013 & 2012 Orthopaedic In-Training Examination

TABLE OF CONTENTS

 

EXAMINATION QUESTIONS Page

SECTION 1: Basic Science and Orthopaedic Diseases Questions 3

SECTION 2: Foot and Ankle 18

SECTION 3: Hand 35

SECTION 4: Hip and Knee Reconstruction 52

SECTION 5: Medically Related Issues 73

SECTION 6: Oncology. 81

SECTION 7: Pediatric Orthopaedics 103

SECTION 8: Shoulder and Elbow 137

SECTION 9: Spine 153

SECTION 10: Sports Medicine 174

 

© 2012 American Academy of Orthopaedic Surgeons 2012 Orthopaedic In-Training Examination

 

 

 

SECTION 11: Musculoskeletal Trauma 187

 

 

 

 

 

 

 

 

Q.1.Which treatment technique for segmental nerve defects leads to the best motor recovery?

  1. Silicon tube

  2. Collagen tube

  3. Nerve allograft

  4. Nerve autograft

  5. Direct repair with tension

PREFERRED RESPONSE: 4

 

 

 

Q.2 Figure 22 is the sagittal MRI scan of the spine of a 68-year-old woman with a history of chronic back pain radiating into her right lower

extremity. She reports paresthesias involving the right great toe and parts of her shin. Nonsurgical treatment consisting of pain medications and epidural steroid injections has failed to provide relief. Assessment of ambulation is most likely to reveal which pathologic gait pattern?

 

  1. Quadriceps avoidance gait attributable to L5 compression and radiculopathy

  2. Scissor gait attributable to spondylolisthesis at L4-5 and bilateral radiculopathy

  3. Trendelenburg gait attributable to L4 compression and weakness of the gluteus medius

  4. Trendelenburg gait attributable to L5 compression and weakness of the gluteus medius

  5. Varus thrust gait during stance attributable to L5 compression and posterolateral corner

weakness

PREFERRED RESPONSE: 4

 

    1. Following laceration, peripheral nerves heal in which manner?

      1. Direct repair across the gap by Schwann cells

      2. Direct repair of the axon

      3. Antegrade growth of the axon

      4. Retrograde growth of the axon

      5. Formation of collagen bridges by fibroblasts

      PREFERRED RESPONSE: 3

       

    2. A 7-year-old boy with short stature, bowed legs, and back pain has 2 sisters whose appearances are unremarkable. Laboratory studies show low serum phosphate; his serum calcium, and 25 hydroxycholecalciferol levels findings are within defined limits, but his 1,25 dihydroxycholecalciferol levels are below normal. The boy has a male cousin with the same disorder. What type of rickets is the most likely diagnosis?

      1. Renal

      2. Vitamin D-deficient

      3. X-linked hypophosphatemic

      4. Type I vitamin D-dependent

      5. Type II vitamin D-dependent

      PREFERRED RESPONSE: 3

       

    3. A 5-year-old boy has superficial heterotopic bone around his chest and difficulty breathing. He has one sibling who died of respiratory distress at age 13. The most likely genetic cause of this disorder is a variant in

      1. SOX9.

      2. matrilin 3.

      3. the FGF3 receptor.

      4. the BMP type I receptor.

      5. the PTH-PTHrP receptor.

       

    4. Postnatal skeletal linear growth occurs most rapidly during

      1. puberty.

      2. prepuberty.

      3. preschool years.

      4. first year of life.

      5. early elementary school years.

      PREFERRED RESPONSE: 4

       

      PREFERRED RESPONSE: 4

    5. In the setting of skeletal muscle lacerations, which effect does transforming growth factor-beta have on scar tissue formation?

      1. Stimulates the proliferation of myocytes and enhances healing

      2. Stimulates the proliferation of myofibroblasts and increases fibrosis

      3. Enhances the differentiation of myoblasts and enhances healing

      4. Decreases the proliferation of myofibroblasts and reduces fibrosis

      5. Decreases the proliferation of myocytes and reduces healing

      PREFERRED RESPONSE: 2

    6. What cell is responsible for the production of receptor activator of nuclear factor kappa beta ligand and osteoprotegerin?

      1. Osteocyte

      2. Osteoclast

      3. Osteoblast

      4. Macrophage

      5. Preosteoclast

       

    7. Which change, if any, will most likely occur within the articular cartilage of the knee as a result of nonweight bearing for 2 months?

      1. No change

      2. Cartilage thinning

      3. Decreased water content

      4. Increased collagen synthesis

      5. Increased proteoglycan concentration

      PREFERRED RESPONSE: 2

       

    8. At the time of heel strike in normal gait, what is the pattern of activation of the anterior tibialis?

      1. Relaxation

      2. Isotonic contraction until midstance

      3. Concentric contraction followed by relaxation

      4. Eccentric contraction throughout the stance phase

      5. Eccentric contraction followed by concentric muscle activity

      PREFERRED RESPONSE: 5

    9. In performing a study comparing infection rates for patients undergoing elective orthopaedic surgery who received irrigation with or without the addition of antibiotics in the solution, what is the most appropriate statistical test to determine whether the proportion of patients developing infection is significant?

      1. ANOVA

      2. Paired t test

      3. Unpaired t test

      4. Chi-square test

      5. Bonferroni correction

      PREFERRED RESPONSE: 4

    10. What component of bone is most responsible for tensile strength?

      1. Hydroxyapatite

      2. Type I collagen

      3. Type II collagen

      4. Type X collagen

      5. Tricalcium phosphate

      PREFERRED RESPONSE: 2

    11. A would improve the rate of fracture healing. The study found that patients who received Drug A had a shorter time to fracture healing (P < .05). A subsequent multicenter follow-up study with 1000 patients showed that there was no real difference in fracture healing rates when comparing the drug to the placebo(P = .25). Which kind of error occurred in the first study?

      1. Type 1

      2. Type 2

      3. Design

      4. Surgeon bias

      5. Underpowered study

       

    12. What is the etiology of fretting corrosion?

      1. Oxidation

      2. Differences in oxygen tension

      3. Micromotion at contact sites under load

      4. Impurities entering metal during manufacturing

      5. Electrochemical potential between 2 metals in a conductive medium

      PREFERRED RESPONSE: 3

       

    13. An 85-year-old woman with osteoarthritis of the knee wants to discuss treatment options for her pain.Which drug is most likely to result in serious renal impairment?

      1. Tramadol

      2. Ibuprofen

      3. Oxycodone

      4. Hydrocodone

      5. Acetaminophen

      PREFERRED RESPONSE: 2

       

       

       

    14. Figure 152 is an arthroscopic view of a reconstructive procedure performed on a 28-year-old man who had a medial femoral osteochondral lesion that measured 1.5 cm x 1.5 cm. The grafts were harvested from the superolateral aspect of the trochlea. What is the most accurate description for the process of graft incorporation?

       

      1. The bone graft will be incorporated into the subchondral bone, and the overlying cartilage remains viable.

      2. The bone graft will be incorporated into the subchondral bone, but the overlying cartilage is nonviable.

      3. The bone graft will be incorporated while the overlying cartilage is used as a scaffold to generate a new cartilage layer.

      4. The bone graft will not be incorporated; healing will occur only within the cartilage layer.

      5. The bone graft and cartilage will both be resorbed and replaced by a fibrocartilaginous matrix.

       

      PREFERRED RESPONSE: 1

       

    15. Rivaroxaban is a novel anticoagulant approved for use in thromboprophylaxis after total hip and knee arthroplasty. What is its mechanism of action?

  1. Blocks factor V

  2. Activates antithrombin III

  3. Directly inhibits thrombin

  4. Directly inhibits factor Xa

  5. Inhibits vitamin K-dependent factors

PREFERRED RESPONSE: 4

 

Q. 18.Shoulder examination under anesthesia reveals symmetric forward elevation and external rotation in the abducted position. However, external rotation with the arm by the side is decreased by 25 degrees when compared to the contralateral shoulder. Which structure is contracted?

  1. Biceps tendon

  2. Anterior-inferior capsule

  3. Anterior-superior capsule

  4. Posterior-superior capsule

  5. Acromioclavicular joint capsule

PREFERRED RESPONSE: 3

Q. 19.Rheumatoid factor as clinically measured detects antibodies to which antigen?

  1. IgG

  2. Scl-70

  3. Histone

  4. Nuclear antigens

  5. Ribonucleoprotein

 

 

PREFERRED RESPONSE: 1

 

Q. 20.Figure 179 is a recent pelvic radiograph of a 73-year-old woman who cannot ambulate. She developed blindness as an infant and has a history of difficulty with healing of limb fractures. What defect is associated with her disorder?

 

  1. PEX gene

  2. Fibrillin 1

  3. Collagen I

  4. Collagen oligomeric protein

  5. Carbonic anhydrase

 

PREFERRED RESPONSE: 5

 

 

Q.21.Figures 185a through 185c are the biopsy specimen and radiographs of a 43-year-old man with a slowly enlarging mass in his right proximal tibia. He has normal kidney function. Examination reveals a firm mass with slight tenderness. Which laboratory value is most likely elevated?

 

  1. Serum calcium

  2. Serum phosphorous

  3. Serum gamma globulin

  4. Serum creatine phosphokinase

  5. Prostate-specific antigen

 

PREFERRED RESPONSE: 1

 

Q. 22.Dessication of the nucleus pulposus results from loss or degradation of which molecule?

  1. Matrilin

  2. Proteoglycan

  3. Type I collagen

  4. Type II collagen

  5. Type X collagen

PREFERRED RESPONSE: 2

 

Q. 23.A study was undertaken to compare outcomes of 3 different treatment alternatives. The study was initiated after treatments had been completed. What is the best term for this type of study?

  1. Case-control

  2. Prospective cohort

  3. Retrospective cohort

  4. Single blinded

  5. Prospective randomized blinded and controlled

PREFERRED RESPONSE: 3

Q. 24.What is the inheritance pattern of Duchenne muscular dystrophy?

  1. Mitochondrial

  2. X-linked recessive

  3. X-linked dominant

  4. Autosomal recessive

  5. Autosomal dominant

PREFERRED RESPONSE: 2

Q. 25.What cell type is responsible for repair of cartilage defects with fibrocartilage following microfracture?

  1. Synovial fibroblasts

  2. Marrow mesenchymal cells

  3. Deep-zone chondrocytes

  4. Middle-zone chondrocytes

  5. Superficial-zone chondrocytes

PREFERRED RESPONSE: 2

Q. 26.A weightlifter is engaged in a curling exercise using a free barbell. He is focused on resisting the weight while his elbow extends. During this maneuver, he feels a snap in his right arm and drops the weight suddenly. He is seen in the emergency department with a palpable prominence in his right biceps. During which type of muscle contraction did this injury occur?

  1. Isotonic eccentric

  2. Isotonic concentric

  3. Isometric eccentric

  4. Isokinetic eccentric

  5. Isokinetic concentric

PREFERRED RESPONSE: 1

 

Q. 27.The pathologic gait shown in Video 218 is most likely attributable to

 

 

  1. hamstring weakness.

  2. quadriceps weakness.

  3. adductor spasticity.

  4. triceps surae spasticity.

  5. tibialis anterior spasticity.

PREFERRED RESPONSE: 4

 

Q. 28.Parents bring in their 8-year-old son for an evaluation. They are concerned because the boy’s paternal grandfather and uncle both died from osteosarcoma in their 40s. In addition, rhabdomyosarcoma was recently diagnosed in the boy’s first cousin. The family is requesting genetic testing to ascertain their son’s risk for developing a malignancy. Which gene is most likely responsible for the increased risk for the malignancy?

 

  1. EXT1

  2. tX,18

  3. t12,22

  4. NF1

  5. p53

PREFERRED RESPONSE: 5

 

Q. 29.Patients with factor V Leiden are at increased risk for deep vein thrombosis following surgery. What is the effect of the mutation in factor V?

 

  1. Blocks antithrombin III

  2. Increases the activity of factor Xa

  3. Prevents inhibition by activated protein C

  4. Prevents the factor Xa conversion of prothrombin to thrombin

  5. Makes it highly susceptible to inhibition by activated protein C

PREFERRED RESPONSE: 3

 

Q. 30.Infliximab (antitumor necrosis factor-alpha antibody) is associated with which adverse condition?

  1. Osteolysis

  2. Osteoporosis

  3. Hyperglycemia

  4. Atypical fracture

  5. Opportunistic infection

PREFERRED RESPONSE: 5

Q.31. Figures 270a through 270c are the MRI scans of a 65-year-old man who has pain in his right groin and buttocks. He had a metal-on-metal hip replacement 3 years ago. What is the most likely cause of his pain?

 

 

  1. Infection

  2. Lymphoma

  3. Sarcomatous change

  4. Large-particle wear debris disease

  5. Pseudotumor hypersensitivity response

 

PREFERRED RESPONSE: 5

 

Question 1..Pharmacologic prophylaxis with low-molecular-weight heparin is associated with an increased risk for

  1. polycytemia vera.

  2. thrombocytopenia.

  3. atrial fibrillation.

  4. hepatitis B transmission.

  5. interaction with other drugs.

PREFERRED RESPONSE:2

 

Question 2 Which inheritance pattern is characteristic of the disorder shown in?

 

 

  1. Autosomal-dominant

  2. Autosomal-recessive

  3. X-linked dominant

  4. X-linked recessive

  5. Mitochondrial inheritance through the maternal line

 

 

 

Question 3 ..Figures 3a and 3b are the radiographs of a 5-year-old child with midtibia swelling and no pain. What isthe most appropriate next step?

 

  1. Bone scan

  2. Observation

  3. Needle biopsy

  4. Wide excision

  5. Referral to a musculoskeletal tumor service for definitive treatment

 

PREFERRED RESPONSE:2

 

 

 

PREFERRED RESPONSE:1

 

Question 4..Renal osteodystrophy is initiated in part by which mechanism?

  1. Phosphate retention

  2. Decreased renin production

  3. Calcium-wasting in the kidney

  4. Increased serum ammonia levels

  5. Low alkaline phosphatase activity

PREFERRED RESPONSE:1

Question 5..A female patient with early degenerative disk disease inquires whether there are any new procedures thatcan reverse the degenerative process. What is the most appropriate response?

 

  1. Inform her that nothing can be done.

  2. Perform intradiskal injection of bone-marrow-derived stem cells.

  3. Perform biologic disk replacement using electropsun nanofiber scaffold.

  4. Perform biologic disk replacement using scaffold-chondrocyte-seeded construct.

  5. Discuss current research but indicate that these interventions are not yet available.

PREFERRED RESPONSE:5

Question 6..Which organism is most commonly associated with the production of a protective glycocalyx/biofilm thatallows it to adhere to surfaces and resist phagocytosis?

 

  1. Escherichia coli

  2. Proteus mirabilis

  3. Proprionobacterium acnes

  4. Staphylococcus epidermidis

  5. Mycobacterium tuberculosis

PREFERRED RESPONSE:4

Question 7..Which disorder is associated with a loss-of-function mutation in the carbonic anhydrase II gene?

 

  1. Osteopetrosis

  2. Ollier disease

  3. Achondroplasia

  4. Polyostotic fibrous dysplasia

  5. Multiple hereditary exostosis

PREFERRED RESPONSE:1

Question 8..N-telopeptide-1 is used as a marker of bone turnover in metabolic bone diseases and represents a breakdown product of which molecule?

  1. Collagen 1

  2. Osteopontin

  3. Osteocalcin

  4. Fibronectin

  5. Bone sialoprotein

 

PREFERRED RESPONSE:1

 

Question 9..Compared to titanium alloys, cold-forged cobalt chrome has which property?

  1. Increased ductility

  2. Increased ultimate strength

  3. Decreased yield strength

  4. Decreased endurance limit

  5. Decreased fatigue resistance

PREFERRED RESPONSE:2

Question 10- Aging causes progressive changes in the nucleus pulposus including

  1. increased cell density.

  2. increased water content.

  3. decreased stiffness.

  4. decreased type I collagen content.

  5. fragmentation of proteoglycans.

PREFERRED RESPONSE:5

Question 11- Which component of articular cartilage contributes the most to its high affinity for water and ability tomaintain hydrostatic pressure?

  1. Anchorin

  2. Chondrocytes

  3. Proteoglycans

  4. Collagen type I

  5. Collagen type II

PREFERRED RESPONSE:3

Question 12-The destruction of cartilage in septic arthritis is mediated by

  1. apoptosis of chondrocytes.

  2. phagocytosis of collagen fibrils by bacteria.

  3. osteoclastic resorption of subchondral bone.

  4. Panton-Valentine leukocidin secreted by bacterium.

  5. matrix metaloproteinases secreted by host cells.

PREFERRED RESPONSE:5

Question 13- Which cell most commonly initiates the inflammatory cascade associated with aseptic loosening of orthopaedic implants?

  1. Plasma

  2. Neutrophil

  3. Macrophage

  4. Helper T cell (CD4+)

  5. Killer T cell (CD8+)

PREFERRED RESPONSE:3

Question 14- Intramedullary nailing of a 12-year-old boy’s midshaft femur fracture, with a starting point in the piriformis fossa, is associated with disruption of which structure?

  1. Inferior gluteal artery

  2. Superficial circumflex iliac artery

  3. First perforating branch of the femoral artery

  4. Deep branch of the medial femoral circumflex artery

  5. Transverse branch of the lateral femoral circumflex artery

PREFERRED RESPONSE:4

 

Question 15- An activating mutation in the parathyroid hormone (PTH)/PTH related protein receptor would be expectedto have what effect on the physis?

  1. Delayed maturation in the zone of hypertrophy

  2. Accelerated maturation in the zone of hypertrophy

  3. Increased proliferation in the resting zone

  4. Increased proliferation in the zone of proliferation

  5. Increased mineralization in the zone of provisional calcification

PREFERRED RESPONSE:1

Question 16- How does Sclerostin, the SOST gene product, affect bone mass?

  1. Decreases by bone morphogenic protein (BMP) activation

  2. Increases by BMP activation

  3. Decreases by inhibiting the Wnt pathway

  4. Increases by activating the Wnt pathway

  5. Decreases by activating the Hedgehog pathway

PREFERRED RESPONSE:3

Question 17- Intermittent administration of teriparatide (parathyroid hormone peptide) as osteoporosis treatment poses potential risk for

  1. diarrhea.

  2. net bone loss.

  3. osteosarcoma.

  4. osteonecrosis of the jaw.

  5. subtrochanteric bone fractures.

PREFERRED RESPONSE:3

 

Question 18-A 23-year-old sprinter had sudden-onset posterior thigh pain in the muscle belly of the medial hamstrings.Examination revealed local tenderness and decreased knee flexion strength. An MRI scan showed edema at the musculotendinous junction, but the muscle tendon unit was in continuity. Which cell type is most responsible for muscle healing?

  1. Myoblasts

  2. Macrophages

  3. Neutrophils

  4. Satellite cells

  5. Endothelial cells

PREFERRED RESPONSE:4

 

Question 19-Which portion of the cortical bone of a long bone is vascularized by nutrient arteries?

1. 20%

2. 40%

3. 60%

4. 80%

5. 100%

 

PREFERRED RESPONSE:3

 

Question 20- What is the most appropriate definition of epigenetics?

  1. Gene mutations across large populations

  2. Creation of new mutations during meiosis

  3. Tendency of certain alleles to be inherited together

  4. Genetic alterations that do not involve DNA mutation

  5. Production of amino acid chains from nucleotide sequences

 

PREFERRED RESPONSE:4

Question21-The maximal tension generated by a given skeletal muscle is most closely predicted by

  1. length.

  2. muscle architecture.

  3. overall muscle mass.

  4. physiologic cross-sectional area.

  5. myofibril types present in the muscle.

 

PREFERRED RESPONSE:4

 

Question 22- Histologically, the superficial zone of the articular cartilage is characterized by

  1. clustering of cells.

  2. round-shaped chondrocytes.

  3. high content of proteoglycans.

  4. presence of vimentin filaments.

  5. collagen fibrils running parallel to the surface.

PREFERRED RESPONSE:5

 

Question 23- What is the most common symptom associated with pulmonary embolism?

  1. Cough

  2. Cyanosis

  3. Dyspnea

  4. Tachycardia

  5. Pleuritic chest pain

PREFERRED RESPONSE:3

 

Question 24-A 45-year-old man has intermittent elbow pain and numbness in the fourth and fifth fingers of his left hand when his elbow is flexed for more than a few minutes. Past medical history is noncontributory and he hasno known acute injury. The altered sensation is most likely attributable to

  1. axonal degeneration.

  2. loss of endoneural tube continuity.

  3. displacement of the nodes of Ranvier.

  4. mechanical disruption of the perineurium.

  5. vascular obstruction of the intraneural vessels.

 

Question 25- What is the lowest level to which the spinal cord should extend?

  1. T10

  2. T12

  3. L2

  4. L4

  5. S1

PREFERRED RESPONSE:3

 

Question 26- In children undergoing lower-extremity amputations compared to controls who do not require surgery,what is the lowest amputation level at which differences in self-selected walking speed can be detected?

 

  1. Hip disarticulation

  2. Knee disarticulation

  3. Syme’s amputation

  4. Transtibial amputation

  5. Transfemoral amputation

PREFERRED RESPONSE:2

 

Question 27- What is the most appropriate definition of an eccentric contraction?

 

  1. The muscle does not change length.

  2. The muscle fibers lengthen as the muscle contracts.

  3. There is shortening of the muscle as it contracts.

  4. There is a constant force applied through a full range of motion.

  5. The tension in the muscle remains constant despite a change in muscle length.

 

PREFERRED RESPONSE:2

 

Question 28- Weakness of the hip flexors limits limb advancement during

  1. midstance.

  2. terminal stance.

  3. swing phase.

  4. initial contact.

  5. limb-loading response.

PREFERRED RESPONSE:3

 

Question 29- Which growth plate zone is most commonly involved in slipped capital femoral epiphysis?

  1. Resting

  2. Maturation

  3. Hypertrophic

  4. Proliferative

  5. Vascular invasion

 

Question 30-Immunogenicity is highest in which type of graft?

  1. Fresh allograft

    3. Freeze-dried allograft

    2. Frozen, irradiated allograft

    4. Demineralized bone matrix

    5. Absorbable collagen hemostatic sponge

    PREFERRED RESPONSE:1

     

    Question 31- A 10-year-old boy twisted his ankle while skateboarding and has pain and swelling around the lateral ankle just distal to the fibula. Radiographs are obtained and a lesion is identified in the distal tibia as seen in Figures 273a and 273b. Two weeks later he has no pain to palpation in the region and denies antecedent pain. What is the most appropriate treatment for this lesion?

     

     

     

    1. Biopsy

    2. Naprosyn

    3. Observation

    4. Radio frequency ablation

    5. Curettage and bone grafting

    PREFERRED RESPONSE:3

     

     

     

     

     

    Question.1 .A 49-year-old weekend athlete has a 4-week history of pain in his unilateral plantar heel that is most severe for the first 20 steps upon arising in the morning. He has an area of maximal tenderness on the plantar medial aspect of the heel pad at the origin of the plantar fascia. He has only improved 30% after a 3-week course of physical therapy with toe intrinsic muscle strengthening and arch- and tendo-Achilles stretching. What is the best next treatment step?

    1. Release the plantar fascia.

    2. Inject the plantar fascia with platelet-rich plasma.

    3. Prescribe a night splint and continue physical therapy.

    4. Administer extracorporeal shockwave therapy to the heel.

    5. Perform a series of 3 steroid injections into the plantar fascia.

    PREFERRED RESPONSE: 3

     

     

     

    Question.2 .Figures 16a and 16b are the radiographs of a 38-year-old carpenter with progressively worsening ankle pain; 14 years ago, he was involved in an all-terrain vehicle collision. Anti-inflammatory medication,corticosteroid injections, and bracing no longer effectively control his pain. The pain now interferes with his work and family responsibilities. Examination reveals an antalgic limp, varus deformity, limited ankle motion, limited eversion, and normal strength. Treatment should now consist of

     

     

     

     

    1. ankle arthrodesis.

    2. total ankle arthroplasty.

    3. distal tibia osteotomy.

    4. lateral ligament repair.

    5. deltoid ligament release.

     

    Question. 3 .A 48-year-old woman had total knee arthroplasty. She is unable to “lift her toes or ankle to her nose.”After 2 months of physical therapy, she has a slapping gait. What is the best next treatment step?

    1. Ankle fusion

    2. Ankle-foot orthosis

    3. Sural nerve graft

    4. Medial heel post

    5. Laminectomy of L4/5

    PREFERRED RESPONSE: 2

     

    Question. 4 .Figures 46a through 46c are the CT scans of an 18-year-old who sustained an injury 3 weeks ago and now has ankle pain. Examination reveals an ankle effusion and painful range of motion. Recommended treatment should consist of

     

     

    1. transtalar drilling.

    2. fixation of the fragment.

    3. osteochondral autograft.

    4. weight bearing in a boot with early range of motion.

    5. cast immobilization and nonweight-bearing activity for 6 weeks.

    PREFERRED RESPONSE: 2

     

    Question. 5 .A 47-year-old woman has a closed, displaced, Weber C bimalleolar ankle fracture. Past medical history includes diabetes mellitus for 7 years controlled with diet and an oral hypoglycemic agent. Semmes-Weinstein sensory testing reveals absence of sensation to the 5.07/10-gm monofilament on the plantar aspect of both feet. The skin is intact with 2+ pedal pulses. Treatment should include

     

    1. open reduction with limited internal fixation.

    2. closed reduction and application of an external fixator.

    3. closed reduction and total contact cast immobilization.

    4. retrograde intramedullary rod fixation with ankle fusion.

    5. internal fixation and an extended period of immobilization.

     

     

     

    Question. 6 .Figures 68a and 68b are the clinical photographs of a 55-year-old woman who had a right hindfoot fusion 3 years ago for a pes planovalgus deformity. Since the surgery, she has had lateral hindfoot pain and places most of the weight-bearing load on the lateral border of her foot when walking. What is the most likely cause of her symptoms?

     

     

     

     

    1. Deltoid insufficiency

    2. Excessive forefoot abduction

    3. Residual heel valgus

    4. Residual Achilles tendon contracture

    5. Malposition of the transverse tarsal joint

     

    PREFERRED RESPONSE: 5

    Question 7 A 23-year-old hiker experienced a twisting injury to his right ankle 10 days ago. His dorsiflexion external rotation test is negative and he is able to hop on his right ankle, but he has pain over the anterior talofibular ligament. His peroneal strength is 4/5. What is the best next treatment step?

    1. A modified Brostrom procedure

    2. Rest, ice, compression, and elevation

    3. Physical therapy with proprioceptive training

    4. Casting of the right ankle in a neutral position

    5. Surgical arthroscopy of the right ankle with anterolateral ankle debridement

    PREFERRED RESPONSE: 3

    Question 8 Abnormal gait attributable to deformity after partial-foot amputation through the talonavicular and calcaneocuboid joints is the result of the unbalanced pull of which structure?

    1. Achilles tendon

    2. Anterior tibialis

    3. Posterior tibialis

    4. Peroneus brevis

    5. Flexor digitorum longus

     

     

    Midfoot osteotomy

    Reapplication of a total contact cast Achilles tendon lengthening procedure Needle biopsy for culture and sensitivity

    Application of an external bone growth stimulator

     

     

    Question 9 Figures 92a and 92b are the current radiographs of a 47-year-old man with an 8-year history of diabetes mellitus treated for 3 months with total contact casting for an erythematous, swollen, warm foot without ulceration. He has had 2 episodes of plantar ulceration that have healed with repeat total contact casting.What is the best next treatment step?

     

    1.

    2.

    3.

    4.

    5.

     

    PREFERRED RESPONSE: 1

    Question 10 Figures 106a and 106b are the radiographs of a 36-year-old woman who had hallux valgus reconstruction2 years ago. She has difficulty with shoe wear and pain with activity. Examination reveals moderate pain with range of motion and medial tightness at the hallux metatarsophalangeal joint. What is the recommended treatment?

     

     

     

    1. Keller resection arthroplasty

    2. Hallux metatarsophalangeal arthrodesis

    3. Distal metatarsal osteotomy

    4. Proximal metatarsal osteotomy with medial soft-tissue release

    5. Metatarsophalangeal reconstruction with extensor hallucis brevis transfer

    PREFERRED RESPONSE: 2

     

    Question 11 A 62-year-old woman with diabetes mellitus and neuropathy has had a plantar foot ulcer at the second metatarsal head for 2 months. Her dorsalis pedis pulse is palpable. Erythema surrounds the ulcer but there is no drainage. The metatarsal head is palpable with a cotton-tipped applicator placed in the wound.Treatment should consist of

    1. dressing changes.

    2. oral ciprofloxacin.

    3. total-contact casting.

    4. surgical debridement.

    5. transmetatarsal amputation.

    PREFERRED RESPONSE: 4

    Question 12 Figures 146a and 146b are the MRI scans of a 67-year-old tennis player who has had intermittent pain in his posterior ankle for 15 years. He felt a “pop” while playing tennis 1 week ago and now has weakness and increased pain. What is the most appropriate surgical option?

     

     

    1. A mini-open primary Achilles repair

    2. An allograft reconstruction of the Achilles tendon

    3. A primary Achilles tendon repair reinforced with xenograft tissue

    4. A peroneus longus tendon transfer with Achilles tendon repair

    5. A flexor hallucis longus tendon transfer with Achilles tendon repair

    PREFERRED RESPONSE: 5

     

    Question 13 Figures 157a through 157c are the radiographs and MRI scan of a 53-year-old woman who has had medial ankle pain and swelling for 7 months. Examination reveals a pes planovalgus deformity. The hindfoot and forefoot deformities are passively correctable. A single-limb toe raise on the affected leg reproduces her pain. Surgical treatment should consist of

     

     

    1. posterior tibialis tendon repair.

    2. posterior tibialis tendon debridement.

    3. flexor digitorum longus tendon transfer.

    4. triple arthrodesis and flexor digitorum longus tendon transfer.

    5. corrective osteotomies and flexor digitorum longus tendon transfer.

    PREFERRED RESPONSE: 5

     

    Question 14 Figures 175a and 175b are the radiographs of a 68-year-old man who has had hallux pain for several years. Corticosteroid injections and orthotics no longer provide relief. He wants to continue his daily 2-mile walk. What is the best next treatment step?

     

     

     

    1. Arthrodesis

    2. Cheilectomy

    3. Implant arthroplasty

    4. Proximal phalanx osteotomy

    5. Keller resection arthroplasty

    PREFERRED RESPONSE: 1

     

    Question 15 Figures 191a and 191b are the radiographs of an 18-year-old man who had an ankle fracture requiring open reduction and internal fixation 2 years ago. He has a progressive symptomatic ankle deformity.Surgical intervention should consist of

     

     

     

     

    1. ankle arthrodesis.

    2. total ankle arthroplasty.

    3. supramalleolar tibial osteotomy.

    4. valgus-producing calcaneal osteotomy.

    5. epiphyseodesis of the distal tibial physis.

    PREFERRED RESPONSE: 3

     

     

    Question 16 What is the foot orthosis/footwear prescription for management of the passively correctable deformity seen in Figure 217?

     

    1. Solid ankle cushion heel with lateral flare

    2. 3/8” heel lift with firm heel counter

    3. Lateral heel and lateral forefoot posting

    4. Medial heel wedge with lateral forefoot posting

    5. Metatarsal pad for global metatarsal head offloading

     

    PREFERRED RESPONSE: 3

     

    Question 17 A 67-year-old woman has a persistent foot drop 18 months after right total hip arthroplasty. Examination reveals passive ankle joint dorsiflexion to 0 degrees. Muscle strength testing results are listed below.Which treatment will provide the highest level of function?

     

    R

    L

    Anterior tibialis

    0/5

    5/5

    EHL/EDL

    0/5

    5/5

    Peroneal

    2/5

    5/5

    Posterior tibialis

    3/5

    5/5

    FHL/FDLb

    3/5

    5/5

    Gastrocsoleus

    4/5

    5/5

    Extensor hallucis longus/extensor digitorum longus Flexor hallucis longus/flexor digitorum longus

    1. Ankle foot orthosis

    2. Gastrocsoleus lengthening

    3. Jones extensor hallucis longus tendon transfer

    4. Posterior tibialis tendon transfer to the dorsum of the foot

    5. Flexor hallucis longus tendon transfer to the dorsum of the foot

    PREFERRED RESPONSE: 1

    Question18 Figures 245a through 245e are the radiographs and MRI scans of a 50-year-old ice hockey referee with a 3-year history of progressive anterolateral ankle pain, a history of multiple ankle sprains, and a fibular fracture he sustained 30 years ago. Examination reveals mild bilateral pes planovalgus feet with passive ankle joint dorsiflexion range of motion of 10 degrees and plantar flexion of 45 degrees without pain. The physician should recommend

     

     

     

    1. ankle joint arthrodesis.

    2. ankle ligament reconstruction.

    3. supramalleolar osteotomy.

    4. total ankle joint arthroplasty.

    5. corticosteroid injection into the ankle.

    PREFERRED RESPONSE: 1

     

     

    Question19 Video 252 shows the intrasurgical examination of a 36-year-old woman 3 weeks after an acute ankle sprain. Surgical treatment should include

     

    1. oblique fibular osteotomy.

    2. peroneal tendon tenodesis.

    3. calcaneofibular ligament release.

    4. lateral ankle ligament reconstruction.

    5. superior peroneal retinacular reconstruction.

     

    PREFERRED RESPONSE: 5

    Question20 Figures 264a and 264b are the radiographs of a 55-year-old woman who has pain at the first metatarsophalangeal joint that has not responded to a change in footwear and she desires surgery.The deformity has been present for 40 years, and she has painless passive range of motion at the metatarsophalangeal joint. The first metatarsophalangeal joint has 70 degrees of dorsiflexion and 20degrees of plantarflexion and the deformity is partially passively correctable. Which procedure(s) should be recommended to correct her deformity?

     

     

    1. First metatarsophalangeal arthrodesis

    2. Lapidus procedure with Akin osteotomy

    3. Distal Chevron osteotomy with Akin phalanx osteotomy

    4. Proximal first metatarsal osteotomy with modified McBride bunionectomy

    5. Double metatarsal osteotomy with proximal and distal metatarsal correction

     

    PREFERRED RESPONSE: 5

    Question21 Figures 273a through 273c are the clinical photograph, radiograph, and coronal MRI scan through the forefoot of a 24-year-old otherwise healthy man with a 1-week history of increasing pain, fevers, and swelling of his right foot. Examination reveals right groin tenderness and adenopathy. Laboratory studies show a peripheral blood leukocyte count of

    22.5X109 cells/L (reference range, 4.5 to 11X 109 cells/L),C-reactive protein of 60.3 mg/L (reference range 0.08-3.1 mg/L), and erythrocyte sedimentation rate of 15mm/h (reference range 0-20 mm/h). What is the best next treatment step?

     

     

    1. Transmetatarsal amputation

    2. Wound culture and intravenous antibiotics

    3. Incision, debridement, and fifth-toe amputation

    4. Incision, debridement, and partial fifth-ray resection

    5. Incision, soft-tissue debridement, and open packing

    PREFERRED RESPONSE: 5

    Question 1-Figures 1a and 1b are the nonweight-bearing radiographs of a 47-year-old man with a 32-year history of diabetes mellitus. He has had a red, swollen foot for 2 weeks after twisting his ankle. He reports no previous foot problems. Pedal pulses are palpable and there are no ulcerations. He cannot feel the tip of a 5.07-gram monofilament anywhere along the plantar aspect of the foot. What is the most appropriate treatment?

     

     

     

    1. Midfoot arthrodesis

    2. Total contact casting

    3. Excision of the extruded bone

    4. Debridement and bone biopsy

    5. Open reduction and internal fixation

    PREFERRED RESPONSE:2

    Question 2- Figures 2a and 2b are the radiographs and MRI scan of a 30-year-old female runner who has had significant right hallux metatarsophalangeal joint pain for 12 months. She has failed a trial of rest,immobilization, and orthotic management. She complains of pain localized to the plantar aspect of the hallux metatarsophalangeal joint. Surgical treatment should consist of

     

     

     

    1. sesamoid shaving.

    2. tibial sesamoidectomy.

    3. hallux metatarsophalangeal arthrodesis.

    4. distal chevron bunionectomy with phalanx osteotomy.

    5. metatarsophalangeal joint debridement with proximal phalanx osteotomy.

    PREFERRED RESPONSE:2

    Question 3-Figures 3a and 3b are the clinical photographs of a 35-year-old man seen 3 months after repair of an acute Achilles tendon rupture. He has no constitutional symptoms and is unable to perform a single heelrise test. The most appropriate treatment is

     

     

     

    1. swab culture of the sinus tract and appropriate oral antibiotics for 6 weeks followed by Achilles reconstruction.

    2. excision of the distal Achilles tendon with flexor hallucis longus tendon transfer to thecalcaneus followed by culture-specific intravenous antibiotics for 12 weeks.

    3. debridement of the Achilles tendon followed by culture-specific intravenous antibiotics for 6 weeks.

    4. debridement of the Achilles tendon with free-flap application and culture-specific intravenous antibiotics for 6 weeks.

    5. debridement of the Achilles tendon with turndown procedure and culture-specific intravenous antibiotics for 12 weeks.

     

    PREFERRED RESPONSE:3

     

     

    Question 4-What is the most likely etiology of the talus abnormality seen in Figure4?

     

    1. Ankle injury

    2. Osteoporosis

    3. Avascular necrosis

    4. Growth disturbance

    5. Rheumatoid arthritis

     

    PREFERRED RESPONSE:1

    Question 5-Figure 5 is the radiograph of a 45-year-old woman with a 4- to 6-week history of heel pain. Pain is exacerbated with weight bearing. Initial treatment should consist of

     

     

     

    1. a night splint.

    2. cast immobilization.

    3. corticosteroid injection.

    4. gastrocnemius stretching protocol.

    5. a plantar-fascial-specific stretching program.

     

    Question 6-A 59-year-old man with diabetic neuropathy has Eichenholtz stage 3 Charcot neuroarthropathy of the tarsometatarsal joints with collapse. Which modification of a custom-molded shoe sole will best reduce risk for ulceration at the plantar apex of the deformity?

     

    1. Mild rocker

    2. Double rocker

    3. Toe-only rocker

    4. Heel-to-toe rocker

    5. Negative heel rocker

    PREFERRED RESPONSE:2

     

    Question 7-What is the primary function of the subtalar joint during the early-stance phase of normal walking gait on a flat surface?

    1. Controls tibial advancement over the talus

    2. Locks the transverse tarsal joint to create a stable midfoot

    3. Reduces transmission of stresses to the contralateral knee and hip

    4. Allows full pelvic rotation while allowing forward foot progression

    5. Converts internal tibial rotation to foot pronation and calcaneal eversion

     

    PREFERRED RESPONSE:5

    Question 8-What is the primary pathology most commonly associated with the second-toe deformity seen in Figures8a and 8b?

     

     

    1. Plantar plate deficiency

    2. Osteonecrosis of the metatarsal head

    3. Extensor digitorum longus rupture

    4. Flexor digitorum longus contracture

    5. Proximal interphalangeal joint contracture

     

    Question 9-What is the most common impediment to successful closed reduction of the injury seen in Figures 9aand 9b?

     

     

    1. Extensor retinaculum

    2. Talonavicular joint capsule

    3. Posterior tibial tendon

    4. Peroneus teritus tendon

    5. Flexor hallucis longus tendon

     

    PREFERRED RESPONSE:3

     

     

    Question 10-A 57-year-old man who has had a foot-drop deformity since undergoing total hip arthroplasty 1 year ago has been unable to tolerate bracing despite multiple modifications. Examination reveals a passively correctable equinovarus deformity, 0/5 dorsiflexion power, 2/5 eversion power, and 5/5 plantar flexion and inversion power. The ankle can be passively dorsiflexed to 15 degrees with the knee flexed and 5 degrees with the knee extended. What is the most appropriate treatment?

     

    1. Ankle arthrodesis

    2. Posterior tibialis tendon transfer

    3. Split anterior tibialis tendon transfer

    4. Peroneus longus transfer to the first cuneiform

    5. Flexor hallucis longus transfer to second cuneiform

     

    PREFERRED RESPONSE:2

     

    Question 11-Syme’s amputation compared to transtibial amputation in patients with diabetes results in

    1. higher infection rates.

    2. decreased survival rates.

    3. more difficulty walking.

    4. more revision surgical procedures.

    5. lower energy expenditure for ambulation.

    PREFERRED RESPONSE:5

    Question 12-Figure 12 is the standing lateral radiograph of a 32-year-old man with a 16-year history of ankle pain.He and his father have high-arched feet. Foot orthoses, bracing, and corticosteroid injections no longer provide adequate relief. Examination reveals limited hindfoot motion and cavovarus alignment. What is

    the most appropriate treatment?

     

    1. Ankle arthrodesis

    2. Triple arthrodesis

    3. Subtalar arthrodesis

    4. Realignment osteotomies

    5. Tarsal coalition excision

    PREFERRED RESPONSE:3

    Question 13-Figures 13a and 13b are the sagittal T2- and axial T1-weighted MRI scans of a 23-year-old man with a3-month history of anterior ankle pain that is aggravated by activity. He had multiple ankle sprains during college. Examination revealed anterolateral tenderness, and maximum passive dorsiflexion reproduced

    the pain. Radiographic findings were normal. What is the most appropriate treatment?

     

     

     

    1. Ankle arthroscopy and debridement

    2. Drilling of the osteochondral lesion

    3. Internal fixation of the osteochondral lesion

    4. Anatomic lateral ankle ligament reconstruction

    5. Tendon transfer lateral ankle ligament reconstruction

    PREFERRED RESPONSE:1

     

    Question 14-A 30-year-old distance runner has a 6-week history of midsubstance Achilles tendon pain and swelling.After an initial period of immobilization, a course of rehabilitation was recommended. The most appropriate rehabilitation regimen includes

    1. an eccentric exercise program.

    2. a concentric exercise program.

    3. passive range of motion exercises alone.

    4. a combination of concentric and passive rehabilitation.

    5. manual manipulation without progressive strengthening.

    PREFERRED RESPONSE:1

    Question 15-Video 207 demonstrates the gait pattern of a 22-year-old man 9 months after a knee dislocation. There was 3/5 power in the lateral compartment muscles and 5/5 power in the posterior compartment muscles with no change in his motor exam since his injury.

    Treatment at this time should consist of

    1. gastrocsoleus muscle lengthening.

    2. split anterior tibial tendon transfer.

    3. peroneus longus to brevis tendon transfer.

    4. continuation of bracing until nerve function returns.

    5. posterior tibial tendon transfer through the interosseous membrane to the dorsum of the foot.

    PREFERRED RESPONSE:2

    Question 16-A 45-year-old active man with a mild cavus foot developed chronic peroneal tendinopathy that failed nonsurgical treatment. Figure 221 is the intraoperative photo of the peroneal tendons found in a sheath inferior to the peroneal tubercle. The appearance of the other peroneal tendon found anterior to the peroneal tubercle was normal. What is the best treatment option?

     

     

    1. Reconstruction of the peroneus brevis tendon with allograft semitendinosis

    2. Repair of the peroneus longus tendon and side-to-side transfer to the brevis tendon

    3. Flexor digitorum longus tendon transfer into the base of the fifth metatarsal

    4. Debridement of the peroneus longus tendon and peroneus longus to brevis tendon transfer

    5. Debridement of the peroneus brevis tendon and transfer of the distal peroneus longus to the base of the fifth metatarsal

     

    PREFERRED RESPONSE:4

    Question 17-What is the primary cause of the deformity in the weight-bearing anteroposterior and lateral radiographs seen in Figures 248a and 248b of a patient with Charcot-Marie-Tooth disease (also known as hereditary sensorimotor neuropathy)?

     

     

    1. The anterior tibialis overpowers the peroneus longus.

    2. The posterior tibialis overpowers the peroneus brevis.

    3. The peroneus longus overpowers the anterior tibialis.

    4. The peroneus brevis overpowers the posterior tibialis.

    5. The flexor hallucis longus overpowers the extensor hallucis longus.

    PREFERRED RESPONSE:3

    Question 18-Figures 18a through 256f are the radiographs and MRI scans of a 25-year-old man with a 6-week history of anterior ankle pain after beginning a plyometric training program. Initial treatment should consist of

     

     

     

    1. ankle arthroscopy and debridement.

    2. surgical fixation with cannulated screws.

    3. immobilization in a removable walking boot.

    4. cast immobilization and nonweight-bearing activity.

    5. ankle rehabilitation and proprioceptive training.

     

    PREFERRED RESPONSE:4

    Question 19-Figure 19 is the lateral weight-bearing radiograph of a 28-year-old man with a 3-week history of unrelenting heel pain after increasing his marathon training intensity. The pain never improves throughout the day. Each step he takes is painful. Examination reveals pain with medial-to-lateral compression of the

    calcaneal tuberosity. What is the most likely diagnosis?

     

     

    1. Plantar fasciitis

    2. Achilles tendinopathy

    3. Tarsal tunnel syndrome

    4. Calcaneal stress fracture

    5. Posterior tibialis tendinopathy

    PREFERRED RESPONSE:4

     

     

    Question 20-Figure 20 is the MRI scan of a 33-year-old woman with a 6-month history of ankle pain aggravated by ambulation on

    slopes and unlevel ground. She had sprained this ankle 1 year ago. Examination revealed tenderness over the posterolateral distal fibula and pain with resisted eversion. What is the mostappropriate treatment?

    1. Peroneal tendon repair

    2. Peroneal retinacular reconstruction

    3. Lateral ankle ligament reconstruction

    4. Osteochondral lesion drilling

    5. Ankle arthroscopy and debridement PREFERRED RESPONSE:1

     

     

     

     

     

    Question1 A 33-year-old woman has left index fingertip pain that is severely exacerbated by reaching movements.An intense T2 signal under the nailbed is visible on the MRI scan seen in Figure 1. What is the best treatment option?

     

    1. Tumor excision

    2. Sympathetic digital block

    3. Oral calcium channel blockers

    4. Tuft amputation with nail ablation

    5. Activity modification and glove wear

     

    PREFERRED RESPONSE: 1

    Question 2 A 24-year-old man with weakness and atrophy of the thumb for 12 months has very slight numbness on the radial side of his thumb that is constant and not progressing. He has no other hand or finger numbness.His 2-point static sensory examination is unremarkable in all digits and there is marked atrophy of the thenar muscles. His carpal tunnel provocative tests are negative. He has no symptoms on the opposite hand and otherwise is in excellent health. Which next step will most likely reveal the diagnosis?

    1. An MRI scan

    2. Muscle biopsy

    3. Carpal tunnel diagnostic injection

    4. Electrodiagnostic testing

    5. Carpal tunnel view radiograph

    PREFERRED RESPONSE: 1

     

     

    Question 3.Figures 28a and 28b are the pre- and postreduction radiographs of a finger. The rehabilitation protocol indicated is

     

    1. buddy tape and active motion.

    2. static splint in full extension.

    3. static splint in 45 degrees of flexion at the proximal interphalangeal joint.

       

    4. static splint in metacarpophalangeal joint flexion and proximal interphalangeal joint extension.

       

    5. extension block splint in 90 degrees of flexion at the proximal interphalangeal joint.

     

    PREFERRED RESPONSE: 1

    Question 4 A 27-year-old jackhammer operator has a 4-month history of hand coldness and severe ischemia that spares his thumb and index finger. Systemic illnesses have been ruled out. Doppler workup reveals aneurysmal changes, and digital subtraction arteriogram confirms the findings. Intervention should consist of

    1. excision and vein graft.

    2. surgical thrombectomy.

    3. systemic anticoagulation.

    4. intravascular fibrinolysis.

    5. interventional embolectomy.

    PREFERRED RESPONSE: 1

    Question 5 A mechanic sustained a high-pressure injection of cleaning solvent into the tip of his index finger 2 hours ago. The finger has good capillary refill and his 2-point discrimination is 7 mm. Initial treatment should include

    1. a corticosteroid injection.

    2. elevation and observation.

    3. elective surgical treatment within 7 days.

    4. oral clindamycin for 10 days.

    5. emergent surgical debridement.

    PREFERRED RESPONSE: 5

     

    Question6 An otherwise healthy 42-year-old woman is scheduled for carpal tunnel release. The physician should adhere to routine sterility protocols

    1. without local or systemic antibiotics.

    2. and irrigate with cefazolin solution.

    3. and irrigate with bacitracin solution.

    4. and administer cefazolin within 1 hour before incision.

    5. and administer cefazolin within 1 hour before incision and continue dosing up to 23 hours after surgery.

    PREFERRED RESPONSE: 1

    Question7 Figures 65a and 65b are the magnetic resonance arthrogram and wrist arthroscopic photograph of a 25-year-old man who has wrist pain during extension and ulnar rotation. Treatment should consist of

     

     

     

    1. synovectomy.

    2. ulnar shortening osteotomy.

    3. diagnostic arthroscopy only.

    4. triangular fibrocartilage complex tear debridement.

    5. triangular fibrocartilage complex repair dorsal ligament.

     

    PREFERRED RESPONSE: 4

     

    Question 8 .The least gliding resistance for a flexor tendon laceration at the thumb palmar-digital crease as shown inFigure 75 can be achieved with

     

    1. a 6-strand repair.

    2. a division and repair.

    3. debridement of the partial laceration.

    4. no debridement, motion therapy only.

    5. multiple-strand core repair with epitendinous repair.

     

     

     

    PREFERRED RESPONSE: 3

    Question.9 . A 42-year-old woman has the injury shown in Figures 78a and 78b. The decision to treat the ulnar styloid surgically is based upon which finding?

     

     

     

    1. Patient age

    2. Displacement of the radius fracture

    3. Displacement of the ulnar styloid fracture

    4. Position of the ulnar styloid after open reduction and internal fixation of the radius

    5. Stability of the distal radioulnar joint after open reduction and internal fixation of the radius

    PREFERRED RESPONSE: 5

     

     

     

    Question10 A 45-year-old man has lateral elbow pain that worsens with weightlifting and pushups. Video 88 shows a dynamic examination of the elbow. The anatomic defect is in which ligament?

     

    1. Annular

    2. Transverse medial collateral

    3. Anterior band of the medial collateral

    4. Posterior band of the medial collateral

    5. Lateral ulnar collateral

     

    PREFERRED RESPONSE: 5

    Question11 The deformity caused by long-term arthritis of the first carpometacarpal joint of the hand often leads to a secondary hyperextension arthrosis of which joint?

    1. Midcarpal

    2. Radiocarpal

    3. Scaphotrapezotrapezoidal

    4. Thumb interphalangeal

    5. Thumb metacarpophalangeal

    PREFERRED RESPONSE: 5

     

    Question12 Figure 103 is the clinical photograph of a 62-year-old man with numbness and weakness that has been progressing for 10 years. What is the most appropriate treatment to improve thumb function?

     

     

     

    1. Functional splinting

    2. Neurotization of the thenar muscles

    3. Hypothenar muscle transfer to thumb intrinsic

    4. Arthrodesis of the thumb carpometacarpal joint in abduction

    5. Transfer of the extensor indicis proprius around the ulnar wrist

    PREFERRED RESPONSE: 5

     

    Question13 A 50-year-old man sustained a clavicle fracture after a motorcycle collision. He has no sensation or motor function in the biceps and triceps; however, he has very weak thenar and finger flexion and extension.Which finding would suggest a postganglionic as opposed to a preganglionic injury?

    1. Preservation of C8, T1 function

    2. Preserved sensory nerve action potential

    3. Pseudomeningocele on CT myelogram

    4. Ptosis and miosis on the same side as the injury

    5. The cervical paraspinal muscle is normal on electromyography

    PREFERRED RESPONSE: 5

    Question14 Figures 133a and 133b are the clinical photographs of a 34-year-old woman with increasing pain in her index finger for 3 days. The pain is worse with passive extension.

    Appropriate treatment should consist of

     

     

    1. observation.

    2. hand therapy.

    3. oral antibiotics.

    4. intravenous antibiotics.

    5. irrigation and debridement of the flexor tendon sheath.

     

    PREFERRED RESPONSE: 5

    Question15 A 38-year-old woman had a distal radius fracture treated with a short-arm cast 3 months ago. The fracture healed in good alignment. Figure 140 shows her attempt to extend her thumb. What is the best treatment option?

     

     

    1. Static splinting

    2. Dynamic splinting

    3. Transfer of the extensor pollicis brevis

    4. Transfer of the extensor indicis proprius

    5. Arthrodesis of the interphalangeal joint

    PREFERRED RESPONSE: 4

     

    Question16 Figures 213a and 213b are the clinical photograph and biopsy specimen of a 65-year-old man with a lesion under his thumbnail that was biopsied by a dermatologist.

    Appropriate treatment should consist of

     

     

    1. observation.

    2. local excision.

    3. marginal excision.

    4. thumb ray resection.

    5. amputation at the interphalangeal joint.

    PREFERRED RESPONSE: 5

     

    Question17 Figure 225 is the clinical photograph of a 26-year-old man who fell through a window and sustained a laceration to his thumb 5 days ago. He is unable to flex his thumb. Treatment should include

     

     

    1. palmaris longus tendon transfer.

    2. reconstruction with a palmaris longus free tendon bridge graft.

    3. direct repair of the flexor pollicis longus with core sutures only.

    4. repair of the flexor pollicis longus with core and epitendinous sutures.

    5. transfer of the flexor digitorum superficialis of the ring finger to the thumb.

    PREFERRED RESPONSE: 4

     

    Question18 Figures 230a through 230d are the pre- and postreduction radiographs of a 6-year-old boy who had a fracture of the radius and ulna shafts in the distal diaphyses.

     

     

    Successful reduction of the completely displaced fractures is achieved. To best maintain reduction while minimizing complications, treatment should include immobilization in a

     

    1. removable splint.

    2. sugar-tong splint.

    3. short-arm cast.

    4. long-arm cast.

    5. long-arm thumb spica cast.

     

    PREFERRED RESPONSE: 3

     

    Question19 Figure 240 is the clinical photograph of a 33-year-old man who sustained a thumb pulp injury. There is 1-cm necrosis and tissue loss at the distal flap edge. What is the most appropriate treatment option?

     

     

    1. Cross-finger flap

    2. Volar advancement flap

    3. Free microvascular pulp reconstruction

    4. Local wound care with wet-to-moist dressings

    5. Local rotation flap from the dorsal metacarpal vessels

    PREFERRED RESPONSE: 2

    Question20 An 84-year-old patient who has been hospitalized for pneumonia has developed isolated wrist pain and swelling with an effusion. The wrist is aspirated, the nucleated cell count is 75,000 cells/mm3, and urate crystals are identified. What is the most important next treatment step?

    1. Begin allopurinol.

    2. Begin nonsteroidal anti-inflammatory drugs.

    3. Administer a corticosteroid wrist injection.

    4. Obtain cultures and begin empiric antibiotics.

    5. Obtain radiographs to evaluate for a wrist fracture.

    PREFERRED RESPONSE: 4

    Question21 Figures 269a and 269b are the MRI scans of a 60-year-old man who has pain and loss of elbow flexion strength. In addition to the distal biceps tendon injury, what is the most likely diagnosis?

     

     

     

    1. Soft-tissue sarcoma

    2. Intraneural ganglion cyst

    3. Denervation of the biceps muscle

    4. Benign peripheral nerve sheath tumor

    5. Malignant peripheral nerve sheath tumor

    PREFERRED RESPONSE: 4

     

    Question 1- For an otherwise healthy carpenter with symptoms of dominant-hand ulnar-sided pain, hand pallor, and coolness, the preliminary work-up should include

    1. a CT scan.

    2. a MRI scan.

    3. a bone scan.

    4. sonography.

    5. angiography.

    PREFERRED RESPONSE:4

    Question 2-A contraindication for attempting a replantation by an experienced surgeon in an appropriately equipped facility includes

    1. amputation through flexor zone I.

    2. major limb amputation in a child.

    3. absence of athrosclerotic disease.

    4. crush or avulsion mechanism of amputation.

    5. sharp transection of the thumb at the metatarsophalangeal joint level.

    PREFERRED RESPONSE:4

     

    Question 3- Figure 3 is the clinical photograph of a 12-year-old boy with a learning disability who was riding a bicycle without wearing gloves for about 10 minutes when the temperature was 10°F. He arrived at the emergency department with painful fingers that were warm and perfused without ischemia. Which of the following treatments is recommended?

     

     

     

    1. Escharotomy

    2. Surgical debridement of the blisters

       

    3. Surgical debridement of the blisters and skin grafting

       

    4. Administration of subcutaneous calcium gluconate

       

    5. Rewarming of the fingers in a warm water bath at 104°F

     

    PREFERRED RESPONSE:2

     

    Question 4- Figure 4 is the sagittal CT scan of a 45-year-old woman who injured her wrist after a fall. Appropriate treatment of the fracture should include

     

     

    1. cast immobilization.

    2. closed reduction and percutaneous pin fixation.

    3. open reduction and volar locking plate fixation.

    4. open reduction and radial styloid column plating.

    5. open reduction and internal fixation of the volar fracture fragment.

     

    PREFERRED RESPONSE:5

    Question 5-Figure 5 features the radiographs of a 22-year-old man. The most appropriate treatment is

     

     

    1. observation.

    2. cast treatment.

    3. buddy tape and active motion.

    4. closed reduction and pinning.

    5. open reduction and internal fixation.

    PREFERRED RESPONSE:4

     

    Question 6-Figure 6 is a T2-weighted MRI scan of a 64-year-old man who has had a right volar radial mass for the past 2 years. What is the most likely diagnosis?

     

     

     

    1. Lipoma

    2. Ganglion

    3. Schwannoma

    4. Radial artery aneurysm

    5. Giant-cell tumor of tendon sheath

     

    PREFERRED RESPONSE:2

     

    Question 7- Figures 89a through 89c are the radiographs of a 66-year-old man who fell 1 week ago. Examination revealed rotational deformity of his index finger. Treatment should consist of reduction and

     

     

     

    1. casting.

    2. buddy taping.

    3. plate fixation.

    4. intramedullary fixation.

    5. interfragmentary fixation.

    PREFERRED RESPONSE:5

    Question 8-The wrist arthroscopy portal that places a subcutaneous sensory nerve at most risk is

    1. 1-2.

    2. 3-4.

    3. 4-5.

    1. ulnar midcarpal.

    2. radial midcarpal.

     

    PREFERRED RESPONSE:1

    Question 9-A 22-year-old man is unable to raise his arm above shoulder level during forward flexion since being involved in a motorcycle collision 4 months ago. Examination revealed scapular winging on forward flexion of the shoulder. Electromyography confirmed serratus anterior muscle palsy. The nerve involved

    branches off from cervical roots

    1. C3-4.

    2. C4-5.

    3. C5-7.

    4. C6-8.

    5. C7-T1.

    PREFERRED RESPONSE:3

     

    Question 10-A 32-year-old man has thromboangiitis obliterans (Buerger disease). When counseling about treatment options, the physician should advise him to stop smoking to

    1. resolve symptoms.

    2. improve nail growth.

    3. eliminate the need for surgery.

    4. reduce the incidence of amputation.

    5. promote growth of new blood vessels.

    PREFERRED RESPONSE:4

     

    Question 11-Figures 11a through 11c are the radiograph and clinical photographs of a 15-year-old left-handdominant boy who amputated his left hand through the midcarpal joint with a saw. The hand and patient arrived within 1 hour of the injury. The hand was wrapped in a moist saline dressing, put in a plastic bag,and placed in a cooler on top of ice. The next treatment step should include

     

    1. replantation of the hand.

    2. free-flap coverage of the wound.

    3. radial artery flap coverage of the wound.

    4. revision of the amputation wound to a distal forearm amputation.

       

    5. revision of the amputation at the wrist to preserve the distal radioulnar joint.

     

     

     

    PREFERRED RESPONSE:1

     

    Question 12-Coverage in a patient who sustains a thumb-tip injury with 2.5-cm pulp skin loss is best achieved with

    1. skin graft.

    2. direct closure.

    3. free soft-tissue transfer.

    4. volar advancement flap.

    5. island volar advancement flap.

    PREFERRED RESPONSE:5

     

    Question 13-Figures 13a through 13c are the radiographs of a 38-year-old man with a previous history of finger and hand infection treated with amputation 2 years ago. While there has been no subsequent history of redness or drainage, he now has difficulty using his hand, especially when grasping objects such as coins.

    Examination revealed a gap between his index and ring fingers, and the index finger rotated toward the ring. Treatment should consist of

     

     

     

     

    1. cosmetic prosthetic fitting.

    2. functional prosthetic fitting.

    3. index metacarpal transposition.

    4. metacarpal derotational osteotomy.

    5. free toe-to-hand microvascular flap.

     

    PREFERRED RESPONSE:3

     

     

     

    Question 14-Figures is the radiograph of a 68-year-old man with dorsal radial wrist pain. He rated his pain as 8 on the 0-10 Numeric Pain Rating Scale and said that his pain has bothered him constantly despite splinting,steroid injections, and administration of nonsteroidal anti-inflammatory drugs. Surgical treatment for

    wrist pain should consist of

     

    1. scaphoidectomy.

    2. radial styloidectomy.

    3. proximal row carpectomy.

    4. complete wrist arthrodesis.

    5. four-corner fusion with scaphoidectomy.

     

    PREFERRED RESPONSE:5

     

    Question 15-In zone II flexor tendon lacerations, repairing only 1 slip compared to repairing both slips of the flexor digitorum sublimis results in

    1. a higher rupture rate.

    2. profundus bowstringing.

    3. improved tendon gliding.

    4. improved passive range of motion.

    5. proximal interphalangeal joint hyperextension.

    PREFERRED RESPONSE:3

     

     

    Question 16-Figure is the clinical photograph of a 70-year-old woman with squamous cell cancer on her thumb.Resection and reconstruction is planned and requires soft-tissue coverage. Thumb region coverage is best obtained with

     

    1. the Moberg flap.

    2. a third dorsal metacarpal artery flap.

    3. a first dorsal metacarpal artery flap.

       

    4. a full-thickness skin grafting.

       

    5. a reverse cross-finger flap from the index finger with full-thickness skin grafting.

     

    PREFERRED RESPONSE:3

     

     

     

    Question 17-Figure is the MRI scan of a 54-year-old woman who fell on her outstretched hand and now has dorsal wrist pain. The structure indicated by the white line is the dorsal

     

    1. wrist ganglion.

    2. radioulnar ligament.

    3. radiocarpal ligament.

    4. intercarpal ligament.

    5. scapholunate ligament.

     

    PREFERRED RESPONSE:4

     

    Question 18-During the preoperative evaluation of a man with Dupuytren’s disease who is about to undergo partial fasciectomy, it is noted that he has a contracture at the metacarpophalangeal joint level with a pit in the skin denoting a possible `spiral cord.` This cord displaces the neurovascular bundle in which direction?

     

    1. Dorsal

    2. Medial

    3. Midline

    4. Dorsolateral

    5. Midline and volar

    PREFERRED RESPONSE:5

     

    Question 19-A 42-year-old woman has had right wrist pain for 2 years. She tried splint wear and naproxen and has had 3 steroid injections, each time experiencing less relief.

    Examination revealed tenderness at and just proximal to the radial styloid, with pain exacerbated with thumb flexion and wrist ulnar deviation. What

    is the best next step in treatment?

     

    1. Physical therapy

    2. Continued splint wear

    3. Repeat injection into the first dorsal wrist compartment

    4. Incision of the first dorsal wrist compartment at the volar edge

    5. Incision of the first dorsal wrist compartment at the dorsal edge

     

    PREFERRED RESPONSE:5

     

     

     

    Question 20-Figure 262 is the video of a 20-year-old man who sustained a striking injury to the right middle finger metacarpophalangeal joint 1 week ago. Initial treatment should consist of

     

    1. observation.

    2. an immediate return to sports.

    3. a metacarpophalangeal joint extension brace.

    4. tendon transfer of the extensor indicis proprius.

    5. acute repair of the extensor tendon sagittal hood.

     

    PREFERRED RESPONSE:3

     

    Question 21-A 37-year-old woman has a 2-month history of weakness in thumb and finger extension, but has normal radial deviation during extension of the wrist. An MRI scan of her forearm shows no abnormality. She does not recall any traumatic event. Needle electromyography findings show fibrillations and reduced

    recruitment in the extensor pollicis longus, abductor pollicis longus, extensor digitorum communis, and extensor carpi ulnaris muscles. Which nerve is most likely compressed?

     

    1. Median

    2. Radial

    3. Anterior interosseous

    4. Posterior interosseous

    5. Lateral antebrachial cutaneous

     

    PREFERRED RESPONSE:4

     

     

     

     

     

    Question 1 The ability of bacteria to adhere to orthopaedic implants and elude antimicrobial therapies through the use of biofilm is attributable to their ability to produce

    1. pyrrolidonyl arylamidase.

    2. virulence factor exotoxin A.

    3. Panton-Valentine leukocidin.

    4. exopolysaccharide glycocalyx.

    5. glyceraldehyde-3-phosphate dehydrogenase.

    PREFERRED RESPONSE: 4

     

    Question2 Figures 20a and 20b are the radiograph and MRI scan of a 58-year-old man who had total hip arthroplasty 3 years ago. His hip has been increasingly painful for 6 months.

    Laboratory studies show an erythrocyte sedimentation rate of 24 mm/h (reference range [rr], 0-20 mm/h) and a C-reactive protein level of 0.3mg/L (rr, 0.08-3.1 mg/L). In Figure 20b, which abnormality is indicated by the arrows?

     

     

     

    1. Infection

    2. Malignancy

    3. Pseudotumor

    4. Polyethylene debris

    5. Heterotopic ossification

    PREFERRED RESPONSE: 3

     

    Question3 Which population is least likely to receive total joint arthroplasty?

    1. Black men

    2. Black women

    3. White men

    4. White women

    5. Hispanic men

     

    PREFERRED RESPONSE: 1

     

    Question 4 A 70-year-old healthy man had total knee arthroplasty 18 years ago, and it now is painful. Radiographs reveal aseptic loosening and the range of motion before surgery is 15 to 85 degrees. The strongest indication for performing a tibial tubercle osteotomy to aid in exposure in his knee would be

     

    1. patella baja.

    2. nonresurfaced patella.

    3. isolated femoral revision.

    4. noncemented tibial component.

    5. previous use of the quadriceps turn-down technique.

    PREFERRED RESPONSE: 1

     

     

     

    Question 5 Figure 50 is the radiograph of a 45-year-old man who has avascular necrosis of the hip attributable to his sickle cell anemia. He is scheduled for total hip arthroplasty. To prevent the most likely intrasurgical technical complication, particular attention should be directed toward

     

    1. dislocating the hip.

    2. preparing the femur.

    3. reaming the acetabulum.

    4. inserting the acetabular screws.

    5. cutting the short external rotators.

     

    PREFERRED RESPONSE: 2

     

    Question6 A 63-year-old woman with rheumatoid arthritis is undergoing a knee arthroplasty. Her rheumatoid arthritis has been well controlled with methotrexate, etanercept, and naproxen. Which medication-related instructions should be followed 7 days before surgery?

     

    1. Continue all medications

    2. Discontinue naproxen

    3. Discontinue naproxen and etanercept

    4. Discontinue naproxen and methotrexate

    5. Discontinue naproxen, etanercept, and methotrexate

     

    PREFERRED RESPONSE: 3

     

    Question7 Figures 76a through 76c are the anteroposterior and lateral radiographs and bone scan of a 66-year-old man with type I diabetes mellitus who had revision right total knee arthroplasty for aseptic loosening 3years ago. He has pain over the proximal tibia with startup and at the end of the day. He has difficulty walking on level ground. Laboratory studies reveal an erythrocyte sedimentation rate of 5 mm/h(reference range [rr], 0-20 mm/h) and C-reactive protein of <3.0 mg/L (rr, 0.08-3.1 mg/L). Synovial fluid has 389 nucleated cells with 11% neutrophils and cultures are negative. What is the most likely failure mechanism for this revision total knee arthroplasty?

     

     

    1. Unrecognized fungal infection

    2. Improper component alignment

    3. Posterior cruciate ligament insufficiency

    4. Aseptic loosening because of inadequate diaphyseal fixation

    5. Aseptic loosening because of inadequate metaphyseal fixation

    PREFERRED RESPONSE: 5

    Question8 When templating total hip arthroplasty, which figure reveals the best recreation of the proper biomechanicsof the hip joint, assuming that the right leg is 5 mm shorter than the left?

     

     

    1. Figure 98a 2. Figure 98b 3. Figure 98c 4. Figure 98d 5. Figure 98e

     

    PREFERRED RESPONSE: 3

     

    Question9 Internal rotation of the femoral component can cause patella maltracking by

    1. increasing the Q angle.

    2. increasing the medial-directed force vector on the patella.

    3. producing valgus malalignment.

    4. tightening of the lateral retinaculum.

    5. overstuffing the patellofemoral compartment.

    PREFERRED RESPONSE: 1

    Question10 A 70-year-old man with osteoarthrosis is scheduled to undergo total knee arthroplasty. He inquires about patellar resurfacing. He should be told that a potential advantage of having the patella resurfaced as opposed to leaving the patella unresurfaced is

    1. increased extensor strength.

    2. lower risk for patellar fracture.

    3. lower risk for requiring reoperation.

    4. lower risk for patellar subluxation.

    5. higher chance of achieving desirable range of motion.

    PREFERRED RESPONSE: 3

    Question11 Figures 121a and 121b are the current radiographs of a 39-year-old woman who had left total hip arthroplasty 1 year ago. She is experiencing squeaking from the left hip while ambulating. Which factor most likely contributes to her symptoms?

     

     

    1. Activity level

    2. Surgical approach

    3. Component design

    4. Component loosening

    5. Component positioning

    PREFERRED RESPONSE: 5

    Question12 What is the optimal treatment for a Vancouver type B2 fracture in a healthy patient?

    1. Retain the stem and fracture fixation with cortical strut graft and cables

    2. Revision to a proximal femoral-replacing stem

    3. Revision to a long porous-coated stem and cable fixation

    4. Revision to a long cemented stem bypassing the fracture site

    5. Revision to a proximally coated stem and open reduction and internal fixation of the fracture

     

    PREFERRED RESPONSE: 3

    Question13 The failure of total hip arthroplasty using a zirconium-ceramic femoral head as seen in Figures 153a and 153b is most likely the result of

     

     

     

     

     

    1. infection.

    2. aseptic loosening.

    3. bony impingement.

    4. material properties.

    5. component alignment.

     

    PREFERRED RESPONSE: 4

     

    Question14 Which figure best shows the femoral component loosening?

     

     

     

    1. Figure 164a

    2. Figure 164b

    3. Figure 164c

    4. Figure 164d

    5. Figure 164e

     

    PREFERRED RESPONSE: 1

     

    Question15 A 57-year-old woman had right total knee arthroplasty for varus gonarthrosis. Before surgery, her range of motion was 5 to 110 degrees. At skin closure, her range of motion was 0 to 120 degrees. Her range of motion at 10 weeks after surgery is 0 to 70 degrees. What is the best next treatment step?

     

    1. Observation

    2. Dynamic bracing

    3. Manipulation under anesthesia

    4. Revision with open adhesiolysis

    5. Physical therapy with aggressive range of motion

     

    PREFERRED RESPONSE: 3

     

    Question.16 . When comparing the results of cemented all-polyethylene tibial components to metal-backed components,the all-polyethylene tibia

    1. is more expensive.

    2. is more susceptible to fracture.

    3. is associated with an elevated risk for polyethylene wear.

    4. has an equivalent rate of aseptic loosening.

    5. has higher failure rates when used in patients younger than age 70.

    PREFERRED RESPONSE: 4

     

    Question. 17 . When the liquid monomer (monomethacrylate) is added to polymer powder (polymethylmethacrylate),the activator in the liquid monomer (N,N-Dimethyl-p-toluidine) comes in contact with the initiator in the polymer powder and polymerization is initiated. What is the initiator?

    1. Hylamer

    2. Polystyrene

    3. Barium sulfate

    4. Benzoyl peroxide

    5. Zirconium dioxide

    PREFERRED RESPONSE: 4

     

     

     

    Question.18 . Figure 197 is the radiograph of a 62-year-old woman who is seen in the emergency department with a dislocated left total hip arthroplasty. This is her seventh dislocation during the last 3 months and she most recently had a liner revision. What is the best next treatment step?

     

    1. Skeletal traction

    2. Open reduction

    3. Closed reduction

    4. Component revision

    5. Hip abduction orthosis

     

    PREFERRED RESPONSE: 4

     

     

     

    Question19 Figure 214 is the current radiograph of a 74-year-old man who had right total hip arthroplasty 3 weeks ago. He stumbled and has increasing pain with weight-bearing activity. What is the best next treatment step?

     

    1. Revision

    2. Resection arthroplasty

    3. Routine follow-up at 3 months

    4. Open reduction and internal fixation

    5. Nonweight bearing activity for 6 weeks

     

    PREFERRED RESPONSE: 1

     

    Question20 Figure 234a is the clinical photograph of an 82-year-old man who had left total knee arthroplasty 1 year ago. He has difficulty with pain and stiffness and recently noted swelling on the medial side. He had aspiration of the knee 1 month ago with a cell count of 22,000/mm3 nucleated cells. Aerobic and anaerobic culture and gram stain findings are negative. Laboratory studies reveal the erythrocyte sedimentation rate and C-reactive protein are within defined limits. He is able to perform a straight-leg raise. Range of motion is 15 to 80 degrees. Anteroposterior and lateral radiographs are shown in Figures 234b and 234c.

    What is the best next step?

     

     

    1. An MRI scan to evaluate for possible vastus medialis oblique disruption

    2. Physical therapy with biofeedback focusing on gentle range of motion

    3. Reaspirate and send for aerobic, anaerobic, fungal, and acid fast bacilli cultures

    4. Resection arthroplasty and placement of vancomycin and gentamicin cement spacer

    5. Revision total knee arthroplasty, elevation of joint line for flexion contracture, repair of the extensor mechanism disruption

     

    PREFERRED RESPONSE: 3

    Question 21 A woman has activity-related right knee pain that is located medially and is sharp in nature. Radiographs reveal medial compartment degenerative changes. She recently lost 40 pounds (intentionally) and has had some improvement in symptoms. What other nonsurgical treatment modality has the best evidence for your recommendation?

    1. Acupuncture

    2. Valgus off-loader brace

    3. Quadriceps strengthening

    4. Intra-articular cortisone injection

    5. Intra-articular viscosupplementation injection

    PREFERRED RESPONSE: 3

    Question22 Figure 253 shows the fracture sustained by an otherwise healthy 61-year-old man who was knocked down by an automobile door that was suddenly opened as he was riding his bicycle. Which treatment will most likely provide him with the best long-term function?

     

     

    1. Hemiarthroplasty

    2. Total hip arthroplasty

    3. Open reduction and internal fixation with a blade plate

    4. Open reduction and internal fixation with a dynamic hip screw

    5. Closed reduction and percutaneous cannulated screw fixation

     

    PREFERRED RESPONSE: 2

     

    Question23 Figure 259 is the radiograph of an 85-year-old man who had hip arthroplasty 15 years ago. He is now living in a nursing home, ambulating with a walker, and has dementia. During the past 3 months, his hip,which had been previously stable, has dislocated 3 times.

    What is the most likely cause of the recurrent dislocations?

     

     

     

    1. Polyethylene wear

    2. Small-diameter femoral head

    3. Damage to the locking mechanism of the liner

    4. Insufficient anteversion of the acetabular cup

    5. Failure to comply with hip dislocation precautions

     

    PREFERRED RESPONSE: 1

     

    Question24 Figures 272a through 272c are the current radiographs and CT reconstruction scan of a 58-year-old woman who has increasing pain with household ambulation. An intrasurgical video is shown in Figure 272e. After undergoing treatment as seen in Figure 272d, what is the most likely complication?

     

     

     

    1. Infection

    2. Instability

    3. Nonunion

    4. Aseptic loosening

    5. Periprosthetic fracture

     

    Question 1- Figures a and b are the radiographs of a man with an 8-month history of pain and deformity since knee replacement. He has not had any systemic symptoms and his erythrocyte sedimentation rate and C-reactive protein levels are within defined limits. The most appropriate next step should consist of

     

     

     

    1. revision total knee.

    2. formal physical therapy.

    3. bracing with a hinged knee brace.

    4. aspiration and culture of the synovial fluid.

    5. reconstruction of the lateral collateral ligament.

    PREFERRED RESPONSE:1

     

    Question 2-At a mean follow-up of 30 years, what are the causes of failure requiring revision of the Charnley lowfriction arthroplasty (most frequent to least frequent)?

     

    1. Deep infection, acetabular, both component, femoral

    2. Acetabular, deep infection, both component, femoral

    3. Acetabular, both component, deep infection, femoral

    4. Both component, deep infection, acetabular, femoral

    5. Femoral, acetabular, deep infection, both component

     

    Question 3-Figures a and b are the radiographs of a 36-year-old man with sickle cell anemia who has an 8-month history of right groin and knee pain that started after tripping and stepping hard onto his right leg. The radiographic changes are most likely caused by

     

     

     

    1. trauma.

    2. infection.

    3. hemarthroses.

    4. vascular occlusion.

    5. collapse of an enchondroma.

    PREFERRED RESPONSE:4

     

    Question 4-A 71-year-old farmer had right total hip arthroplasty 2 years ago. He did well for 6 months and then developed pain in his right hip. He has pain when rising from a seated position for the first few steps and after a full day of activities. He denies fevers or chills and the incision healed well. Laboratory studies show an erthyrocyte sedimentation rate of 8 mm/h (reference range, 0-20 mm/h), C-reactive protein of 3.0 mg/L (reference range, 0-3.0 mg/L), and white blood cell count of 7.7/mm3 (reference range, 3.5-10.5/mm3). No fluid was obtained during aspiration. Figures 50a and 50b show the immediate postoperative radiographs and Figures 50c and 50d reveal the most recent radiographs. What is the best definitive management option?

     

    1. Revision of the femoral and acetabular component

    2. Revision of the femoral component to a long cemented revision stem

    3. Revision of the femoral component to an uncemented diaphyseal engaging stem

    4. Irrigation and debridement with the femoral head and liner exchange

    5. Resection with placement of the antibiotic spacer, 2-stage reimplantation

     

     

     

     

    PREFERRED RESPONSE:3

     

    Question 5-A 61-year-old woman has pain in her right hip after a fall. She had a right total hip arthroplasty 15 years ago for osteoarthritis secondary to dysplasia. Figures a andb are radiographs taken after the fall.What is the best treatment option?

     

     

     

    1. Hemiarthroplasty

    2. Twelve weeks of nonweight-bearing activity

    3. A large hemispherical cup

    4. A posterior plate and porous metal cup

    5. Medial cancellous grafting with a hemispherical cup

    PREFERRED RESPONSE:4

     

    Question 6-During implantation of a tapered, proximally coated femoral stem, the stem implant subsided during final impaction and a 1-cm longitudinal split was discovered in the calcar. What is the best treatment option?

    1. Stem removal, cabling, and reinsertion

    2. Stem removal and use of a cemented stem

    3. Conversion to a long, revision full-coat stem

    4. Cable placement around the stem in the subsided position

    5. Leave the stem in place and order nonweight-bearing activity for 6 weeks

    PREFERRED RESPONSE:1

    Question 7-The implant shown in Figures a and b was one of the earliest attempts to manufacture a total knee arthroplasty. What most likely led to its early mechanical failure?

     

     

     

    1. Lack of high flexion

    2. Metal-on-metal wear debris

    3. Loosening attributable to overconstraint

    4. Wear of polyethylene irradiated in air

    5. Absence of a patella-femoral resurfacing

     

    PREFERRED RESPONSE:3

     

    Question 8-During revision total knee arthroplasty, difficulty was encountered while attempting to gain adequate exposure. The medial parapatellar arthrotomy was extended proximally and a lateral release was performed. The tibia was externally rotated with a medial release. Exposure of the knee was still not adequate to safely perform the revision. What is the best option to enhance exposure?

     

    1. Quadriceps snip

    2. Patellar eversion

    3. V-Y turndown of the extensor mechanism

    4. Tibial tubercle osteotomy with internal fixation

    5. Patellar turndown (modified Coonse-Adams approach)

    PREFERRED RESPONSE:1

    Question 9-The center of rotation of the knee can be best described as

    1. remaining fixed during flexion.

    2. moving posterior as flexion increases.

    3. shifting forward and then back with flexion.

    4. shifting posterior on the lateral side with flexion.

    5. sliding forward on the medial side with increasing flexion.

    PREFERRED RESPONSE:4

     

     

     

    Question 10-If the hip seen in Figure is inserted as templated, what is the expected outcome compared to the native hip?

     

    1. Offset increased; leg length increased

    2. Offset increased; leg length unchanged

    3. Offset unchanged; leg length increased

    4. Offset decreased; leg length unchanged

    5. Offset decreased; leg length decreased PREFERRED RESPONSE:1

    Question 11- What is considered to be an absolute indication for patellar resurfacing during total knee arthroplasty?

    1. Patella thickness is 16 mm.

    2. Patella diameter is 22 mm.

    3. A patient’s BMI is >40.

    4. A patient has rheumatoid arthritis.

    5. An eburnated trochlea is present.

     

    PREFERRED RESPONSE:4

     

    Question 12- Figures a and b are the radiographs of a 55-year-old man with left hip pain, popping, and grinding since having a metal-on-metal left total hip arthroplasty 18 months ago. He has had 3 dislocations during the last 8 months. What is the best treatment option?

     

     

     

    1. Both component revision

    2. Acetabular component revision and femoral head exchange

    3. Exchange of the liner to a face-changing liner and a larger femoral head

    4. Exchange of the head and liner to a polyethylene liner and ceramic femoral head

    5. Resection arthroplasty with an antibiotic spacer and 2-stage reimplantation

     

    PREFERRED RESPONSE:2

     

    Question 13- Who was the first surgeon to develop and use a stemmed implant that replaced the femoral head with metal?

     

    1. Otto E. Aufranc

    2. John Charnley

    3. Austin T. Moore

    4. Maurice Muller

    5. Marius Smith-Peterson

     

    PREFERRED RESPONSE:3

     

    Question 14- In the absence of infection as seen in Figures a through d, which femoral bone loss classification and treatment option best describes this scenario?

     

     

     

     

     

    1. Paprosky IIIA - distal modular tapered stem

    2. Paprosky IIIB - proximal modular stem

    3. Paprosky IIIB - 10-inch extensively coated stem

    1. Paprosky IV - 10-inch extensively coated stem

    2. Paprosky IV - femoral replacing endoprosthesis

    PREFERRED RESPONSE:5

     

    Question 15- Which cup position in a metal-on-metal hip arthroplasty is most commonly associated with elevated serum metal ion levels?

    1. Anteversion >20 degrees

    2. Anteversion <20 degrees

    3. Cup abduction <35 degrees

    4. Cup abduction >55 degrees

    5. Cup abduction of 45 degrees

     

    PREFERRED RESPONSE:4

     

    Question 16-During total knee arthroplasty, after placement of the trials, the patella subluxates laterally out of the trochlea. The axial alignment is appropriate. What is the next best step to improve patella tracking?

     

    1. Tibial tubercle transfer

    2. Additional resection of the patella

    3. Downsizing the femoral component

    4. External rotation of the tibial component

    5. Internal rotation of the femoral component

    PREFERRED RESPONSE:4

     

    Question 17- When using highly cross-linked ultra-high-molecular-weight polyethylene acetabular liners in primary

    total hip arthroplasty, what is the optimal head size in relation to wear rates?

     

    1. 22 mm

    2. 28 mm

    3. 32 mm

    4. 40 mm

    5. Head size does not affect wear rates.

    PREFERRED RESPONSE:5

     

    Question 18- A 62-year-old man has moderate knee osteoarthritis that interferes with golfing. He has a history of type II diabetes, obesity (body mass index 37), and cardiac stenting. What do the 2011 American Academy of Orthopaedic Surgeons Guidelines recommend for the treatment of his osteoarthritis?

     

    1. Weight loss

    2. Arthroscopy

    3. Chondroitin sulfate

    4. Lateral heel wedges

    5. Hyaluronic acid injections

     

    Question 19-The use of a continuous passive motion device after total knee replacement results in

    1. increased blood loss.

    2. improved 6-month quad strength.

    3. improved 6-month range of motion.

    4. improved early active knee flexion.

    5. decreased incidence of deep venous thrombosis.

     

    PREFERRED RESPONSE:4

     

    Question 20-Figures a and b are the anteroposterior and lateral radiographs of a 65-year-old man who had a total hip arthroplasty 5 years ago through a posterior approach. His arthritic hip pain was relieved, but now he has painful popping in the hip. He has to use his hands to lift his leg into the car. Pain is worse with resisted hip flexion and active straight leg raise. C-reactive protein, erythrocyte sedimentation rate,and bone scan values are within defined limits. What is the most likely pain etiology?

     

     

     

    1. Infection

    2. Osteolysis

    3. Subsidence

    4. Spinal stenosis

    5. Illiopsoas impingement

     

     

     

    Question21-Which type of cells has been implicated in the process shown inFigure?

     

    1. Monocytes

    2. Histiocytes

    3. Leukocytes

    4. Neutrophils

    5. Macrophages

     

    PREFERRED RESPONSE:5

     

     

     

    Question 22-Figure is the radiograph of a 32-year-old woman treated with high-dose steroids for a flare of systemic lupus erythematous. The most appropriate surgical treatment for the avascular necrosis lesion would be

     

    1. core decompression.

    2. total hip arthroplasty.

    3. bipolar hemiarthroplasty.

    4. vascularized fibula grafting.

    5. injection of platelet-rich plasma.

     

    PREFERRED RESPONSE:2

     

    Question 23-The elution of antibiotics from a cement spacer is increased by

    1. vacuum mixing.

    2. increased cement porosity.

    3. the use of a static spacer.

    4. the use of vancomycin alone.

    5. the use of rifampin in combination with other antibiotics.

     

     

     

    Question 24-Figure is the radiograph of a 55-year-old veteran who developed avascular necrosis after a traumatic hip dislocation. He was treated with hemiarthroplasty 10 years ago and also has posttraumatic stress disorder and chronic pain. He has had multiple spinal surgeries and takes 30 mg of methadone daily. He now has severe groin pain and is unable to ambulate. Laboratory studies showed a C-reactive protein level of 0.2 mg/L (reference range, 0-3 mg/L), erythrocyte sedimentation rate of 50 mm/h (reference range, 0-20 mm/h), hip aspiration of 500/mm3 white blood cell count, 50% polynucleated cells, 30%monocytes, and 20% lymphocytes What is the most likely cause of his hip pain?

     

    1. Infection

    2. Osteolysis

    3. Acetabular protrusio

    4. Loosening of implant

    5. Complex regional pain syndrome

     

    PREFERRED RESPONSE:3

     

     

     

     

     

     

     

     

    Question1 A 60-year-old Middle Eastern woman with a dark complexion is seen in the emergency department for a nondisplaced humeral fracture. She has osteoporosis based on a previous bone mineral density test and a history of fracture. Laboratory studies should include measuring levels of

     

    1. vitamin D2.

    2. vitamin D3.

    3. 25 hydroxycholecalciferol.

    4. 1,25 dihydrocholecalciferol.

    5. 24,25-dihydroxycholecalciferol.

    PREFERRED RESPONSE: 3

     

    Question2 A 63-year-old African-American man is scheduled for right shoulder arthroplasty for degenerative joint disease. He has no history of infection, connective tissue disease, or tobacco and alcohol abuse.Laboratory studies drawn on the day of surgery show a leukocyte count of 2.2 X 109 cells/L (reference range, 4.5 X 109 cells/L). All other laboratory and presurgical evaluations are within defined limits. What is the best appropriate course of action?

     

    1. Perform the surgery after consultation with a hematologist.

    2. Perform the surgery because this value is not abnormally low for this patient.

    3. Delay surgery and immediately repeat the white blood cell count.

    4. Cancel the surgery because of the abnormally low white blood cell count.

    5. Cancel the surgery; a delay is needed to consider this value.

    PREFERRED RESPONSE: 2

     

    Question3 What is the leading cause of medication errors, delays in diagnosis and treatment, and wrong-site surgeries?

     

    1. Not enough sleep

    2. Too heavy a caseload

    3. Inadequate preparation

    4. Communication failures

    5. Lack of patient participation

    PREFERRED RESPONSE: 4

     

    Question4 Figures 85a through 85c are the injury and reconstruction images of a 48-year-old man who had his right arm amputated 5 cm below his elbow by a machine. His postsurgical course is uneventful. He is pleasant in all his interactions with those treating him. He is fitted with and has learned to use a myoelectric prosthesis. He is discharged from physical therapy with the evaluation that he is capable of returning to his job activities. The best next step should be to

     

     

    1. return him to work as soon as possible.

    2. evaluate independently his ability to use his myoelectric prosthesis.

    3. obtain a functional capacity evaluation and compare it to his job description.

    4. offer the opportunity to be evaluated by psychology before returning to work.

    5. recommend vocational rehabilitation because his prosthesis is too slow to use for work.

    PREFERRED RESPONSE: 4

    Question5 Which finding is most associated with intimate partner violence?

    1. Multiple extremity fractures

    2. Isolated abdominal injury

    3. Isolated lower-extremity fracture

    4. Evidence of drug or alcohol use by the partner

    5. Repeated visits to the emergency department

    PREFERRED RESPONSE: 5

    Question6 A 25-year-old red-haired healthy woman with no history of substance abuse underwent fixation of a fractured distal radius under supraclavicular block but required an unusual amount of intravenous sedation and analgesia. Afterwards, she came to the emergency department in extreme pain. Compartment syndrome was ruled out; on maximum dose oral opiates, she improved over time. At her postoperative visit, she comes in with her red-haired mother, who related similar need for high dose pain medications. Examination is otherwise uneventful. What is the most appropriate course?

    1. Perform a drug test

    2. Question her drug and alcohol history

    3. Refer her to the pain service within a week

    4. Give her more high-dose pain medication

    5. Discuss a possible inherited lower pain tolerance

     

    PREFERRED RESPONSE: 5

    Question7 A 38-year-old Hindu man underwent serial debridement for necrotizing fasciitis of the hand and forearm.Although the infection has cleared, he has extensive areas of exposed tendon both volarly and dorsally.An option for coverage is an acellular collagen matrix derived from fetal bovine dermis. In addition to obtaining routine informed consent for this procedure, the physician should explain the material’s

    1. origin.

    2. durability.

    3. permeability.

    4. resorption rate.

    5. tensile strength.

    PREFERRED RESPONSE: 1

     

    Question8 Ganglion excision is scheduled for a 55-year-old male laborer born in Mexico. You communicate between his limited English, your modest Spanish, and his daughter’s command of both languages. He agrees to the procedure, and wishes no further information by a translator, despite your offering patient brochures and use of a patient-oriented computer kiosk. What is the next most appropriate step?

    1. Postpone the surgery

    2. Obtain translator services anyway

    3. Ask whether he and his daughter understand the procedure

    4. Request further family members be present for the decision

    5. Ask him and his daughter to, in their own words, explain the proposed plan

    PREFERRED RESPONSE: 5

     

    Question9 An otherwise healthy 25-year-old Hispanic man is seen in the emergency department; he is accompanied by his supervisor. He has a cut to his right hand from a table saw and requires emergency surgery. All of his responses are single-word answers, and when asked if he has any questions before proceeding with surgery, he says “No.” The physician should now

    1. proceed with surgery, considering he has signed the informed consent sheet.

    2. ask the supervisor to explain the procedure to him.

    3. ask him to explain in his own words his injury and the proposed procedure.

    4. have a second surgeon examine him and sign the informed consent.

    5. discuss his surgery and planned after-surgery care through a translator before proceeding with the surgery.

    PREFERRED RESPONSE: 3

     

    Question10 A 21-year-old man has leg weakness after a motor vehicle collision. Examination reveals normal strength in his upper extremities, with 2/5 strength in the quadriceps, 2/5 ankle plantar flexion, and 0/5 ankle and great toe extension. Examination shows no rectal tone but intact perirectal sensation. A CT scan reveals a T9-T10 dislocation. What best describes his spinal cord injury?

    1. Complete, American Spinal Injury Association (ASIA) A 2. Complete, ASIA B

      1. Incomplete, ASIA B 4. Incomplete, ASIA C 5. Incomplete, ASIA D

         

        PREFERRED RESPONSE: 4

        Question11 When using a presurgical safety checklist and timeout, which surgical team member has been shown to be the most effective in reducing surgical complications?

        1. Scrub nurse

        2. Circulating nurse

        3. Anesthesiologist

        4. Surgeon

        5. Surgeon delegating different aspects of the presurgical checklist/timeout to various members of the team

        PREFERRED RESPONSE: 4

         

        Question12 After treating a supracondylar humeral fracture in a 4-year-old child, nerve palsy is identified. The treating physician should acknowledge the nerve damage and should offer

        1. an apology and accept blame.

        2. an apology and not accept blame.

        3. an apology and accept partial blame.

        4. no apology and accept blame.

        5. no apology and not accept blame.

        PREFERRED RESPONSE: 2

         

        Question 1-Accidental injury is the most common cause of death in children. Which cause of death ranks second?

        1. Cancer

        2. Child abuse

        3. Cardiac disease

        4. Birth-related illness

        5. Respiratory infection

         

        PREFERRED RESPONSE:2

         

        Question 2-A 6-year-old boy was seen in the emergency department with a spiral fracture of the left femur. His father indicated that he sustained the fracture when he slipped while getting off of a couch. The boy has mild cerebral palsy with spastic diplegia; however, 2 siblings are developing normally. His mother indicated that his birth was the result of an unplanned pregnancy and that her father was a strict disciplinarian who frequently hit her with a belt. His father was recently laid off from a steady job, the family had to move to a new neighborhood, they have not met any neighbors, and no relatives live nearby. In addition to the suspicious fracture, how many social indicators of increased risk for nonaccidental injuries are represented in this scenario?

        1. 1

    2. 2

    3. 3

    4. 4

    5. 5

    PREFERRED RESPONSE:5

     

    Question 3- A 15-year-old male soccer player sustained a blow to the head during a corner kick. He reports a headache but denies loss of consciousness, nausea, diplopia, tinnitus, or amnesia. His neurologic examination is unremarkable. The most appropriate return to play is

     

    1. at a later date.

    2. after a CT scan of the brain.

    3. after a CT scan of the brain and neuropsychologic testing.

    4. 20 minutes after symptom resolution.

    5. immediately, providing parental consent can be obtained and considering his normal neurologic examination findings.

     

    PREFERRED RESPONSE:1

     

    Question 4-A 25-year-old woman is seen in the emergency department with a displaced right ankle fracture and multiple contusions as a result of a fall down the stairs. She is otherwise healthy but reports being 12 weeks pregnant. In addition to treating her ankle fracture, the orthopaedic surgeon should

     

    1. recommend evaluation of bone density to rule-out osteoporosis of pregnancy.

    2. recommend additional obstetrical consultation because of the risk for malpractice claims.

    3. question the patient regarding her calcium intake during pregnancy.

    4. question the patient privately regarding the circumstances of this and any prior injuries.

    5. refer to a specialist who can safely manage surgical treatment during pregnancy.

     

    PREFERRED RESPONSE:4

     

    Question 5- Which type of study represents the highest level of evidence?

     

    1. Case series

    2. Case control study

    3. Prospective cohort study

    4. Meta-analysis of randomized trials with homogeneous results

    5. Unblinded randomized controlled trial with uncertain results

     

    PREFERRED RESPONSE:4

     

    Question 6-Figures a and b are the T2-weighted sagittal and axial MRI scans of a 35-year-old woman with severe pain and numbness in her right lower extremity who failed multiple nonsurgical treatments. Which characteristic is most associated with a decreased treatment effect of surgery?

     

     

     

    1. Marital status

    2. Less formal education

    3. Workers’ compensation

    4. Lack of peripheral joint disease

    5. Worsening symptom trend at baseline

    PREFERRED RESPONSE:3

     

    Question 7-An 88-year-old woman with limited community ambulation who resides in an assisted living facility has a hip fracture and mild cognitive deficit. When discussing the anticipated treatment outcome with her and her family, what should be conveyed regarding anticipated 1-year mortality and independence compared

    to prefracture status?

     

    1. 20% mortality, 20% chance of loss of some independence

    2. 20% mortality, 50% chance of loss of some independence

    3. 50% mortality, 50% chance of loss of some independence

    4. 50% mortality, 20% chance of loss of some independence

    5. 75% mortality, 50% chance of loss of some independence

     

    Question 8-What does an unpaired student t-test require?

    1. Matched data

    2. Categorical data

    3. Noncontinuous data

    4. Normally distributed data

    5. Three or more comparison groups

    PREFERRED RESPONSE:2

     

    PREFERRED RESPONSE:4

     

    Question 9- What is the approximate prevalence of intimate partner violence (emotional, physical, and sexual abuse) among women within the 12 months before they are seen at orthopaedic clinics for the treatment of musculoskeletal injuries?

     

    1. 12% 2. 32 3. 52% 4.72% 5. 92%

     

    PREFERRED RESPONSE:2

     

    Question 10-Which study design is characterized by a systematic review that combines the results of multiple studies to answer a focused clinical question?

    1. Meta-analysis

    2. Cohort study

    3. Prospective case-control study

    4. Randomized clinical trial

    5. Nonrandomized clinical trial

    PREFERRED RESPONSE:1

     

    Question 11- A study’s power is its probability

     

    1. of rejecting the null hypothesis when it is really true.

    2. of failing to reject the null hypothesis when it is really false.

    3. of a given test result in a patient with the target disorder.

    4. of obtaining the same or more extreme data assuming the null hypothesis is of no effect.

    5. that the sample mean will be sufficiently different from the mean under the null hypothesis to allow rejection of the null hypothesis.

     

    PREFERRED RESPONSE:5

     

    Question 12-The most widely accepted scoring system used to evaluate generalized joint hypermobility is the Beighton-Horan scale. Under this scoring system, 5 criteria are evaluated for a total maximum score of

     

    1. 5. 2. 6. 3. 8. 4. 9. 5. 10.

     

    PREFERRED RESPONSE:4

     

     

     

     

    Question1 A 60-year-old woman has a high-grade pleomorphic undifferentiated sarcoma of the thigh. Which figure supports this pathologic diagnosis?

     

     

     

    1. Figure 4a

    2. Figure 4b

    3. Figure 4c

    4. Figure 4d

    5. Figure 4e

     

    PREFERRED RESPONSE: 4

     

    Question2 Which soft-tissue sarcoma is most likely to develop lymphatic metastasis?

     

    1. Liposarcoma

    2. Leiomyosarcoma

    3. Synovial sarcoma

    4. Myxoid liposarcoma

    5. Pleomorphic sarcoma

    PREFERRED RESPONSE: 3

     

     

     

    Question3 Figures 25a through 25c are the axial T1 and postcontrast MRI scans and biopsy specimen of a 35-yearold man with a painless right thigh mass. He noticed the mass about 2 weeks ago and is unsure if it has changed in size. Which translocation most commonly is associated with this type of tumor?

     

    1. t(2;13)(q35;q14)

    2. t(12;22)(q13;q12)

    3. t(12;16)(q13;p11)

    4. t(X;18)(p11;q11)

    5. t(17;22)(q22;q13)

     

    PREFERRED RESPONSE: 3

     

     

     

    Question4 Figures 34a through 34d are the radiographs and biopsy specimen of a 68-year-old woman with an 8-month history of a slowly enlarging, painful distal left thigh mass. What is the recommended treatment?

     

    1. Surgery alone

       

    2. Radiotherapy alone

       

    3. Radiotherapy and surgery

       

    4. Neoadjuvant chemotherapy and surgery

       

    5. Neoadjuvant chemotherapy, radiotherapy, and surgery

     

    PREFERRED RESPONSE: 1

     

    Question5 Figures 47a through 47d are the plain radiographs, axial MRI scan, and biopsy specimen of an 8-yearold boy with progressive right elbow pain that awakens him from sleep. Examination reveals soft-tissue fullness around his elbow and pain with active or passive motion. What is the most likely diagnosis?

     

     

     

     

    1. Lymphoma

    2. Osteomyelitis

    3. Osteogenic sarcoma

    4. Ewing sarcoma

    5. Langerhans cell histiocytosis

    PREFERRED RESPONSE: 5

     

     

     

    Question6 Figures 60a through 60e are the radiographs, MRI scan, and biopsy specimen of a 17-year-old boy with a3-month history of left hip pain that is constant and not relieved by anti-inflammatory medication. What is the most likely diagnosis?

     

    1. Enchondroma

    2. Ewing sarcoma

    3. Osteoblastoma

    4. Chondrosarcoma

    5. Chondroblastoma

     

    PREFERRED RESPONSE: 5

     

    Question7 Figures 60a through 60e are the radiographs, MRI scan, and biopsy specimen of a 17-year-old boy with a 3-month history of left hip pain that is constant and not relieved by anti-inflammatory medication. What is the most likely diagnosis?

    1. Enchondroma

    2. Ewing sarcoma

    3. Osteoblastoma

    4. Chondrosarcoma

    5. Chondroblastoma

    PREFERRED RESPONSE: 5

     

    Question8 What is the treatment recommendation for an American Joint Committee on Cancer stage IIB (Enneking stage IIB) malignant fibrous histiocytoma of bone?

     

    1. Wide excision alone

    2. Radiotherapy and wide excision

    3. Chemotherapy and radiation

    4. Chemotherapy and wide excision

    5. Chemotherapy, wide excision, and radiotherapy

     

    PREFERRED RESPONSE: 4

     

     

     

    Question9 Figures 79a through 79d are the plain radiographs and axial CT scans of an 80-year-old woman with severe dementia and a newly noted thigh mass. Examination reveals a large, nonmobile anterior thigh mass that is minimally tender. What is the best next treatment step?

     

    1. Biopsy

    2. Chest CT scan

    3. Observation

    4. Wide resection

    5. Marginal resection

     

    PREFERRED RESPONSE: 3

     

     

     

    Question10 Figures 86a through 86c are the radiographs and biopsy specimen of a 14-year-old boy who has had left knee pain for 4 weeks. What is the most likely diagnosis?

     

    1. Chondroblastoma

    2. Chondrosarcoma

    3. Parosteal osteosarcoma

    4. Osteoblastoma

    5. Osteosarcoma

     

    PREFERRED RESPONSE: 5

     

     

     

    Question11 Figures 109a through 109d are the plain radiographs and axial T2-weighted MRI scans of a 30-year-old woman who has had right hip pain for the past month. Examination reveals an antalgic gait and a firm,fixed proximal femoral mass. What is the best next treatment step?

     

    1. Biopsy

    2. Observation

    3. Chemotherapy

    4. Internal fixation

    5. Radiation therapy

     

    PREFERRED RESPONSE: 1

     

    Question12 Figures 127a through 127c are the femur radiographs of a 76-year-old man who is a community ambulator. This man, who has biopsy-proven metastatic prostate cancer, has been experiencing left hip and thigh pain while walking for 3 weeks. What is the best next treatment step?

     

     

    1. Total hip arthroplasty

    2. Long cephalomedullary fixation

    3. Long cephalomedullary fixation and radiotherapy

    4. Plate and screw hip fixation

    5. Plate and screw hip fixation and radiotherapy

    PREFERRED RESPONSE: 3

     

    Question13 A 70-year-old woman with multiple myeloma is scheduled to begin intravenous bisphosphonate treatment and has concerns about the side effects of this medication. In addition to reviewing the common side effects, the following is advised before therapy initiation

    1. an echocardiogram.

    2. a chest radiograph.

    3. a complete blood count.

    4. a gastric swallowing study.

    5. completion of invasive dental work.

    PREFERRED RESPONSE: 5

     

    Question14 Figures 155a and 155b are the plain radiographs of a 17-year-old boy who recently noted painless swelling in his distal thigh. Examination reveals a firm, fixed, deep distal thigh mass. There is no associated tenderness. What is the best next treatment step?

     

     

    1. Biopsy

    2. Resection

    3. A CT scan

    4. An MRI scan

    5. Observation

    PREFERRED RESPONSE: 5

     

     

     

    Question15 Figures 170a through 170c are the plain radiographs and coronal short inversion time inversion recovery(STIR) MRI scan of a 44-year-old woman with metastatic thyroid carcinoma and right shoulder pain. She reports no history of trauma. Examination reveals no masses and considerable tenderness of the proximal humerus. What is the best next treatment step?

     

    1. Observation

    2. Physical therapy

    3. Radiation therapy

    4. Prophylactic internal fixation

    5. Proximal humeral replacement

     

    PREFERRED RESPONSE: 3

     

    Question16 Figures 180a through 180f are the plain radiographs, coronal short inversion time inversion recovery MRI scans, and biopsy specimen of a 27-year-old woman with an enlarging painless calf mass. She first noticed the area several months ago and is unsure if the mass has increased in size. What is the most likely diagnosis?

     

     

    1. Desmoids

    2. Liposarcoma

    3. Schwannoma

    4. Synovial sarcoma

    5. Pleomorphic sarcoma

     

    PREFERRED RESPONSE: 4

    Question17 Figures 210a and 210b are the axial T1 and postgadolinium MRI scans of a 67-year-old woman with a right thigh mass that has grown in size and has become increasingly symptomatic for 6 months. What is the best next treatment step?

     

     

    1. Observation

    2. Core biopsy

    3. Wide resection

    4. Marginal resection

    5. Radiation therapy

    PREFERRED RESPONSE: 4

     

    Question18 Figures 220a through 220c are the radiograph, MRI scan, and biopsy specimen of a 5-year-old boy with a 4-week history of right hip pain, limp, and a low-grade fever. He also has diabetes insipidus, exopthalmus, and multiple lesions in the skull. What is the most likely diagnosis?

     

     

    1. Ollier disease

    2. Gorham disease

    3. Hand-Schuller-Christian disease

    4. Mazabraud syndrome

    5. McCune-Albright syndrome

    PREFERRED RESPONSE: 3

     

    Question19 Figures 233a through 233c are the radiographs and biopsy specimen of a 32-year-old woman who has had progressive ankle pain for 6 months. What is the most appropriate treatment option?

     

     

    1. Radiotherapy alone

    2. Intralesional curettage

    3. Intralesional curettage with adjuvants

    4. Wide surgical resection and radiotherapy

    5. Neoadjuvant chemotherapy and wide resection

    PREFERRED RESPONSE: 3

     

     

     

    Question20 Figure 242 is the anteroposterior radiograph of a 28-year-old man who underwent resection and reconstruction for an Ewing sarcoma. What is the most common functional deficit encountered during rehabilitation?

     

    1. Hip instability

    2. Abductor weakness

    3. Adductor weakness

    4. Quadriceps weakness

    5. Leg-length discrepancy

     

    PREFERRED RESPONSE: 2

     

     

     

     

     

    Question21 Figures 258a and 258b are the radiographs of a 48-year-old woman who had knee pain after falling from a standing height. Her pain subsequently resolved. She was treated 2 years ago for invasive ductal breast carcinoma. What is the best next step in management?

     

    1. An MRI scan

    2. Biopsy

    3. Bone scan

    4. Observation

    5. Prophylactic fixation

     

    PREFERRED RESPONSE: 4

     

    Question22 Figures 267a through 267c are the radiographs and biopsy specimen of a 10-year-old boy who is experiencing lateral ankle pain during sports and recreational activities. What is the best next treatment step?

     

     

    1. Wide resection

    2. Curettage and bone graft

    3. Aspiration and steroid injection

    4. Chemotherapy and wide resection

    5. Observation and activity restriction

     

    PREFERRED RESPONSE: 2

    Question 1-Figures a and b are the radiograph and CT scan of a 22-year-old man with a painful forearm and elbow that is worse at night but painful at all times. Pain has been increasing during the past 5 months.He has no history of trauma or fevers. What is the most likely diagnosis?

     

     

    1. Osteomyelitis

    2. Osteoid osteoma

    3. Ewing sarcoma

    4. Eosinophilic granuloma

    5. Small-cell osteosarcoma

    PREFERRED RESPONSE:2

    Question2- Figures a and b are the gadolinium-enhanced MRI scans of a 68-year-old woman with intermittent midthigh pain for 2 months. She has an anterolateral proximal thigh mass of approximately 9 cm.Radiographs reveal no bone lesion, but there is a soft-tissue mass. A needle biopsy of the thigh lesion isseen in Figure c. What is the most likely diagnosis?

     

     

     

    1. Liposarcoma

    2. Fibrosarcoma

    3. Desmoid tumor

    4. Myxoid liposarcoma

    5. Pleomorphic sarcoma

    PREFERRED RESPONSE:5

     

    Question 3-An experienced orthopaedic surgeon practicing in a community hospital setting has a 55-year-old woman with a mixed lytic and blastic proximal humeral lesion. She has no other symptoms and no history of malignancy. What is the most appropriate next step in managing this condition?

     

    1. Open biopsy alone

    2. Open biopsy and curettage of the lesion

    3. Needle biopsy performed in radiology

    4. Referral to a musculoskeletal tumor center

    5. Fluorodeoxyglucose positron emission tomography

    PREFERRED RESPONSE:4

     

    Question 4- Figures a through c are the radiographs and CT scan of a 22-year-old man with a minor knee injury.Findings from a CT scan of the chest and abdomen and a bone scan were negative with the exception of the lesion. A biopsy specimen is shown in Figure d. What is the appropriate next step in treatment?

     

     

     

     

     

    1. Observation

    2. Marginal resection

    3. Wide resection only

    4. Wide resection and chemotherapy

    5. Radiation followed by wide resection PREFERRED RESPONSE:3

     

    Question 5-A 51-year-old man has a slowly expanding upper-extremity mass. Examination reveals a firm 3-cm mass in his midvolar forearm. Radiographs are normal. You suspect a soft-tissue sarcoma. The best imaging study would be

    1. ultrasound.

    2. CT scan.

    3. bone scan.

    4. gadolinium-enhanced MRI scan.

    5. fluorodeoxyglucose positron emission tomography.

    PREFERRED RESPONSE:4

     

     

     

    Question 6-The cause of the condition shown in Figure 69 is associated with which genetic abnormality?

     

    1. SYT-SSX gene

    2. t(11;22) translocation

    3. Mutation of p53

    4. Mutation of the EXT1 or EXT2 genes

    5. Mutation of the GNAS gene on chromosome 20

     

    PREFERRED RESPONSE:4

     

    Question 7-Figures a through e are the radiographs, MRI scan, and biopsy specimens of a 41-year-old man with mild shoulder pain. A whole-body bone scan showed an isolated area of uptake in the right proximal humerus. What is the next most appropriate treatment?

     

     

     

     

     

     

    1. Observation

    2. Curettage and bone grafting

    3. Culture-directed antibiotics

    4. CT scan of the chest, abdomen, and pelvis

    5. Preoperative chemotherapy followed by wide resection and reconstruction

    PREFERRED RESPONSE:1

    Question 8-Figures a through f are the radiograph, CT scan, MRI scan, and bone scan of a 28-year-old woman with a newly diagnosed breast carcinoma who underwent a bone scan for staging. She has no hip pain but a lesion is noted. What is the best next treatment step?

     

     

    1. Biopsy of the right elbow

    2. Needle biopsy of the right femoral bone lesion

    3. Reassurance and repeat plain radiographs in 3 months

    4. Prophylactic intramedullary rod fixation of the right femur

    5. Radiation to the right femoral bone lesion coordinated with systemic therapy

     

    PREFERRED RESPONSE:3

     

     

    Question 9-Figures a through c are the plain radiograph, MRI scan, and biopsy specimen of a 14-year-old girl who had right hip pain for 2 months. After a fall, the pain increased. What is the most likely diagnosis?

     

     

    1. Osteomyelitis

    2. Aneuysmal bone cyst

    3. Unicameral bone cyst

    4. Eosinophilic granuloma

    5. Telangiectactic osteosarcoma

     

    PREFERRED RESPONSE:2

     

    Question 10-Figures a through f are the MRI scans and synovial biopsy specimens of a sexually active 20-yearold man with a swollen, painful knee. He has no history of fevers or trauma. Knee culture results are negative. What is the most likely diagnosis?

     

     

     

     

     

    1. Synovial sarcoma

    2. Rheumatoid arthritis

    3. Gonococcal arthropathy

    4. Giant-cell tumor of bone

    5. Pigmented villonodular synovitis

    PREFERRED RESPONSE:5

    Question 11-Figures a through d are the radiographs and needle biopsy specimen of a 21-year-old man with gradual onset of ankle pain and no history of trauma. What is the next appropriate step in management?

     

     

     

    1. Observation

    2. Wide resection

    3. Wide resection and chemotherapy

    4. Curettage with bone graft

    5. Curettage followed by radiotherapy

    PREFERRED RESPONSE:4

    Question 12-Surgical resection without radiation or chemotherapy is the recommended treatment for which bone sarcoma?

    1. Paget

    2. Ewing

    3. Osteogenic

    4. Chondrosarcoma

    5. Malignant fibrous histiocytoma of bone

    PREFERRED RESPONSE:4

     

    Question 13-Figures a and b are the radiographs of a 56-year-old man with weight-bearing pain in the proximal thigh. A CT scan of his abdomen is shown in Figure c. The next appropriate treatment step is

     

     

     

    1. observation.

    2. chemotherapy.

    3. radiation therapy.

    4. preoperative embolization.

    5. prophylactic internal fixation.

    PREFERRED RESPONSE:4

     

    Question 14-When evaluating postmenopausal women who are taking osteoporosis medication, history should include use of

     

    1. statins.

    2. antiepileptics.

    3. cephalosporins.

    4. high-dose vitamin C.

    5. antigout medications.

     

    PREFERRED RESPONSE:2

     

    Question 15-Figures a through d are the anteroposterior and lateral radiographs, MRI scan, and biopsy specimen of a 14-year-old girl who has had knee pain for 6 weeks. What is the most appropriate treatment?

     

     

     

     

     

    1. Chemotherapy alone

    2. Wide resection alone

    3. External beam radiation alone

    4. Chemotherapy and wide resection

    5. Prophylactic nailing and external beam radiation

     

    PREFERRED RESPONSE:4

     

    Question 16-Figures a through f are the radiographs, MRI scans, and biopsy specimen of a 56-year-old woman with right thigh pain. What is the most likely diagnosis?

     

     

     

     

     

     

    1. Lymphoma

    2. Fibrous dysplasia

    3. Multiple myeloma

    4. Metastatic carcinoma

    5. Eosinophilic granuloma

    PREFERRED RESPONSE:3

     

    Question 17-Figures a and b are the biopsy specimens of a 30-year-old man with an enlarging gluteal mass.What is the most likely diagnosis?

     

     

    1. Fibrosarcoma

    2. Desmoid tumor

    3. Nodular fasciitis

    4. Pleomorphic sarcoma

    5. Atypical lipomatous tumor

    PREFERRED RESPONSE:2

    Question 18-Ewing sarcoma is most commonly associated with which translocation?

    1. t(9;22)

    2. t(2;13)

    3. t(11;22)

    4. t(12;16)

    5. t(X;18) PREFERRED RESPONSE:3

     

    Question 19-Which figure matches the radiographs seen in Figures 228a and 228b from a 15-year-old boy with a 3-month history of knee pain?

     

     

     

     

    1. Figure 228c

    2. Figure 228d

    3. Figure 228e

    4. Figure 228f

    5. Figure 228g

     

     

     

    PREFERRED RESPONSE:3

     

     

     

    Question 20-Which cell is identified by the arrow in Figure ?

     

    1. Osteoblast

    2. Osteoclast

    3. Osteocyte

    4. Plasma cell

    5. Megakaryocyte

     

    PREFERRED RESPONSE:2

     

    Question 21-A 14-year-old girl has a painless deformity of the right tibia. A radiograph from 2 years ago is seen in Figure a; nothing was done at that time. Her current radiograph is seen in Figure b. She has no pain, fever, or drainage. What is the most likely diagnosis?

     

     

     

     

    1. Adamantinoma

    2. Fibrous dysplasia

    3. Osteofibrous dysplasia

    4. Nonossifying fibroma

    5. Chronic osteomyelitis

    PREFERRED RESPONSE:3

     

     

     

     

    Question1 Based on the findings shown in Figures 3a and 3b, what is the most likely

    diagnosis?

     

     

    1. Mucopolysaccharidosis

    2. Osteogenesis imperfecta

    3. Legg-Calvé-Perthes disease of both hips

    4. A history of developmental hip dysplasia that has been treated

    5. A history of developmental hip dysplasia that has not been treated

    PREFERRED RESPONSE: 1

    Question2 Figures 13a through 13f are the coronal and sagittal CT scans of a boy who sustained a Salter-Harris II fracture through the physis of the distal tibia, with an associated Salter-Harris I distal fibula fracture at 10 years of age. He was treated with closed reduction and cast immobilization. Now at age 12, he is asymptomatic and has a lower limb-length discrepancy that is 1 cm shorter on the involved side and bone age consistent with standards for 13-year-olds. His parents should be informed that

     

    1. his growth has stopped and no additional visits are necessary.

    2. the areas of physeal closure are too complex for reliable bar resection.

    3. differential continued growth between the tibia and the fibula is unlikely.

    4. epiphyseodesis of the contralateral tibia and fibula is indicated to reestablish length equality.

    5. physeal bar resection with interposition of fat or bone cement is indicated to permit continued growth of the distal tibia.

     

     

     

     

    PREFERRED RESPONSE: 2

     

    Question3 A 10-year-old boy has had the insidious onset of anterior knee pain in both knees for 3 months. He had no inciting traumatic event and no pain at night or when walking, but has pain when ascending stairs or running. Examination reveals full active and passive range of motion of both knees, no hip pain with log roll, mild swelling over the tibial tubercle of both knees, and reproduction of the knee pain with direct palpation of the tibial tubercles.

    The most appropriate next step should include

     

    1. a 3-phase bone scan.

    2. an MRI scan of both knees.

    3. core strengthening exercises.

    4. bilateral tibial tubercle osteotomies.

    5. nonsteroidal anti-inflammatory drugs, activity modification, and gentle quadriceps stretching.

     

    Question4 A 15-year-old girl involved in a motor vehicle collision has severe back pain and is unable to move or feel her legs. The emergency medical technician noticed a large ecchymotic area on her back at the thoracolumbar junction. What is the most appropriate initial evaluation?

    1. A CT scan of the entire spine

    2. Placement of a Foley catheter

    3. Radiographs of the thoracolumbar junction

    4. Evaluation of the bulbocavernosus reflex

    5. Evaluation of the airway, breathing, and circulation

    PREFERRED RESPONSE: 5

     

    Question5 Figures 35a and 35b are the radiographs of a 9-year-old boy who fell from a tree and sustained a left elbow fracture. With open reduction and internal fixation, which technique would minimize after-surgery lateral spurring?

     

     

     

    1. Screw fixation of the fracture

    2. Bone wax over the lateral metaphysis

    3. Removal of the Kirschner wires by 3 weeks after surgery

    4. Delayed range of motion until 6 weeks after surgery

    5. Anatomical restoration of the lateral periosteum

     

     

     

    Question6 A 10-year-old girl has the injury seen in Figures 43a and 43b. She is treated with closed reduction and percutaneous pinning with smooth Kirschner wires. What is the most common complication of this fracture?

     

    1. Medial meniscus tear

    2. Arthrofibrosis of the knee

    3. Distal femoral growth arrest

    4. Superficial peroneal nerve palsy

    5. Lateral femoral condyle osteochondral fracture

     

    PREFERRED RESPONSE: 3

     

    Question7 Figure 49 is the radiograph of a child with a forearm fracture. Access to follow-up care will be limited in most regions of the United States by

     

     

     

     

    1. body mass index.

    2. patient comorbidities.

    3. type of immobilization in place.

    4. referral expertise.

    5. the number of orthopaedic surgeons willing to see pediatric patients.

     

    Question8 A 13-year-old boy sustained the injury shown in Figure 56a. Closed treatment under general anesthesia was performed within 3 hours. Posttreatment CT scans shown in Figures 56b through 56d revealed acceptable results. At the 7-week visit, the boy has remained asymptomatic and has begun some running despite being given instructions for activity restrictions. Figures 56e through 56g show current standing anteroposterior, standing “false profile,” and supine internally rotated radiographs of the hip. Based on these findings, the parents should be informed that

     

     

    1. there is concern about possible chondrolysis.

    2. there is no further concern about possible osteonecrosis.

    3. the hip is not completely reduced.

    4. the radiographs show evidence of osteonecrosis of the femoral head.

    5. heterotopic ossification is the result of the patient’s early return to activity.

    PREFERRED RESPONSE: 1

     

    Question9 A 12-year-old right-handed girl has right shoulder pain after pitching baseball. Examination reveals right shoulder tenderness to palpation over the anterolateral aspect and mild weakness with resisted internal rotation and abduction. Radiographs of her shoulder reveal no abnormalities. What is the most appropriate next step in management?

    1. MRI scan of the right shoulder

    2. MR arthrogram of the right shoulder

    3. Glenohumeral corticosteroid injection

    4. Arthroscopic evaluation and repair of the middle glenohumeral ligament

    5. 2-month hiatus from pitching followed by a progressive throwing program

     

    Question10 Figures 70a through 70e are the radiographs and MRI scan of a 12-year-old boy with worsening thigh pain. What is the most appropriate definitive surgical treatment?

     

     

    1. Observation with repeat follow up in 3 months

    2. Curettage and placement of a bone graft substitute

    3. Incision and drainage and placement of antibiotic beads

    4. Resection of the distal one-third of the femur and knee fusion

    5. Resection of the distal two-thirds of the femur and custom implant

    PREFERRED RESPONSE: 5

     

    Question11 For treatment of displaced supracondylar humeral fractures in children, the AAOS clinical practice guideline, The Treatment of Pediatric Supracondylar Humerus Fractures, recommends closed reduction and pin fixation using which pin configuration?

    1. 1 medial and 1 lateral

    2. 1 medial and 2 lateral

    3. 2 medial and 1 lateral

    1. 2 to 3 lateral

    2. 2 to 3 medial

     

    Question12 A 2-week-old infant has an Ortolani positive right hip. She is placed in a Pavlik harness with her hips flexed to 120 degrees. Three days after the harness is started, her parents notice that she is not extending her right knee. What is the most likely reason for the change?

    1. Septic right knee

    2. Right femoral nerve palsy

    3. Avascular necrosis of the right hip

    4. Compartment syndrome of the right leg

    5. Development of right hip Pavlik harness disease

    PREFERRED RESPONSE: 2

     

    Question13 An otherwise healthy 5-year-old girl underwent closed reduction and percutaneous pin fixation of an uncomplicated supracondylar fracture of the distal humerus. Four weeks later, radiographs show bone healing and the 2 smooth Kirschner wires are removed. The patient has range of motion from 30 degrees of flexion to 90 degrees of flexion. You inform her parents that

     

    1. spontaneous play and gentle household chores will almost always allow a patient to regain full elbow range of motion.

    2. progressive static splinting should be initiated immediately to regain full elbow extension.

    3. constant-force (spring loaded) splinting should be initiated immediately to regain full elbow flexion.

    4. formal therapy sessions emphasizing forearm rotation should begin immediately.

    5. arthroscopic anterior capsular release is commonly indicated following a pediatric supracondylar humerus fracture.

     

    PREFERRED RESPONSE: 1

     

    Question14 An 8-year-old girl underwent a drainage procedure of her left hip joint 3 days ago. She is being treated with appropriate antibiotics. She remains febrile, has left anterior groin pain, keeps her left hip flexed at about 20 degrees, and resists any left hip extension. T1 and T2 STIR images in the axial, coronal, and sagittal projections are shown in Figures 94a through 94f. What conclusions can be drawn from these findings?

     

    1. These are expected MRI scan findings following drainage of acute septic arthritis of the hip.

    2. In addition to the septic hip joint, there is osteomyelitis of the iliac wing and involvement of several muscle groups including the psoas.

    3. There is sufficient recurrent fluid in the hip joint itself to explain the girl’s continued discomfort.

    4. The MRI scans confirm osteomyelitis of the femoral head.

    5. The septic arthritis of the hip is now proven to be secondary to a sarcoma of the iliacus muscle.

     

     

     

    PREFERRED RESPONSE: 2

    Question15 A 2-week-old infant has had decreased spontaneous motion of the right upper limb since birth. There were no birth fractures and no infection is present. Persistent posture is shown in Figure 97a, and the posture during a Moro response is shown in Figure 97b.

    Examination reveals full passive and active motion of the neck. Based on these findings, the parents should be instructed to implement what actions for the next 3 months?

     

     

    1. Perform passive stretching for the neck; most important: rotate the head toward the involved limb and extend the cervical spine.

    2. Perform passive stretching for the involved shoulder and elbow; most important: move the shoulder to extension/adduction and internal rotation.

    3. Perform passive stretching for the involved shoulder and elbow; most important: move the

       

      shoulder to elevation/abduction and external rotation.

    4. Perform passive stretching for the involved hand; most important: move the fingers into flexion at the interphalangeal joints and the thumb into adduction-flexion.

    5. Refrain from performing any passive stretching; let the infant move spontaneously and monitor for improvement.

    PREFERRED RESPONSE: 3

    Question16 An 18-month-old boy is evaluated because he is not walking. He is found to have generalized hypotonia, asymmetry in his muscle strength, with his proximal muscles weaker than his distal muscles, absent deep tendon reflexes, and tongue fasciculations. What is the most appropriate next step in determining a

    diagnosis?

    1. Obtain a skeletal survey.

    2. Schedule a muscle biopsy.

    3. Electromyography and nerve conduction velocity studies

    4. Referral for genetic testing of the survival motor neuron 1 gene

    5. Referral for genetic testing to evaluate for trisomy 21

    PREFERRED RESPONSE: 4

    Question17 A physician is called to the well-baby nursery to consult regarding an otherwise healthy female newborn;the clinician states that “one of the baby’s legs is backwards.” Examination of the involved limb reveals intact circulation and motor functions. Radiographs are shown in Figures 114a through 114c. Based on these findings, what is the most appropriate initial treatment?

     

     

    1. Casting or bracing with the knee in flexion

    2. Casting or bracing with the knee in extension

    3. Anterior knee release should be performed in about 6 months.

    4. An MRI scan of the knee should be obtained before beginning any attempt at treatment.

    5. Observation should be conducted for 3 weeks with the expectation of gradual spontaneous improvement.

     

    PREFERRED RESPONSE: 1

     

    Question18 Figures 122a and 122b are the radiographs of a 3-year-old girl with a flexed interphalangeal joint of the thumb on the left hand. Her parents notice that she has been unable to extend the interphalangeal joint of her thumb for 18 months; however, she has no pain and is able to fully use her hand. The parents deny any previous trauma to her hand.

    Examination reveals no tenderness, full motion of the metacarpophalangeal joint, and passive extension of the interphalangeal joint to 25 degrees short of neutral. A small volar mass is palpated at the level of the metatarsophalangeal joint. What is the most appropriate next step?

     

     

     

     

    1. Observation for 6 months

    2. Release of the A1 pulley of the thumb

    3. Repair of the flexor pollicis longus tendon

    4. Extension osteotomy of the proximal phalanx

    5. Physical therapy for improved extensor pollicis longus strength

     

    Question19 Figures 129a through 129b are the radiographs and MRI and CT scans of the lumbar spine of a 10-yearold premenarchal girl who has back pain and scoliosis. What is the most likely etiology of her scoliosis?

     

     

     

    1. Olisthetic

    2. Idiopathic

    3. Tethered cord

    4. Myelodysplasia

    5. Osteoid osteoma

     

    PREFERRED RESPONSE: 5

     

     

     

    Question20 The posteroanterior radiograph seen in Figure 138 is of a 15-year-old girl who is evaluated for scoliosis.She has a slightly elevated right shoulder, a moderate rib prominence of forward bend test, and normal strength and reflexes in her lower extremities. She is 2 years postmenarchal. The radiograph reveals a 30-degree right thoracic and 25-degree left lumbar scoliosis. What is the most appropriate treatment?

     

    1. Observation

    2. Obtain a total spine MRI scan

    3. Apical vertebral body stapling

    4. Posterior spinal fusion from T4-L1

    5. Use of a custom thoracolumbar orthosis for 23 hours per day

     

    Question21 Figures 143a through 143d are the radiographs of a 13-year-old girl who sustained a knee injury during a volleyball game. She has been otherwise asymptomatic, denies any previous musculoskeletal injury, and has been playing competitive team sports for several years. Examination of her forearms reveals neutral rotation position with restricted pronation-supination on the dominant right and complete absence of pronation-supination on her left arm. What is the most appropriate intervention?

     

     

     

    1. Early total elbow arthroplasty

    2. Immediate physical therapy and progressive splinting

    3. Resection of the radial heads after skeletal maturity

    4. Immediately avoid sports that require repetitive use or impact loading of the upper limbs.

    5. Rotational osteotomy to position the dominant hand in pronation and the nondominant hand in supination.

     

    Question 22 A 15-year-old patient sustained the injuries shown in Figures 151a through 151c in a motor vehicle collision and is otherwise medically stable. What is the most appropriate treatment?

     

     

    1. A spica cast on the right and hanging-arm cast on the left

    2. A right cephalomedullary femoral nail, right long-leg cast, and left hanging-arm cast

    3. A right cephalomedullary femoral nail, right intramedullary tibial rod, and left hanging-arm cast

    4. Multiple screws across the right femoral neck, right long-leg cast, and left humeral intramedullary rod

    5. Multiple screws across the right femoral neck, right intramedullary tibial rod, and left humeral intramedullary rod

     

    PREFERRED RESPONSE: 5

     

    Question23 A 10-year-old gymnast fell from the parallel bars and sustained an elbow dislocation. It is appropriately reduced in the emergency department acutely. What is the most appropriate treatment option?

     

    1. Splinting for 10 days, then begin protected range of motion

    2. Immediate range of motion and return to activities as tolerated

    3. Long-arm cast for 4 weeks, then begin protected range of motion

    4. Long-arm cast for 6 weeks, then splint for an additional 2 to 3 weeks

    5. Surgical repair of the medial collateral ligament and long-arm cast for 4 weeks

     

    Question24 Figure 168a is the initial radiograph and Figure 168b is the radiograph taken after a reduction was performed on a 15-year-old girl who fell from a horse. She has had persistant pain and swelling in her left shoulder since presentation. She has full motor function and sensation in her left arm and is 1.5 years postmenarchal. The most appropriate next treatment step is

     

     

    1. immobilization in a hanging-arm cast.

    2. fracture immobilization with a figure-of-8 brace.

    3. fracture fixation with an antegrade locked intramedullary nail.

    4. open reduction and internal fixation with a blade plate.

    5. closed reduction and percutaneous pinning.

    PREFERRED RESPONSE: 5

     

    Question25 An 18-month-old boy has a bowleg deformity. His parents have noticed an increase in the appearance of the deformity since he started walking at 12 months of age. Examination reveals full range of motion of his hips and knees with a mild bowleg deformity. He walks with a 10-degree internal foot progression angle and has no lateral knee thrust.

    What is the most appropriate next treatment step?

     

    1. Continued observation

    2. Start bilateral antivarus bracing

    3. Recommend guided growth surgery

    4. Recommend bilateral proximal tibial osteotomies

    5. Obtain bilateral knee MRI scans to evaluate for medial physeal bars

     

    Question26.Figures 177a and 177b are the radiographs of a 7-year-old boy with spastic cerebral palsy. He has quadriparetic involvement and is unable to ambulate. He has very limited abduction, 30 degrees of flexion contractures, and pain on abduction. Bilateral varus osteotomies are scheduled with acetabular procedures to improve stability. Which type of acetabular osteotomy should be performed?

     

     

     

    1. Dega iliac

    2. Salter iliac

    3. Pemberton iliac

    4. Steele triple

    5. Ganz or Bernese periacetabular

    PREFERRED RESPONSE: 1

     

     

     

    Question27 A child sustained the injury shown in Figure 182 1 month ago. The parents did not follow up with orthopaedic care. What is the most likely reason for parents to not follow through?

     

    1. Concern about cost

    2. Instructions were too difficult to follow.

    3. The parents do not understand the plan.

    4. The parents do not agree with the physician’s treatment plan.

    5. Recommendations conflict with the parents’ personal beliefs

     

    PREFERRED RESPONSE: 4

     

    Question28 The radiographs in Figures 187a through 187c were obtained 10 months after closed reduction and percutaneous pin fixation of a minimally displaced (Jacobs type 1) lateral condyle fracture of the distal humerus in an otherwise healthy 4-year-old girl. She has regained full elbow and forearm range of motion and has resumed all preinjury activities without pain or swelling at the elbow. What information can be given to the parents about the current radiographic findings?

     

     

    1. A spike of bone at the tip of the lateral condyle is frequently seen, is attributed to periosteal displacement from the injury, and should be functionally insignificant.

    2. An osteochondroma at the lateral condyle has resulted from displacement of a piece of the physeal plate that occurred at the instant of injury.

    3. The irregularity at the lateral condyle is sometimes known as a “Pelkan spur,” and implies an underlying vitamin C deficiency.

    4. The irregularity may have a significant cartilaginous component with capitellar damage, and an MRI arthrogram is indicated.

    5. The minimal cubitus varus is an infrequent sequela, but will almost always remodel and develop into a normal carrying angle.

     

    PREFERRED RESPONSE: 1

    Question29 A 5-year-old boy sustained a vertical shear fracture of the pelvis, such that his left hemipelvis is displaced upward 2 cm. A CT scan reveals widening of the pubic symphysis, mild external rotation of the left hemipelvis, and a small avulsion fracture of the left sacrum. He has no abdominal or urologic injury. What is the best next treatment step?

    1. Ambulation nonweight-bearing on the left for 6 weeks

    2. Closed reduction with a spica cast for 2 months

    3. Closed reduction with a posterior sacroiliac screw fixation

    4. Closed reduction with application of an anterior external fixator

    5. Closed reduction with an anterior external fixator and posterior sacroiliac screw fixation

    PREFERRED RESPONSE: 2

     

    Question30 An otherwise healthy adolescent girl was treated for left slipped capital femoral epiphysis. The contralateral hip had not slipped, but was stabilized prophylactically with a single cannulated screw. The implants were removed after 1 year. The pelvic radiographs (Figures 215a and 215b) and the MRI scans of the hip that had not originally slipped (Figures 215c through 215e) were obtained 10 months after screw removal (22 months after the original fixation). Which findings are shown in these studies?

     

     

    1. Both hips are normal and no further assessments will be needed.

    2. A neoplasm has developed in the femoral head on the unslipped side.

    3. There is now increased risk for a slip in the hip and a new screw should be inserted.

    4. Osteonecrosis has developed in the unslipped hip adjacent to the previous screw position.

    5. The screw track in the bone has not filled spontaneously as expected and grafting should beconsidered.

    PREFERRED RESPONSE: 4

     

     

    Question31 Figures 224a and 224b are the radiographs of a skeletally mature child with cerebral palsy and worsening difficulty with sitting who meets Gross Motor Function Classification System level IV criteria. Surgery is planned. What is the most appropriate treatment option?

     

    1. Anterior fusion from T12 to L5

    2. Anterior release from L1 to L4 and posterior fusion from T2 to L5

    3. Anterior release from L1 to L4 and posterior fusion from T12 to L5

    4. Posterior fusion from T2 to the sacrum

    5. Posterior fusion alone from T2 to L5 with Aponte osteotomies from L1 to L4

     

    PREFERRED RESPONSE: 4

    Question32 A 4-year-old boy sustained the fracture seen in Figures 239a and 239b. Examination reveals normal sensation to light touch throughout his left hand. Specific motor testing shows he is able to extend his ipsilateral thumb fully and cross his fingers, but is unable to actively flex the distal interphalangeal joint of his ipsilateral index finger. What is the most likely etiology of his motor deficit?

     

     

    1. Neuropraxia of the radial nerve

    2. Neuropraxia of the anterior intraosseous nerve

    3. Development of compartment syndrome

    4. Laceration of the flexor digitorum profundus of the index finger

    5. Laceration of the flexor digitorum superficialis of the index finger

    PREFERRED RESPONSE: 2

     

    Question33 Figures 251a and 251b are the radiographs of a 2-year-old boy who is otherwise healthy. Clinicalnphotographs of the prosthesis and the child are shown in Figures 251c through 251e. What recommendations should be given to the parents?

     

     

     

    1. Surgical hip reduction and acetabuloplasty should be performed now, anticipating staged femoral lengthening.

    2. Physical therapy to regain ankle dorsiflexion should begin to maximize the success of prosthetic fitting.

    3. Epiphyseodesis of the proximal tibia should be performed now to minimize the functional discrepancy between the tibia and fibula.

    4. Ankle equinus in the current prosthesis is acceptable because this may facilitate substitution for knee function in an eventual Van Ness rotationplasty.

    5. Syme’s amputation and fitting with an above-knee prosthetic design is recommended.

     

    PREFERRED RESPONSE: 4

     

    Question34 Figure 262 is the radiograph of a 15-year-old with cerebral palsy who meets Gross Motor Function Classification System level V criteria. What is the most appropriate treatment?

     

     

     

    1. Bilateral open reduction and pelvic osteotomies

    2. Hip abduction bracing with administration of onabotulinum toxin A

    3. Hip abduction bracing without administration of onabotulinum toxin A

    4. No treatment or radiographic follow up is needed unless the patient is in pain

    5. No treatment at this point, but close radiographic follow up is needed to monitor for progression

     

    PREFERRED RESPONSE: 4

     

    Question1- Pelvic fractures in children differ from those in adults because children with pelvic fractures are more likely to

    1. have hemodynamic instability.

    2. develop pseudarthrosis.

    3. develop coagulation abnormalities.

    4. develop deep venous thrombosis.

    5. be treated nonsurgically.

    PREFERRED RESPONSE:5

    Question 2- A child of short stature with cauliflower ears, hitchhikers’ thumb, and kyphoscoliosis typically has a defect in what gene?

    1. DTDST

    2. FGFR-3

    3. COL2A1

    4. Dystrophin

    5. Cartilage oligomeric matrix protein

    PREFERRED RESPONSE:1

    Question3- Figureis the 3-dimensional CT scan of a 9-month-old infant who had vertebral abnormalities noted on a standard chest radiograph. He was asymptomatic, but his parents saw that he was ‘crooked.’ He had no abnormalities in other organ systems. An MRI scan of the spine revealed the distal cord to be at L-1 with no cord compression, syrinx, or diastematomyelia. Which is the most appropriate treatment?

     

     

    1. No surgical treatment until age 5

    2. Hemifusion and instrumentation from T-11 to L-1

    3. Vertebral excision at T-12 with fusion and instrumentation from T-8 to L-3

    4. Vertebral excision at T-12 with fusion and instrumentation from T-10 to L-1

    5. Vertebral excision at T-12 with anterior and posterior fusion from C-7 to L-2

     

    PREFERRED RESPONSE:4

     

    Question 4-Figures a and b are the radiographs of a 10-year-old boy with bowing of the tibia. What is the most appropriate treatment for his left tibia deformity?

     

     

     

    1. Medial hemiephysiodesis of the proximal tibia

    2. Lateral hemiephysiodesis of the proximal tibia

    3. Acute tibial varus osteotomy with plate fixation

    4. Acute tibial valgus osteotomy with plate fixation

    5. Gradual tibial varus osteotomy with external fixation

    PREFERRED RESPONSE:2

     

    Question 5- At what age does longitudinal growth of the distal femoral growth plate typically stop in girls?

     

    1. 8

    2. 10

    3. 12

    4. 14

    5. 16

     

    PREFERRED RESPONSE:4

     

    Question6- Figures a through c are the radiographs of a 10-year-old with a 2-week history of anterior right knee pain after playing sports. Examination reveals no effusion and reproducible tenderness over the superior lateral patella. The extensor mechanism is intact. What is the most appropriate treatment?

     

     

     

    1. Medial patellofemoral ligament reconstruction

    2. Symptomatic treatment

    3. Arthroscopic lateral release

    4. Long-leg casting for 4 to 6 weeks

    5. Open reduction and internal fixation of the patella

    PREFERRED RESPONSE:2

    Question 7-A 15-year-boy has had unrelenting nonmechanical neck pain for 1 year. Aspirin is his only form of pain relief. The pathology seen in Figures 48a and 48b is typical of

     

     

    1. osteoblastoma.

    2. chondroblastoma.

    3. osteoid osteoma.

    4. Brodie abscess.

    5. aneurysmal bone cyst.

     

    Question 8-After closed reduction for a displaced pediatric supracondylar humeral fracture, pin removal is typically recommended during which postoperative time period?

    1. 1 week

  2. 3 weeks

  3. 6 weeks

  4. 9 weeks

  5. 12 weeks

PREFERRED RESPONSE:2

Question 9-Figure is a radiograph of a 14-year-old boy seen in the emergency department with right groin pain that occurred suddenly during a soccer game. What is the most appropriate treatment?

 

 

  1. Open surgical biopsy

  2. Urgent in situ hip pinning

  3. Urgent open reduction and internal fixation of the pelvis

  4. Open reduction and internal fixation via a hip dislocation approach

  5. Symptomatic treatment with partial weight bearing and physical therapy

PREFERRED RESPONSE:5

Question 10-A 13-year-old girl with idiopathic scoliosis was evaluated for maturity so clinicians could decide if she could safely discontinue brace treatment. A radiograph of her left hand revealed the middle phalangeal metaphysis was covered by the epiphysis, but capping was only present in 2 of the 4 metaphyses. With

which phase of curve acceleration and Risser sign does this degree of phalangeal maturity correlate?

  1. Risser 0 prior to maximal curve acceleration

  2. Risser 0 at maximal curve acceleration

  3. Risser I at maximal curve acceleration

  4. Risser II postmaximal curve acceleration

  5. Risser III postmaximal curve acceleration

 

Question 11-A 13-year-old female soccer player has acute hip pain after being blocked while kicking a goal.Examination revealed tenderness over the anterior iliac crest, acute pain on attempting a left straight-leg raise, and mild swelling in the groin. Radiographs revealed a minimally displaced avulsion fracture of the

iliac crest. What is the recommended treatment?

 

  1. Surgical reduction and screw fixation

  2. Spica cast with the hip bent for 6 weeks

  3. Crutch use and activity limitations until asymptomatic

  4. Crutch use and activity limitations for a minimum of 12 weeks

  5. Crutches and a knee immobilizer with the knee extended for 12 weeks

 

PREFERRED RESPONSE:3

 

Question 12-Which gene mutation is associated with the condition shown in Figures a and b?

 

 

 

 

  1. Sonic Hedgehog

  2. Collagen type I (COL1A2)

  3. Neurofibromatosis 1 (NF1)

  4. Low dietary intake of vitamin D

  5. Fibroblast growth factor receptor 3 (FGFR3)

 

PREFERRED RESPONSE:2

 

 

 

Question 13-The gene most likely linked to the findings in the MRI scan seen in Figure is

 

  1. EXT1.

  2. FGFR-1.

  3. FGFR-3.

  4. FBN1.

  5. collagen 1A.

 

PREFERRED RESPONSE:4

 

Question 14-Figures a through e are the radiographs and MRI scans of a 10-year-old boy who fell and sustained a closed type 1 supracondylar humeral fracture and was placed in a splint. He was referred to you and showed up 1 week later with a history of fever to 102°F for the past 2 days and increased pain in his arm.Upon removal of the splint the skin was intact without abrasion or laceration, but the arm had diffuse swelling and tenderness and any elbow motion was painful. He was neurologically intact. Laboratory studies showed a white blood cell count of 11,250/mm3, a C-reactive protein level of 4.5 mg/L (reference range, 0-3 mg/L), and an erythrocyte sedimentation rate of 42 mm/h (reference range, 0-20 mm/h).

Which surgical treatment is most appropriate?

 

 

 

 

 

  1. Arthroscopically drain the elbow.

  2. Aspirate the elbow and place it in a long-arm cast.

  3. Debride and drain the elbow through an open incision.

  4. Debride and drain the humeral diaphysis through a long lateral incision.

  5. Debride and openly drain both the humerus and elbow.

PREFERRED RESPONSE:5

Question 15-The C2 synchondrosis at the base of the dens typically closes at which age?

  1. 0 to 3 years

  2. 3 to 6 years

  3. 6 to 9 years

  4. 9 to 12 years

  5. 12 to 15 years

PREFERRED RESPONSE:2

Question 16-Figure is the radiograph of an 8-year-old boy who sustained a fracture after he fell from a 6-foot platform. What is the most appropriate treatment?

 

 

  1. Splint with orthopaedic follow-up in 3 days.

  2. Splint and perform closed reduction and pinning immediately.

  3. Splint and perform closed reduction and pinning the next day.

  4. Attempt closed reduction and splint with follow-up in 3 days.

  5. Obtain a skeletal survey and consult social services; splint and observe until social services finishes the evaluation.

 

PREFERRED RESPONSE:2

Question 17-Which pediatric elbow injury has a high association with child abuse?

  1. Nursemaid’s elbow

  2. Displaced lateral condyle fracture

  3. Displaced distal humerus physeal separation

  4. Displaced medial epicondyle avulsion fracture

  5. Displaced extension-type supracondylar humerus fracture

PREFERRED RESPONSE:3

 

Question 18-A 14-year-old girl has the insidious onset of left anterior tibial pain. The pain initially occurred primarily during ballet practice, but now occurs even with normal walking. Pain is minimal at night. She is a serious ballet student who plans a classical ballet career.

Examination reveals anterior midshaft tibial tenderness and radiographs reveal an anterior transverse midshaft lucency with surrounding bony sclerosis. Treatment should now include

  1. surgical debridement and internal fixation.

  2. a discussion of menstruation and eating habits.

  3. a nonweight-bearing bent knee cast for 12 weeks.

  4. cessation of ballet dancing for at least 1 year.

  5. biopsy and consideration for limb salvage surgery.

PREFERRED RESPONSE:2

 

Question 19-A 15-year-old boy underwent open reduction and internal fixation for a tibial tubercle fracture. The next morning he had dramatically increased pain, hypotension, ascending rash, and fever to 103°F. What is the most appropriate course of action?

 

  1. Close follow-up

  2. Begin a steroid dose pack

  3. Blood cultures followed by antibiotics only if the cultures are positive

  4. Order a CT scan (because of the metal implants) and follow up later that day

  5. Start antibiotics immediately and schedule a biopsy and debridement

 

PREFERRED RESPONSE:5

 

 

 

Question 20-Figure is the anteroposterior pelvic radiograph of a 5-year-old girl. What is the Risser classification?

 

1. 0

2. 1

3. 2

4. 4

5. 5

 

PREFERRED RESPONSE:1

 

Question 21-Which typical condition is linked to glenoid hypoplasia and retroversion?

 

  1. Neurofibromatosis

  2. Little League shoulder

  3. Neonatal hip dislocation

  4. Brachial plexus birth palsy

  5. Posterior shoulder impingement

PREFERRED RESPONSE:4

 

Question 22-Figures a and b are the posteroanterior and lateral radiographs of a 13-year-old girl with a progressive curve despite bracing with a thoracolumbosacral orthosis. Examination reveals no pain or neurologic findings. The lumbar curve measures 59 degrees and the thoracic curve measures 52 degrees.The most appropriate treatment is

 

 

 

  1. spinal manipulations.

  2. posterior spinal fusion.

  3. anterior/posterior spinal fusion.

  4. spine staples placed thorascopically.

  5. changing to a ‘spine-core’ flexible brace.

 

PREFERRED RESPONSE:2

 

 

 

Question 23-A likely candidate for treatment with a thoracic lumbosacral orthosis scoliosis brace is seen in

 

  1. Figure 178a.

  2. Figure 178b.

  3. Figure 178c.

  4. Figure 178d.

  5. Figure 178e.

 

PREFERRED RESPONSE:4

 

 

 

Question 24-A 7-year-old boy sustained transverse amputations of the ring, long, and index fingers when his hand was caught in a bicycle chain. All amputations were distal to the nail base. He was treated in the emergency department with debridement of the projecting bones and an application of dressings. Upon removal of the dressings 4 days later, the hand appeared as seen in Figure 186. Which treatment is likely to lead to the best functional and cosmetic result?

 

  1. Split-thickness skin graft

  2. Full-thickness skin graft

  3. Volar V-Y advancement flaps

  4. Lateral and medial V-Y advancement flaps

  5. Continued treatment with dressing changes

 

PREFERRED RESPONSE:5

 

Question 25-Which hand malformation is associated with the anomaly shown in Figure ?

 

 

  1. Radial club

  2. Symbrachydactyly

  3. Thumb polydactyly

  4. Complex atypical syndactyly (Apert syndrome)

  5. Amniotic band syndrome

PREFERRED RESPONSE:2

 

Question 26-Figure is the radiograph of a 16-year-old with cerebral palsy and left hip pain who meets Gross Motor Function Classification System level V criteria. What is the most appropriate treatment?

 

 

 

 

  1. Proximal femoral resection

  2. Cortisol injection to the hip

  3. Varus derotational osteotomy

  4. Varus derotational and pelvic osteotomies

  5. Soft-tissue release of the adductors and hip flexors

 

Question 27-A 15-year-old boy is seen in the emergency department with a large knee effusion. Knee aspiration revealed a white blood cell count of 50,000/μL (reference range, 4,500-11,000 μL). History reveals that he recently attended a summer camp in Connecticut. His fluorescent antibody blood test is positive for

Lyme disease. What is the most appropriate next step in treatment?

 

  1. Await synovial Lyme culture

  2. Tap and evaluate the spinal fluid

  3. IV vancomycin without surgical lavage

  4. Oral doxycycline and arthroscopic lavage

  5. Oral doxycycline without surgical lavage

PREFERRED RESPONSE:5

 

Question 28-What is the most common type of neonatal brachial plexus palsy?

 

  1. Suprascapular

  2. Horner syndrome

  3. Upper-trunk injury

  4. Lower-trunk injury

  5. Global plexus injury C6 and T1

PREFERRED RESPONSE:3

 

Question 29-Figures a and b are the radiographs of a 12-year-old girl who fell while walking down the stairs. When she is seen 2 hours after the injury, she is in pain and is unable to walk. What is the most appropriate treatment option?

 

 

 

  1. Closed reduction and spica casting

  2. Closed reduction, capsular decompression, and screw fixation

  3. Open reduction, pinning, and bone grafting

  4. Open reduction with sliding blade plate fixation

  5. Open reduction and internal fixation with a cephalomedullary nail

PREFERRED RESPONSE:2

 

Question 30-An 11-month-old infant has not started crawling and was late in sitting independently. He has good head control, appropriate social responses, and apparently normal hand function. Knee reflexes are absent, though a biceps reflex is present. Mild fasciculations are seen in the tongue. The mother’s prenatal course, labor, and delivery were uneventful, and the child was delivered at term. The most likely diagnosis is

 

  1. athetotic cerebral palsy.

  2. Duchenne muscular dystrophy.

  3. spinal muscular atrophy, type I.

  4. spinal muscular atrophy, type II.

  5. cerebral palsy with spastic diplegia.

PREFERRED RESPONSE:4

 

Question 31-Figures a and b are the radiographs of a 13-year-old girl with right hip pain. An MRI scan would most likely show

 

 

 

  1. a degenerative labral tear.

  2. an intra-articular loose body.

  3. a femoral neck stress fracture.

  4. metaphyseal edema of the right hip.

  5. avascular necrosis of the right femoral head.

 

Question 32-Which percentage of boys sustain a fracture before age 16?

 

  1. 0% to 5%

  2. 10% to 15%

  3. 20% to 30%

  4. 40% to 60%

  5. 80% to 100%

PREFERRED RESPONSE:4

 

Question 33-An infant was born with complex syndactyly involving all 4 fingers of both hands, short and deformed thumbs, and similar syndactyly involving both feet. In addition, an altered facial appearance was noted with protruding eyes, a towered cranium, and midface hypoplasia. This appearance is characteristic of which syndrome?

 

  1. Apert

  2. Poland

  3. Holt-Oram

  4. VACTERRL

  5. Thrombocytopenia-absent radius (TAR)

PREFERRED RESPONSE:1

 

 

 

Question 34-Figure is the hip ultrasound of a 6-month-old boy. Based on the image findings, what is the most likely diagnosis?

 

  1. Labral tear

  2. Joint effusion

  3. Snapping psoas tendon

  4. Graf type I hip (no dysplasia)

  5. Graf type III hip dysplasia (dysplasic)

 

Question 35-A 10-year-old boy sustained a displaced Salter-Harris type II supination/plantar flexion fracture of the left ankle. He underwent closed reduction under conscious sedation; however, postreduction radiographs showed continued 5-mm anterior widening of the tibial physis. What is the most likely cause of the

widening?

 

  1. Interposed bony fragment

  2. Interposition of the periosteum

  3. Interposition of the anterior tibial tendon

  4. Persistent malrotation of the physis

  5. Fibular plastic deformation and malalignment

 

PREFERRED RESPONSE:2

 

 

 

 

 

Question 1 .A 7-month-old infant is evaluated after right brachial plexus birth palsy. Examination reveals scapular winging and no active elbow flexion. Chest radiographs reveal an elevated right hemidiaphragm. The best next step should be

  1. free functional gracilis muscle transfer.

  2. observation and re-examination in 3 months.

  3. therapy to maintain shoulder internal rotation and elevation.

  4. direct brachial plexus repair with sural nerve grafting if necessary.

  5. musculocutaneous neurotization from ulnar/median donor fascicles.

 

PREFERRED RESPONSE: 5

 

Question 2 .A patient is unable to actively externally rotate the shoulder when the arm is placed into 90 degrees of abduction and neutral rotation. This finding is most consistent with a tear of the

  1. biceps tendon.

  2. isolated subscapularis.

  3. isolated supraspinatus.

  4. superior and anterior labrum.

  5. infraspinatus and teres minor.

PREFERRED RESPONSE: 5

 

 

 

Question 3 .The innervation to the upper portion of the structure noted in Figure 36 arises directly from what aspect of the brachial plexus?

 

  1. Medial cord

  2. Lateral cord

  3. Posterior cord

  4. Upper trunk

  5. C5-C7 roots

 

Question4 .Figures 41a through 41c are the radiograph and MRI scans of a 76-year-old woman who has intractable left shoulder pain. She was given 2 cortisone injections and oral pain medication without experiencing lasting relief. Examination reveals 60 degrees of active forward elevation (120 degrees passively), 30 degrees of external rotation lag, and a positive Hornblower sign. Pain relief and improved functionality will most likely be achieved with

 

 

  1. continued nonsurgical treatment.

  2. hemiarthroplasty with partial rotator cuff repair.

  3. reverse total shoulder arthroplasty with latissimus dorsi transfer.

  4. rotator cuff repair without acromioplasty, preserving the coracoacromial ligament.

  5. limited-goals debridement of the rotator cuff and glenohumeral joint without rotator cuff repair.

PREFERRED RESPONSE: 3

 

 

 

Question5 .Figure 53 is the CT scan of a 38-year-old woman who has pain with movement of her right arm and shortness of breath after an assault. She is evaluated in the emergency department and her shoulder radiograph findings are normal. The physician should recommend

 

  1. a chest tube.

  2. a sling and outpatient follow up.

  3. closed reduction.

  4. incision and drainage.

  5. an MRI scan of the shoulder.

 

Question6 .A 40-year-old man with a history of a nondisplaced radial head fracture was initially treated with cast immobilization for 3 weeks followed by a course of physical therapy. Six months later, he has limited elbow range of motion. Examination reveals he lacks 30 degrees of extension and has flexion to only 90 degrees. To restore flexion, which structure must be released?

  1. Triceps tendon

  2. Anterior capsule

  3. Ulnar part of the lateral collateral ligament

  4. Anterior bundle of the medial collateral ligament

  5. Posteromedial bundle of the medial collateral ligament

PREFERRED RESPONSE: 5

Question 7 .Figures 89a and 89b are the radiographs of an 18-year-old woman who has had elbow pain after falling on an outstretched hand. She is evaluated 5 days after the injury.

 

 

Examination reveals the wrist is normal and her elbow has a limited arc of motion of 30 to 90 degrees of flexion/extension and 20 to 20 degrees of pronation and supination, with tenderness isolated to the lateral side of the elbow. What is the most appropriate treatment option?

 

  1. Cast for 2 weeks

  2. Initiate mobilization

  3. Radial head excision

  4. Radial head replacement

  5. Open reduction and internal fixation

 

PREFERRED RESPONSE: 2

 

 

 

Question 8 .Figures 101a and 101b are the radiographs of a 50-year-old man who has difficulty with overhead work following a superior labrum anterior to posterior (SLAP) repair 12 months ago. He had no early postsurgical complications and was in therapy for 9 months after surgery. Examination of the shoulder reveals 110 and 45 degrees of active elevation and active external rotation with his arm at his side,respectively. His passive range of motion is symmetric to his active range of motion. What is the best treatment option?

 

  1. Arthroplasty

  2. Acromioplasty

  3. Continue therapy

  4. Revision SLAP repair

  5. Arthroscopic capsular release PREFERRED RESPONSE: 5

 

Question 9 .Figures 118a through 118d are the radiographs and selected CT sequences of a 24-year-old man who had arthroscopic Bankart repair 1 year ago after a traumatic dislocation. He has recurrent instability and denies any recent trauma. He has instability with even trivial activities and states his shoulder was“never really stable” after his surgery. He has been to the emergency department on multiple occasions for manipulative reduction.

Examination reveals good muscle tone and bulk around the shoulder. He has no external rotation weakness, and his abdominal compression test is normal. He has a markedly positive anterior apprehension sign in the 90-degree abducted, externally rotated position and in midabduction.The sulcus sign is negative and there is no evidence of posterior instability.

What is the best treatment option?

 

 

 

  1. Physical therapy

  2. Arthroscopic rotator cuff repair

  3. Revision arthroscopic Bankart repair

  4. Bone grafting of the Hill-Sachs lesion

  5. Transfer of the coracoid process to the anterior glenoid neck PREFERRED RESPONSE: 5

 

Question10 .A 45-year-old woman has elbow stiffness 3 months after treatment of an elbow dislocation consisting of self-directed exercises. Examination reveals that her elbow is stable. Range-of-motion testing reveals a 35-degree flexion contracture, full flexion, and 80 degrees of both pronation and supination. What is the best next treatment step?

  1. Hinged elbow brace

  2. Open contracture release

  3. Arthroscopic debridement and release

  4. Supervised therapy with splinting

  5. Examination under anesthesia and manipulation

PREFERRED RESPONSE: 4

 

Question11 .A 22-year-old collegiate baseball pitcher has had posterior shoulder pain with decreased throwing velocity and accuracy over the past several months. Examination of the abducted shoulder in the supine position reveals 120 degrees of external rotation, 40 degrees of internal rotation on the throwing side, 100 degrees of external rotation, and 70 degrees of internal rotation on the nonthrowing side. The remainder of the clinical examination is unremarkable. An MRI scan shows a small partial articular-sided infraspinatus tear. Initial treatment should consist of

  1. arthroscopic rotator cuff repair.

  2. arthroscopic anterior capsulorrhaphy.

  3. arthroscopic selective posterior capsular release.

  4. selective posterior rotator cuff strengthening.

  5. posterior capsular stretching with scapular stabilization.

PREFERRED RESPONSE: 5

 

 

 

Question 12 .Which factor is associated with an increased risk for the complication shown in Figure 166 following reverse total shoulder

arthroplasty?

 

  1. Failed previous arthroplasty

  2. Anterosuperior approach

  3. Complete repair of the subscapularis

  4. Inferior inclination of the glenosphere baseplate

  5. Humeral stem placement in 10 degrees’ retroversion

 

Question13 .Figures 174a through 174d are the radiographs and selected MRI sequences of a 35-year-old man with a history of alcoholism. He has right shoulder pain that has been progressively worsening over the past several years. Examination reveals active forward elevation of 150 degrees, external rotation of 50 degrees with his arm by his side, and internal rotation to the T-12 vertebral level. He had a cortisone injection, but experienced no relief. What is the best treatment option?

 

 

 

  1. Hemiarthroplasty

  2. Core decompression

  3. Arthroscopy and capsular release

  4. Intra-articular hyaluronate injections

  5. Physical therapy and nonsteroidal anti-inflammatory medication

 

Question14 .Figures 194a and 194b are the radiographs of a 59-year-old right-hand-dominant woman who has pain in her dominant shoulder following a fall 1 day ago. Examination reveals tenderness over the proximal humerus and ecchymosis about the midarm. She is distally neurovascularly intact. What is the most appropriate treatment option?

 

 

  1. Hemiarthroplasty

  2. Reverse total shoulder arthroplasty

  3. Open reduction and internal fixation

  4. Sling immobilization for 6 weeks followed by passive range of motion

  5. Symptomatic sling use followed by early active range-of-motion exercises

 

PREFERRED RESPONSE: 3

 

Question 15 .A woman who underwent an unconstrained total shoulder arthroplasty fell 6 weeks after surgery. She now has a documented anterior shoulder dislocation and undergoes closed reduction. All 3 heads of the deltoid contract, passive elevation is limited by pain to 30 degrees, and her internal rotation is to the side and her passive external rotation is 80 degrees. At the 2-week postsurgical visit, her elevation is 60 degrees,external rotation is 10 degrees, and internal rotation is to the side. Radiographs reveal no loosening, fractures, or dislocations. Further evaluation should consist of

  1. an indium scan.

  2. C-reactive protein.

  3. electromyography.

  4. aspiration of the shoulder.

  5. ultrasound of the shoulder.

 

PREFERRED RESPONSE: 5

 

Question16 .A fall onto an outstretched arm places an axial load on the wrist and forearm. What other combination of forces at the elbow leads to a terrible triad of radial head fracture, coronoid fracture, and lateral collateral ligament injury?

 

  1. Forearm supination and varus thrust

  2. Forearm pronation and varus thrust

  3. Forearm supination and valgus thrust

  4. Forearm pronation and valgus thrust

  5. No rotational forces, only axial loading

PREFERRED RESPONSE: 3

 

Question17 .Figures 243a through 243d are the plain radiographs and selected sequences from an MRI scan of a 52-year-old man with a history of prior arthroscopic rotator cuff repair. He has persistent pain and limited range of motion. Examination reveals no deltoid atrophy, but significant atrophy of the infraspinatus.He has active overhead elevation of 140 degrees with a painful arc and significant weakness of shoulder external rotation both with his arm by his side and in abduction. Both the lift-off and abdominal compression tests are within defined limits. What is the best treatment option?

 

 

  1. Hemiarthroplasty

  2. Superior labral repair

  3. Latissimus dorsi transfer

  4. Reverse total shoulder arthroplasty

  5. Revision arthroscopic rotator cuff repair

PREFERRED RESPONSE: 3

 

Question18 .A 40-year-old man who had an intra-articular supracondylar humerus fracture was treated by bicolumnar plating 1 year ago. He has pain and dysfunction of the elbow. His range of motion is from 30 to 90 degrees of flexion-extension and 80 to 80 degrees of pronation-supination. The mid arc of motion is pain free. He has medial-sided elbow pain, reproduced with forced elbow flexion. Radiographs reveal a healed fracture, no hardware breakage, and mild joint space incongruency with a well-maintained joint space. What is the most appropriate treatment?

 

  1. Hardware removal

  2. Arthroscopic debridement

  3. Osteotomy and reconstruction

  4. Manipulation under anesthesia

  5. Ulnar nerve decompression and capsular release

PREFERRED RESPONSE: 5

 

Question19 .Figures 255a through 255c are the radiographs and MRI scan of a 73-year-old man who has severe pain and functional disability of the right shoulder despite receiving several cortisone injections and physical therapy. Examination reveals restricted shoulder range of motion in forward elevation and both internal and external rotation. There is moderately diminished strength and pain with resisted forward elevation.What is the best treatment option?

 

 

 

 

  1. Reverse total shoulder arthroplasty

  2. Unconstrained total shoulder arthroplasty

  3. Hemiarthroplasty with biologic glenoid resurfacing

  4. Arthroscopic subacromial decompression

  5. Arthroscopic capsular release with manipulation under anesthesia

 

PREFERRED RESPONSE: 2

 

 

 

Question20 .Figure 266 is the anteroposterior radiograph of a 6-year-old boy who sustained an injury to his left elbow after a fall. Examination of his elbow reveals intact skin. There is tenderness over the radial head, but he is nontender elsewhere, including his wrist. His distal neurovascular examination is unremarkable. A closed reduction was attempted; however, there was no improvement in position. What is the best next treatment option?

 

  1. Radial head arthroplasty

  2. Long-arm cast for 4 weeks

  3. Open reduction and plate fixation

  4. Immediate range of motion and physical therapy

  5. Percutaneous Kirschner wire-assisted reduction and casting

 

PREFERRED RESPONSE: 5

 

Question 1-A 16-year-old female swimmer had bilateral shoulder pain for 3 months that limited her ability to complete workouts. Her shoulder range of motion was symmetric. Rotator cuff and deltoid strength findings were normal. Her Hawkins impingement sign and speed test results were positive on both shoulders. Radiograph and MRI scan findings were normal. Which intervention is the next step in evaluation and treatment?

  1. Therapy

  2. CT scans

  3. Electromyography

  4. Cortisone injections

  5. Arthroscopic acromioplasty

PREFERRED RESPONSE:1

Question 2-The longest average survivorship for total elbow arthroplasty has been reported in patients who undergo replacement for which diagnosis?

  1. Fracture

  2. Flail elbow

  3. Osteoarthritis

  4. Rheumatoid arthritis

  5. Posttraumatic arthritis

PREFERRED RESPONSE:4

 

Question 3-Which risk factor is most commonly associated with cuff nonhealing after repair?

  1. Smoking status

  2. Age older than 65

  3. Smaller cuff tears

  4. Early repair of an acute tear

  5. Workers’ compensation claim

PREFERRED RESPONSE:2

 

Question 4-Which artery provides the dominant blood supply to the humeral head?

  1. Deltoid

  2. Suprascapular

  3. Thorocoacromial

  4. Anterior humeral circumflex

  5. Posterior humeral circumflex

PREFERRED RESPONSE:5

Question 5-Figures a and b are the plain radiographs of a 52-year-old right-hand-dominant woman with a 6-month history of atraumatic left shoulder pain and limited range of motion, both of which affect her functional level. She complains of both rest and night pain and denies any constitutional symptoms. Past medical history is notable only for hypothyroidism. Examination reveals no muscle atrophy, but she has active and passive global loss of motion. She has marked pain with passive stretch of the glenohumeral joint. An MRI scan reveals the rotator cuff is intact and no labral tear is evident. The etiology of the clinical condition is thought to be related to

 

 

 

  1. crystal arthropathy.

  2. an autoimmune process.

  3. an infectious process.

  4. a fibroblastic process.

  5. a chondrolytic process.

PREFERRED RESPONSE:4

Question 6-A 20-year-old collegiate pitcher has a 6-week history of pain and stiffness when throwing, which has caused a decrease in maximal velocity and the inability to pitch competitively. Examination reveals tenderness over the medial collateral ligament. An MRI scan shows increased signal in the anterior band of the ulnar collateral ligament without a full-thickness tear. What is the most appropriate treatment?

  1. Immobilization of the elbow

  2. Continue throwing as tolerated

  3. Arthroscopic evaluation and debridement

  4. Medial collateral ligament reconstruction

  5. Flexor pronator strengthening and gradual return to sports

 

PREFERRED RESPONSE:5

Question 7-Figures a and b are the plain radiographs of a 26-year-old man with an elbow contracture. He denies any specific elbow trauma but reports a history of a closed-head injury sustained in a motor vehicle collision. Examination reveals the elbow lacked 55 degrees of extension and has flexion of 85 degrees.Supination and pronation are well preserved. Release of which structure is essential to restore elbow flexion?

 

 

  1. Radial head

  2. Anterior capsule

  3. Olecranon fossa osteophytes

  4. Lateral ulnar collateral ligament

  5. Posterior oblique band of the medial collateral ligament

PREFERRED RESPONSE:5

 

 

 

Question 8-What should rehabilitation include during the initial 3 weeks after an arthroscopic repair of the lesion seen in Figure ?

 

  1. Resistive biceps exercises

  2. Complete immobilization in a sling

  3. Isotonic posterior rotator cuff exercises

  4. External rotation stretching at 90 degrees of abduction

  5. Passive- and active-assisted elevation in the scapular plane

 

PREFERRED RESPONSE:5

 

Question 9-When is heterotopic ossification around the elbow considered mature following an injury?

 

  1. Six months after the injury

  2. One year after the injury

  3. Elbow motion plateaus

  4. Bone scan findings are negative

  5. Sharp cortical margins have developed on radiographs.

PREFERRED RESPONSE:5

 

Question 10-Figure 141 is the MRI scan of a 68-year-old man referred by his primary care physician for evaluation of his shoulder. Examination reveals atrophy in the supraspinatus and infraspinatus fossa. He denies pain but has moderate weakness in empty can and external rotation strength testing. What is the most appropriate treatment recommendation?

 

 

 

  1. Nonsurgical treatment

  2. Reverse total shoulder arthroplasty

  3. Rotator cuff repair with acromioplasty, resecting the coracoacromial ligament

  4. Rotator cuff repair without acromioplasty, preserving the coracoacromial ligament

  5. Limited-goals debridement of the rotator cuff and glenohumeral joint without rotator cuff repair

PREFERRED RESPONSE:1

Question 11-Which structure is labeled by the arrow seen in Figure ?

 

 

 

  1. Biceps tendon

  2. Subscapularis tendon

  3. Middle glenohumeral ligament

  4. Inferior glenohumeral ligament

  5. Superior glenohumeral ligament PREFERRED RESPONSE:3

 

 

 

Question 12-A 65-year-old man with glenohumeral arthritis is scheduled to undergo shoulder arthroplasty. Apreoperative CT scan is shown in Figure. His rotator cuff is intact. What is the most appropriate surgical technique?

 

  1. Reverse shoulder arthroplasty

  2. Eccentric reaming of the anterior glenoid

  3. Bone grafting of the posterior glenoid face

  4. Increase anteversion of the humeral component

  5. Posterior opening-wedge osteotomy of the glenoid

 

PREFERRED RESPONSE:2

 

Question 13-Figures a and b are the radiographs of a 25-year-old man seen 3 days after a posterior lateral elbow dislocation. He has a feeling of subluxation when his arm is extended past 20 degrees. The physician should recommend

 

 

 

  1. a CT scan.

  2. an MRI scan.

  3. a posterior splint for 2 weeks.

  4. therapy with a 20-degree extension block.

  5. lateral collateral ligament reconstruction.

 

PREFERRED RESPONSE:4

 

Question 14-Figures a through d are the plain radiographs and selected CT scans of a 67-year-old man with intractable right shoulder pain and severe limitation of range of motion. His symptoms have been refractory to nonsurgical treatment measures. An MRI scan reveals a partial-thickness articular-sided tear of the supraspinatus. The remainder of his rotator cuff is intact. What is the best treatment option?

 

 

  1. Hemiarthroplasty

  2. Arthroscopic capsular release

  3. Arthroscopic rotator cuff repair

  4. Reverse total shoulder arthroplasty

  5. Total shoulder arthroplasty with rotator cuff repair

PREFERRED RESPONSE:5

 

Question 15-Which treatment is most important in managing a ‘terrible triad’ elbow fracture-dislocation?

  1. Anterior capsular repair

  2. Resection of the radial head

  3. Four weeks of postoperative casting

  4. Repair of the medial collateral ligament

  5. Repair of the lateral collateral ligament

PREFERRED RESPONSE:5

 

 

 

Question 16-A 60-year-old man shown in Video 258 has shoulder pain and difficulty lifting his right arm overhead.This abnormality

is the result of dysfunction in which nerve?

 

  1. Long thoracic

  2. Thoracodorsal

  3. Spinal accessory

  4. Dorsal scapular

  5. Suprascapular

 

PREFERRED RESPONSE:3

 

Question 17-A collegiate offensive football lineman has decreased bench-press strength and shoulder pain as off-season workouts begin. Examination revealed no atrophy, and deltoid and rotator cuff strength testing findings were normal. Translational testing was difficult to achieve because of his large size.Apprehension and relocation test findings were negative. An O’Brien’s active compression test result was negative. Jerk testing was positive on the affected side. Which diagnosis is most likely revealed on an MRI arthrogram?

 

  1. SLAP tear

  2. Subscapularis tear

  3. Supraspinatous tear

  4. Anterior labral tear

  5. Posterior labral tear

 

PREFERRED RESPONSE:5

 

 

 

Question 1 -Figures 7a through 7c are the radiograph and MRI scans of a 72-year-old woman

who has had back and leg pain for 3 months. Her pain is worse with prolonged walking and relieved with bending forward.Examination reveals normal strength and sensation in her legs with intact pedal pulses. She has persistent pain despite physical therapy, medications, and epidural injections. What is the most appropriate treatment option?

 

 

 

  1. Laminectomy

  2. Laminectomy and instrumented fusion

  3. Laminectomy and uninstrumented fusion

  4. Endovascular aortic bypass

  5. Anterior lumbar interbody fusion

PREFERRED RESPONSE: 2

 

 

Question 2 -Figure 19 is the T2-weighted MRI scan of a 25-year-old man who is seen in the emergency department after falling off of a roof. Examination revealed he has 3/5 strength in his bicep muscles bilaterally but no motor or sensory function in his hands. For this type of injury, early decompression within 24 hours gives what advantage?

 

  1. Reduced mortality

  2. Improved neurologic outcomes

  3. Lower risk for pulmonary embolus

  4. Decreased incidence of hospital readmission

  5. Earlier discharge to a skilled rehabilitation facility

 

PREFERRED RESPONSE: 2

 

Question3 -Figures 27a and 27b are the MRI scans of a 31-year-old woman with low-back and left leg pain radiating into her posterior thigh and calf for 2 weeks. Examination reveals a positive straight-leg raise, normal strength, and normal sensation in the lower extremities.

What is the most appropriate treatment option?

 

 

 

 

  1. Nonsurgical care

  2. Microdiscectomy

  3. Subtotal discectomy

  4. Anterior decompression and fusion

  5. Posterior decompression and fusion

PREFERRED RESPONSE: 1

Question4 -A physician shows interest in determining the evidence base for use of a specific interbody fusion technique in the treatment of lumbar degenerative disc disease. A search of the literature reveals 4 studies that retrospectively reviewed outcomes for series comprising fewer than 20 patients each. Another study retrospectively compared results of the interbody fusion technique to posterolateral fusion. All of the studies reported satisfactory outcomes for the interbody fusion technique, while the comparative study found interbody fusion to be superior to posterolateral fusion. The quality of evidence supporting the use of the interbody fusion technique would be graded as

 

  1. B (fair-quality evidence), attributable to the fact that a single level III study supports use of the interbody fusion technique

  2. B (fair-quality evidence), attributable to the fact that multiple level IV studies and a single level III study support its use

  3. C (poor-quality evidence), attributable to the fact that multiple level IV studies and a single level III study support its use

  4. C (poor-quality evidence), attributable to the fact that multiple level V studies and a single level III study support its use

  5. I (no evidence), attributable to the fact that the studies found in the literature are of insufficient quality to allow recommendation in support of the technique

PREFERRED RESPONSE: 3

 

Question5 -Figures 45a and 45b are the sagittal and axial T2-weighted MRI scans of a 39-year-old man with a 3-month history of symptoms. Examination findings are most likely to indicate decreased sensation in the left

 

 

  1. upper arm, with weakness in the biceps.

  2. index finger, with weakness in the hand intrinsics.

  3. small finger, with weakness in the wrist extension.

  4. middle finger, with weakness in the wrist flexion.

  5. radial forearm, with weakness in shoulder abduction.

PREFERRED RESPONSE: 4

Question 6 -Figures 55a and 55b are the radiograph and CT scan of a 61-year-old woman who has had neck pain after being involved in a high-speed motor vehicle collision.

Examination reveals normal strength and sensation in both upper and lower extremities, normal rectal tone, and no other injuries. The C1-C2 lateral mass overhang measures 8.5 mm. What is the most appropriate treatment option?

 

 

  1. Halo-vest orthosis

  2. C1-C2 posterior cervical fusion

  3. Occiput to C2 posterior cervical fusion

  4. Cervical traction and closed reduction

  5. Open reduction and internal fixation of C1

 

PREFERRED RESPONSE: 2

 

 

Question7 -Figure 67 is the MRI scan of a 43-year-old man with an acute onset of neck pain, bilateral upper-extremity weakness, and burning pain in his arms after hitting his head on a bookshelf. Examination initially revealed 3/5 strength in both upper extremities, with normal motor strength in the lower extremities. What is the best description of his spinal cord injury?

 

  1. Central cord syndrome

  2. Anterior cord syndrome

  3. Posterior cord syndrome

  4. Brown-Séquard syndrome

  5. Complete spinal cord injury

 

PREFERRED RESPONSE: 1

 

Question8 -Figures 93a through 93c are the radiograph and CT and MRI scans of a 35-year-old man with diabetes mellitus. He had severe neck pain for 6 weeks. He now has fevers and progressive weakness and numbness in his upper extremities. Examination reveals 4/5 strength in both upper extremities, with decreased sensation in both arms and hands and hyperreflexia. What is the most appropriate treatment option?

 

 

  1. Halo-vest fixation

  2. Intravenous antibiotics

  3. Posterior laminectomy

  4. Percutaneous aspiration

  5. Circumferential decompression and fusion

 

PREFERRED RESPONSE: 5

 

Question9 -For patients undergoing posterior lumbar fusion, which risk factor is most responsible for development of adjacent segment degeneration that necessitates further surgery?

 

  1. Male gender

  2. Single-level construct

  3. Patient age younger than 45 years

  4. Extension of the fusion to the sacrum

  5. Laminectomy adjacent to the fusion

PREFERRED RESPONSE: 5

 

Question 10 -Figures 124a and 124b are the MRI scans of a 74-year-old man who has difficulty walking distances attributable to pain in both lower extremities. His leg pain is worse with lumbar extension and improves with forward flexion. Examination reveals full strength and sensation in all 4 extremities. He shows hyperreflexia and gait imbalance. He has tried physical therapy, medications, and epidural injections without experiencing symptom relief. What is the most appropriate next step?

 

 

 

 

  1. Daily lumbar traction

  2. Referral to pain management

  3. Lumbar decompression surgery

  4. Spinal cord stimulator placement

  5. An MRI scan of the cervical spine

 

PREFERRED RESPONSE: 5

 

Question11 -Figures 141a and 141b are the lumbar CT scans of a 16-year-old baseball pitcher who has had low-back pain for 3 months. He has no radiating pain, numbness, or weakness. His pain is worsened by extension and relieved with flexion. Examination reveals normal strength and sensation in his lower extremities.What is the most likely diagnosis?

 

 

  1. Spondylosis

  2. Spondylolysis

  3. Spondylolisthesis

  4. Osteoid osteoma

  5. Congenital dysplasia

PREFERRED RESPONSE: 2

Question12 -Figure 162 is the CT scan of a 74-year-old woman who struck her head during a ground-level fall and has severe neck pain. Examination reveals normal strength and sensation in her upper and lower extremities.What is the most appropriate treatment option?

 

 

 

  1. Cervical traction

  2. Halo-vest orthosis

  3. Anterior single-level fusion

  4. Posterior single-level fusion

  5. Posterior multilevel fusion

 

PREFERRED RESPONSE: 5

 

 

 

Question 13 -Figure 181 is the MRI scan of a 59-year-old woman who has had no medical comorbidities but has had difficulty with walking and balance for the past 6 months. She has severe pain in her neck and arms as well as clumsiness and weakness in her arms. Examination reveals hyperreflexia in her upper and lower extremities, a positive Hoffmann sign, and inability to perform rapid alternating movements. What intervention would most likely produce the best long-term result?

 

  1. Immobilization

  2. Physical therapy

  3. Surgical decompression

  4. Neurology consultation

  5. Cervical epidural injection PREFERRED RESPONSE: 3

Question 14 -Figures 190a and 190b are the sagittal and axial T2-weighted MRI scans of a 75-year-old man who is experiencing progressively worsening bilateral lower-extremity pain and difficulty walking distances. In another 4 years, nonsurgical treatment of his condition--compared to surgical treatment--is expected to result in

 

 

 

  1. equal improvement in pain.

  2. equal improvement in function.

  3. less improvement in pain.

  4. more improvement in pain.

  5. more improvement in function.

 

PREFERRED RESPONSE: 3

 

 

 

Question15 -Figure 198 is the T2 sagittal MRI scan of a 47-year-old woman who has experienced pain in her lower back and difficulty walking distances during the past 3 years. She has tried physical therapy, nonsteroidal anti-inflammatory drugs, and multiple epidural injections without symptom relief. Which surgical treatment is associated with the best outcome?

 

  1. L4-L5 microdiscectomy

  2. L4-L5 anterior interbody fusion

  3. L4-L5 laminectomy

  4. L4-L5 laminectomy and posterior uninstrumented fusion

  5. L4-L5 laminectomy and posterior instrumented fusion

 

PREFERRED RESPONSE: 5

 

Question16 -Figures 222a and 222b are the radiograph and MRI scan of a 41-year-old man who has had severe leg pain for 6 months despite physical therapy and medications.

 

 

Examination reveals normal strength and sensation in both lower extremities. What is the most effective treatment option?

 

  1. Lumbar interlaminar epidural injection

  2. Lumbar transforaminal epidural injection

  3. Posterior lumbar laminectomy

  4. Posterior lumbar laminectomy and fusion

  5. Posterior lumbar laminectomy and interbody fusion

 

PREFERRED RESPONSE: 3

 

 

 

Question17 -Figure 236 is the lateral radiograph of a 77-year-old man who had neck pain after a low-speed motor vehicle collision. He had diffuse tenderness to palpation over his posterior cervical spine but the remainder of the examination was unremarkable. Plain radiographs including Figure 236 were negative for any evidence of fracture. What is the best next step in management?

 

  1. Pulmonary function testing

  2. An MRI scan of the cervical spine

  3. Physical therapy and oral steroids

  4. Immobilization in a rigid cervical collar for 6 weeks

  5. Referral to a rheumatologist

 

PREFERRED RESPONSE: 2

 

Question18 -Figures 247a through 247e are the lateral radiographs of different spinal surgical procedures. Which figure shows the surgical procedure for which rhBMP-2 has been FDA approved?

 

 

  1. Figure 247a

  2. Figure 247b

  3. Figure 247c

  4. Figure 247d

  5. Figure 247e

PREFERRED RESPONSE: 1

 

 

 

Question19 -Figure 260 is the MRI scan of an 84-year-old woman who is admitted to the hospital with pain in her midback that started 8 weeks ago after a fall from a standing height. Examination reveals normal strength and sensation. Her reflexes and rectal tone are normal. According to the most recent AAOS clinical practice guideline, The Treatment of Symptomatic Osteoporotic Spinal Compression Fractures, which recommendation is based on conclusive evidence for the treatment of her pain?

 

  1. Brace

  2. Calcitonin

  3. Ibandronate

  4. Bed rest and opioids

  5. A supervised exercise program PREFERRED RESPONSE: 3

Question20 -Figures 271a and 271b are the MRI scans of a 49-year-old woman with pain that has been radiating down her right lower extremity to the dorsum of her foot for the last 3 weeks. Examination reveals significant pain with a straight-leg raise and decreased sensation in the right big toe. What is the most appropriate next step in management?

 

 

  1. Right L4-L5 microdiscectomy

  2. Posterior L4-L5 decompression and fusion

  3. Trial of nonsteroidal anti-inflammatory drugs

  4. Neurology referral for nerve conduction studies

  5. Transcutaneous electrical nerve stimulation treatment for 3 hours daily

 

PREFERRED RESPONSE: 3

 

Question21-Figures 274a and 274b are the MRI scans of a 53-year-old woman who has had severe, increasing midback pain for 3 weeks. She had a small bowel transplant 10 years ago. Examination reveals tenderness to palpation over the thoracic spine. She has full strength and sensation in all extremities, is normoreflexic throughout, and is currently afebrile. What is the most appropriate next step in management?

 

 

 

 

  1. Whole-body bone scan

  2. Outpatient physical therapy

  3. CT-guided biopsy of the thoracic spine

  4. Thoracic decompression and stabilization

  5. Referral to pain management for an epidural injection

 

PREFERRED RESPONSE: 3

 

Question 1-Figures a and b are the anteroposterior and lateral plain radiographs of a 45-year-old woman who had severe bilateral leg pain for 6 months. Figures 5c and 5d are her sagittal and axial T2-weighted MRI scans. After attempting nonsurgical treatment including physical therapy and epidural injections, she continued to experience persistent pain. What is the most appropriate treatment?

 

 

 

 

 

 

  1. Open biopsy

  2. Incision and debridement

  3. Decompression

  4. Decompression and instrumented fusion

  5. Decompression and uninstrumented fusion

 

PREFERRED RESPONSE:4

 

 

 

Question 2-Figure is the T2-weighted sagittal MRI scan of a 46-year-old man with low-back pain. He underwent provocative discography with L4-5 as an internal control. Compared to people not undergoing this test, he may be at an increased risk for developing

 

  1. a malignancy.

  2. disk herniation.

  3. compression fracture.

  4. benign marrow changes.

  5. vertebral osteomyelitis.

 

PREFERRED RESPONSE:2

 

Question 3-Figures a through c are the lateral plain radiograph and postcontrast sagittal and axial T1-weighted MRI scans of a 20-year-old man with worsening back pain for 3 weeks. He notes malaise, nausea, and emesis but denies fevers. Findings from his neurologic examination are normal. Laboratory studies show a white blood cell count of 9,200/mm3, an erythrocyte sedimentation rate of 65 mm/h (reference range,0-20 mm/h), and C-reactive protein of 9.5 mg/L (reference range, 0-3.0 mg/L). A guided biopsy was performed and revealed methicillin-sensitive Staphylococcus aureus. In addition to parenteral antibiotics

,what is the most appropriate treatment?

 

 

  1. Observation

  2. Anterior lumbar debridement

  3. Anterior lumbar debridement and fusion

  4. Posterior lumbar debridement

  5. Posterior lumbar debridement and fusion

PREFERRED RESPONSE:1

 

Question 4-Figures a and b are the sagittal and axial T1-weighted MRI scans with contrast of a 42-year-old man with severe low-back pain. He underwent posterior lumbar decompression surgery 1 month ago for spinal stenosis and was doing well until 3 days ago. What is the most appropriate treatment?

 

 

 

 

  1. Epidural steroid injection

  2. Broad-spectrum oral antibiotics

  3. Physical therapy and oral steroids

  4. Surgical irrigation and debridement

  5. Nonsteroidal anti-inflammatory medications

PREFERRED RESPONSE:4

 

Question 5-According to the National Acute Spinal Cord Injury Study III, following an acute spinal cord injury,methylprednisolone should be administered for

 

  1. 24 hours if started within 3 hours.

  2. 36 hours if started within 3 hours.

  3. 40 hours if started within 8 hours.

  4. 72 hours if started within 8 hours.

  5. 12 hours if started within 24 hours.

 

PREFERRED RESPONSE:1

 

Question 6-A 76-year-old woman has neck pain after falling down a flight of stairs. Figures 65a and 65b show a lateral radiograph and sagittal CT scan of her cervical spine. Which factor is an absolute contraindication for the placement of C1-C2 transarticular screws?

 

 

 

 

  1. Osteoporosis

  2. Aberrant vertebral artery

  3. Previous C2 laminectomy

  4. Concomitant C1 ring fracture

  5. Disruption of the transverse ligament

 

PREFERRED RESPONSE:2

 

Question 7-Which complication most likely results in poor clinical outcomes following adult spinal deformity surgery?

 

  1. Wound seroma

  2. Decubitus ulcer

  3. Cerebrospinal fluid leak

  4. Venous thromboembolic event

  5. Excessive intraoperative bleeding

 

PREFERRED RESPONSE:4

 

Question 8-Figures a and b are the MRI scans of the cervical spine without contrast of a 38-year-old man with neck pain radiating into the right upper extremity for the past 4 weeks. He denies numbness or weakness.Examination was significant for reproduction of pain going down the right arm with neck extension and right lateral rotation. What is the next treatment step?

 

 

 

  1. Physical therapy

  2. Epidural steroid injection

  3. High-dose intravenous steroid

  4. Posterior cervical foraminotomy

  5. Anterior cervical discectomy and fusion

PREFERRED RESPONSE:1

 

 

 

Question 9-Figures a and b are the postoperative anteroposterior and lateral radiographs of a 76-year-old woman with low-back pain following a ground-level fall. Compared to a sham procedure, vertebroplasty is most likely to result in

 

  1. improved pain relief.

  2. improved quality of life.

  3. improved functional disability.

  4. no difference in patient-reported outcomes.

  5. a higher incidence of subsequent vertebral compression fractures (VCFs).

 

Question 10-Figures a through c are the coronal and sagittal CT scans and upright lateral radiograph of the cervical spine of a 67-year-old man with severe neck pain after a fall. He denies numbness or weakness in his arms and legs. He is awake, alert, and oriented and findings from his neurologic examination are normal. His medical history was noted for osteoporosis and long-term tobacco use. What is the most definitive treatment?

 

 

 

  1. Cervical collar

  2. Cranial traction

  3. Halo-vest orthosis

  4. Posterior C1-C2 fusion

  5. Posterior C1-C3 fusion

PREFERRED RESPONSE:3

 

Question 11-A 76-year-old man with a long history of tobacco use had an acute onset of thoracic back pain with progressive numbness and weakness in his lower extremities.

Examination reveals loss of sensation below the level of the nipples with 3/5 strength in his legs. Figures a and b show the sagittal and axial T2-weighted MRI scans of his thoracic spine. He also has multiple lesions in his chest, liver, and right humerus. Following a biopsy, which treatment is most definitive?

 

 

 

  1. Chemotherapy

  2. Radiation therapy

  3. Palliative measures

  4. Intravenous steroids

  5. Surgical decompression and fusion

 

Question 12-The use of a soft cervical orthosis is most supported for which injury?

  1. Whiplash

  2. C4 burst fracture

  3. Rotatory subluxation at C1-C2

  4. Displaced type II odontoid fracture

  5. Ligamentous injury with translation of C4 on C5

 

PREFERRED RESPONSE:1

 

 

 

Question 13-An 82-year-old woman has had acute low-back pain for 2 weeks and reports no falls or back trauma.Her past medical history is negative and she takes no medications. She has tenderness to palpation over the lumbar spine and a normal neurologic examination. A lateral radiograph from 2 years ago is seen in Figure a and a current lateral radiograph is seen in Figure 155b. In addition to bracing, what is the next step in treatment?

 

  1. Open biopsy

  2. Vertebroplasty

  3. Epidural steroid injection

  4. Evaluation for osteoporosis

  5. Decompression and instrumented fusion

 

PREFERRED RESPONSE:4

Question 14-Figures a and b are the sagittal and axial T2-weighted MRI scans of a 62-year-old man who has low-back pain with numbness radiating into his left calf. The pain has been refractory to nonsurgical treatments. Which procedure is associated with the lowest risk for persistent back pain and recurrence of the lesion?

 

 

  1. Laminectomy

  2. Hemilaminectomy

  3. Percutaneous aspiration

  4. Facetectomy with arthrodesis

  5. Transforaminal epidural injection PREFERRED RESPONSE:4

 

Question 15-A 67-year-old woman has difficulty walking distances and occasional numbness in her feet. Her medical history is significant for type 2 diabetes mellitus. Examination reveals full strength and sensation in her bilateral lower extremities. She has difficulty with tandem gait and hyperreflexia and has a positive Hoffman’s sign. The lumbar spine MRI scan was significant for severe L4-L5 spinal stenosis. What is the most appropriate next step?

  1. Physical therapy

  2. Lumbar decompression

  3. Lumbar epidural steroid injection

  4. MRI scan of the cervical spine

  5. Laboratory testing of hemoglobin A1C level

PREFERRED RESPONSE:4

 

Question 16-What type of spinal cord injury exhibits the highest potential for neurologic improvement?

  1. Conus medullaris

  2. Complete thoracic (T4-T9)

  3. Incomplete thoracic (T4-T9)

  4. Complete thoracolumbar (T10-T12)

  5. Incomplete thoracolumbar (T10-T12)

PREFERRED RESPONSE:1

Question 17-Figures a and b are the sagittal and axial T2-weighted MRI scans of a 33-year-old woman who has had pain, numbness, and weakness radiating into her right arm for 2 weeks. What are the most likely findings on examination?

 

 

 

  1. Numbness in the index finger, deltoid weakness, hypoactive biceps reflex

  2. Numbness along the lateral shoulder, deltoid weakness, hypoactive biceps reflex

  3. Numbness along the lateral shoulder, triceps weakness, hypoactive biceps reflex

  4. Numbness in the middle finger, biceps weakness, hypoactive triceps reflex

  5. Numbness in the middle finger, wrist flexor weakness, hypoactive brachioradialis reflex

PREFERRED RESPONSE:2

 

Question 18-Figures a and b are the T2-weighted MRI scans of a 28-year-old woman with low-back pain radiating down the right lower extremity for 3 months. Compared to nonsurgical treatment, surgery most likely will result in

 

 

  1. no difference in leg pain.

  2. significantly worse disability.

  3. significantly worse back pain.

  4. significantly better work status.

  5. significantly better physical function.

PREFERRED RESPONSE:5

 

Question 19-Figures a and b are the T2-weighted MRI scans of a 37-year-old left-hand-dominant man with a 3-month history of neck pain radiating down the back of his left arm and into his left hand. He also noted difficulty with buttoning his shirt. Examination reveals full strength and sensation in all extremities. He has radiating pain in all extremities with neck extension and flexion. He also has hyperactive reflexes and difficulty with tandem gait. What is the most appropriate treatment?

 

 

 

  1. Physical therapy

  2. Transforaminal epidural injections

  3. Anterior discectomy and fusion at C6-7

  4. Nonsteroidal anti-inflammatory medications

  5. Posterior decompression and fusion from C3-T1

PREFERRED RESPONSE:3

 

 

 

Question 20-Figure shows the axial CT scan of a 33-year-old man with severe neck pain after a motor vehicle collision. Which structure is most important in guiding treatment?

 

  1. Vertebral artery

  2. Alar ligament

  3. Apical ligament

  4. Transverse ligament

  5. Posterior ligamentous complex

 

PREFERRED RESPONSE:4

 

Question 21-A 16-year-old girl was seen after a motor vehicle collision. Imaging studies including plain radiographs,MRI scans, and CT scans confirm bilateral jumped facets at C5-6 without disk herniation. She is alert,oriented, and neurologically intact. What is the most appropriate next step?

 

  1. Awake closed reduction

  2. Application of a halo orthosis

  3. Placement of a cervical collar

  4. Open reduction under anesthesia

  5. Closed reduction under anesthesia

 

PREFERRED RESPONSE:1

 

 

 

 

Question 1 -A 20-year-old college basketball player has lateral ankle pain after sustaining an

ankle sprain. His pain persists despite allowing a sufficient period of rest and rehabilitation. He has a history of multiple previous sprains, and describes this pain as being different than his usual pain after a sprain. He has tenderness to palpation along the posterior fibula and reproducible pain with resisted eversion. What is the most appropriate treatment option?

 

  1. Core repair and tubularization of the peroneus brevis tendon

  2. Direct repair of the anterior talofibular and calcaneofibular ligaments

  3. Ankle arthroscopy and debridement of the lateral gutter and tibiofibular joint

  4. Lateral ankle stabilization with a transfer of the peroneus brevis through the fibula

  5. Continued physical therapy with proprioceptive training and peroneal strengthening

 

PREFERRED RESPONSE: 1

 

Question2 -During preparticipation physicals for college football, an athlete tests positive for the sickle-cell trait.With regard to clearance to play, his team physician should

 

  1. counsel the athlete about his personal risk for bone infarcts.

  2. recommend a prophylactic splenectomy prior to participation.

  3. bar the athlete from participating in National Collegiate Athletic Association-sanctioned events.

  4. assure the athlete that he can participate in football without concern.

  5. ensure that the athlete is given adequate recovery time and remains hydrated.

PREFERRED RESPONSE: 5

 

 

 

Question3-Figure 39 is the anteroposterior radiograph of a marathon runner who has left groin pain that prevents her from running. She recently got back into her usual running routine after an ankle injury preventedbher from running for several months. She now has pain with any weight bearing. What is the most appropriate treatment option?

 

  1. Hip resurfacing arthroplasty

  2. Hip arthroscopy with removal of the cam lesion

  3. Internal fixation of the femoral neck with multiple screws

  4. Trial of nonsurgical treatment with no weight bearing on the left leg

  5. Vitamin D level assessment and supplementation with 50000 units weekly

 

PREFERRED RESPONSE: 3

 

 

 

Question 4 -Denervation most typically associated with the finding seen in Figure 42 results in which characteristic finding?

 

  1. Internal rotation weakness

  2. External rotation weakness in adduction

  3. External rotation weakness in abduction

  4. No noticeable weakness of the shoulder

  5. Forward flexion weakness (more than 90 degrees)

 

PREFERRED RESPONSE: 2

 

Question 5 -A 36-year-old man who was playing recreational basketball felt a pop in the back of his leg and is now unable to walk. Rest, ice, and elevation have been ineffective at restoring his leg. Examination reveals pain over the posterior calf, some ecchymosis, and weak plantar flexion strength. A Thompson test result is positive. Compared with nonsurgical treatment, surgical treatment is more likely to

 

  1. carry a lower risk for equinus contracture.

  2. restore strength (closer to usual levels).

  3. reduce risk for rerupture.

  4. allow for quicker ambulation and recovery.

  5. result in better outcomes on blind randomized studies.

PREFERRED RESPONSE: 3

 

Question 6 -A 14-year-old girl has a 6-week history of diffuse pain in both knees after attending cheerleading camp without trauma. She denies mechanical symptoms or swelling, but does state her knees “give-way”and “click” occasionally. Examination and radiographs are unremarkable, with the exception of global discomfort to palpation of both knees. What is the most appropriate next step?

 

  1. MRI scans of both knees

  2. Corticosteroid injection into both knees

  3. Bone scan with pinhole views of both knees

  4. Lab studies to rule out a rheumatologic condition

  5. A physical therapy regimen to both lower extremities

 

PREFERRED RESPONSE: 5

 

Question 7 -A 20-year-old collegiate rower has pain along the left side of his chest just anterior to the midaxillary line. The pain began approximately 4 weeks after he started preseason training. The pain occurs almost immediately after he begins rowing and goes away when he stops. He has not noticed the pain while running. Which study will most likely reveal the diagnosis?

  1. Echocardiogram

  2. Electrocardiogram

  3. 3-phase bone scan

  4. Rib series radiographs

  5. Posteroanterior chest radiograph

PREFERRED RESPONSE: 3

 

Question 8 -A 42-year-old man has a chondral defect of the medial femoral condyle that is approximately 1 cm in diameter. He has a very athletic lifestyle, wishes to remain active, and is now seeking a third opinion. He has seen 2 orthopaedic surgeons; the first surgeon recommended microfracture for the chondral defect, and the other recommended an osteochondral autograft transplantation (OATS). What should the patient be told?

 

  1. Recovery is faster with microfracture, the outcomes are better with OATS, and both techniques produce the same reparative surface.

  2. Recovery is faster with OATS, the outcomes are better with microfracture, and both techniques produce the same reparative surface.

  3. The outcomes are better with OATS, rehabilitation is faster with OATS, and the reparative surface is articular cartilage with OATS.

  4. Recovery time and outcomes are similar between these 2 techniques, and the reparative surface with microfracture is fibrocartilage.

  5. Recovery and outcomes are similar between these 2 techniques, and the reparative tissue with OATS is fibrocartilage.

 

PREFERRED RESPONSE: 4

 

Question 9 -While performing an arthroscopic procedure, an instrument has a mechanical failure resulting in a 1.5-mm segment of metallic debris incarcerated within the soft tissue. After 45 minutes of fluoroscopic localization and special arthroscopic techniques, the fragment is determined to cause no harm to the patient. Upon recognizing the event, the treating surgeon should

  1. immediately abandon the procedure, close the portals, and obtain further imaging.

  2. complete the surgery, determine the risk for potential injury to the patient, and immediately notify the patient and family following the procedure.

  3. complete the surgery, follow the patient clinically for any unintended consequences before disclosing the event.

  4. call the patient’s family from the operating room to explain the error before proceeding to complete the procedure.

  5. refrain from formal disclosure if the fragment is unlikely to cause any further damage.

 

 

 

Question10-Video 125 shows a subscapularis repair viewing from the posterior portal looking anteriorly at the lateral border of the rotator interval. What serves as a landmark for appropriate placement of the anchor for the upper border repair?

 

  1. Biceps tendon

  2. Middle glenohumeral ligament

  3. Leading edge of the supraspinatus

  4. Posterior band of the inferior glenohumeral ligament

  5. Superior glenohumeral and coracohumeral ligaments

 

PREFERRED RESPONSE: 5

Question 11 -A 23-year-old woman sustained an ankle inversion injury 6 weeks ago. She reports pain and difficulty returning to recreational basketball. Examination reveals tenderness of the anterior talofibular ligament,pain laterally with inversion stress, weakness without pain in eversion against resistance, and a negative anterior drawer. What is the best next treatment step?

  1. Physical therapy

  2. Cast immobilization

  3. Peroneal tendon repair

  4. Chrisman-Snook reconstruction

  5. Modified Brostrum reconstruction

PREFERRED RESPONSE: 1

 

Question 12 -An elite-level pitcher with a history of chronic moderate medial elbow pain reports a sudden pop and severe pain along the medial elbow while throwing a pitch.

Examination reveals a positive moving valgus stress test. What is the most appropriate next treatment step?

  1. Rehabilitation of the flexor-pronator musculature

  2. Bracing of the elbow to facilitate a return to pitching

  3. Early primary repair of the ulnar collateral ligament

  4. Early ulnar collateral ligament reconstruction

  5. Early ulnar collateral ligament reconstruction and ulnar nerve transposition

PREFERRED RESPONSE: 4

 

Question13 -At her 6-week follow-up visit after arthroscopic repair of a full-thickness rotator cuff tear, a patient has not yet attended formal physical therapy. Which outcome at 1 year is expected?

  1. Persistent stiffness of the shoulder, resulting in loss of function

  2. High likelihood of needing a second procedure for a capsular release

  3. Improved healing of the rotator cuff despite persistent stiffness of the shoulder

  4. No long-term difference in motion compared to early physical therapy

  5. A significant decrease in Constant score compared to early physical therapy

 

Question 14 -Which of the following is the most important restraint to medial instability of the long head of the biceps tendon?

  1. Tendon of the subscapularis

  2. Coracohumeral ligament

  3. Superior glenohumeral ligament

  4. Morphology of the bicipital groove

  5. Origin of the long head of the biceps in the supraglenoid tubercle

 

PREFERRED RESPONSE: 1

Question15-An inferior placement of the posterior shoulder portal endangers which vital structure?

  1. Radial nerve

  2. Axillary nerve

  3. Long thoracic nerve

  4. Thoracoacromial artery

  5. Anterior humeral circumflex artery

PREFERRED RESPONSE: 2

Question 16-Toward the end of a preseason football practice, a player approaches his trainer with difficulty remembering what he is supposed to do during his position drills. He is confused and disoriented, clearly fatigued, soaked in sweat, and his skin is pale. What is the most appropriate next step?

  1. Have the athlete lie down on the sidelines for administration of intravenous fluid.

  2. Immediately perform a thorough neurologic evaluation on the sidelines.

  3. Assure the athlete that he is simply dehydrated and can return after rehydrating.

  4. Administer a Sideline Assessment of Concussion test to determine return to play.

  5. Obtain a core temperature in a cooled training room while hydrating the athlete.

 

PREFERRED RESPONSE: 5

Question 17-A 29-year-old athlete reports recurrent anterior shoulder instability after surgery. Performing an arthroscopic revision surgery is contraindicated when there is capsular attenuation or

  1. glenoid bone loss of 15%.

  2. an associated rotator cuff tear.

  3. postthermal capsular necrosis.

  4. a Hill-Sachs lesion involving 20% of the humeral head.

  5. the patient participates in sports that involve contact and collision.

PREFERRED RESPONSE: 3

Question 18-During routine knee arthroscopy, the anterior cruciate ligament is visualized with the knee in 95 degrees of flexion. At this angle of knee flexion, the

  1. posteromedial bundle is loose and the anterolateral bundle is tight.

  2. posterolateral bundle is loose and the anteromedial bundle is tight.

  3. anteromedial bundle is loose and the posterolateral bundle is tight.

  4. anterolateral bundle is loose and the posteromedial bundle is tight.

  5. anterolateral bundle is tight and the posteromedial bundle is tight.

 

Question 19-What is an absolute contraindication to meniscal transplantation?

  1. Stable joint

  2. Angular deformity

  3. Inflammatory arthritis

  4. Localized chondral defect

  5. Anterior cruciate ligament deficiency

PREFERRED RESPONSE: 3

 

Question 20-A 29-year-old athlete with postmeniscectomy pain syndrome after prior arthroscopic meniscectomy is referred for a meniscal allograft. What is the most likely longterm outcome for a meniscal allograft transplantation?

  1. Rejection with early failure

  2. Cartilage regeneration

  3. Relative acellularity and possible tearing

  4. Permanent reduction in pain and swelling

  5. Acceleration in the progression of osteoarthritis

PREFERRED RESPONSE: 3

 

Question21-Figures 275a through 275c are the radiographs of a 28-year-old recreational basketball player who underwent autograft anterior cruciate ligament reconstruction and a partial medial meniscectomy 4 years ago. Although his initial results were favorable, he has persistent instability symptoms and “giving way”when attempting to participate in desired sports activities. Examination reveals the following: a 2ALachman, 3+ pivot shift, negative external rotation dial, and a positive McMurray maneuver for the medial compartment. His recurrent instability symptoms are most likely related to

 

 

  1. an unstable lateral meniscal tear.

  2. the development of posterolateral instability.

  3. femoral tunnel placement that did not restore rotatory stability.

  4. femoral tunnel placement that did not restore the posteromedial bundle.

  5. femoral tunnel fixation that did not adequately address the anterolateral bundle.

 

Question 1- Arthroscopic remplissage of the shoulder is most indicated for patients with

  1. irreparable subscapularis tear.

  2. irreparable supraspinatus tendon tear.

  3. large (>25%) glenoid rim deficiency.

  4. large (>25%) humeral head deficiency.

  5. glenohumeral internal rotation deficit.

PREFERRED RESPONSE:4

Question 2-Figures a through c are the MRI scans of a 21-year-old woman with recurrent shoulder instability and pain after an open anterior stabilization procedure. Positive belly-press test findings were positive.At surgery she was found to have an irreparable tear of the tendon injury identified preoperatively. The procedure to address the dynamic stabilizer deficit places which nerve at most risk?

 

 

  1. Ulnar

  2. Radial

  3. Axillary

  4. Median

  5. Musculocutaneous

PREFERRED RESPONSE:5

 

 

 

Question 3- A 21-year-old right-hand-dominant competitive swimmer has had right shoulder pain for approximately 9 months. He denies any specific injury. The pain is localized to the posterior aspect of his shoulder.Examination reveals well-preserved glenohumeral motion and negative impingement signs. Plain radiographs are unremarkable. There is no evidence of a rotator cuff tear on the MRI scan. Based on the findings seen in Figure which muscle belly most likely has atrophy?

 

  1. Biceps

  2. Teres minor

  3. Teres major

  4. Infraspinatus

  5. Subscapularis

 

PREFERRED RESPONSE:4

 

Question 4- Figures a and b are the radiographs of a 15-year-old boy who felt a ‘pop’ in his hip while running.The injury occurred as a result of a forceful contraction of the

 

 

  1. sartorius.

  2. rectus femoris.

  3. gluteus medius.

  4. vastus lateralis.

  5. vastus intermedius.

PREFERRED RESPONSE:2

Question 5-What is the occurrence ratio of noncontact anterior cruciate ligament (ACL) injuries among men and women (men:women)?

 

  1. 1:1 ACL injuries occur more commonly in women by a ratio of 9:1, are frequently contactinjuries, and are related primarily to the cyclical effects of sex hormones on ligament tensile strength.

  2. 4.5:1 ACL injuries occur more commonly in men by ratio of 1.5:1, are frequently contact injuries, and may be related to neuromuscular characteristics of jumping and landing.

  3. 1:4.5. ACL injuries occur more commonly in women by a ratio of 4.5:1 and are frequently noncontact injuries; a genotype within the COL5A1 gene is associated with reduced risk for ACL ruptures in women.

  4. 2:1 ACL injuries occur more commonly in men by a ratio of 2.5:1 and are frequently noncontact injuries; a genotype within the COL5A1 gene is associated with increased risk for ACL ruptures in men.

  5. 1:2. ACL injuries occur more commonly in women by a ratio of 2:1, are frequently noncontact injuries, and may be related to neuromuscular characteristics of jumping and landing.

PREFERRED RESPONSE:3

Question 6- The femoral origin of the medial patellofemoral ligament is located between what two anatomic landmarks?

  1. Medial epicondyle and adductor tubercle

  2. Medial epicondyle and gastrocnemius tubercle

  3. Adductor tubercle and gastrocnemius tubercle

  4. Adductor tubercle and medial collateral ligament

  5. Medial epicondyle and semimembranosus tibial insertion

 

PREFERRED RESPONSE:1

Question 7- What location is the primary source of vascular ingrowth for tendon-bone healing with rotator cuff repair?

 

  1. Intra-articular bleeding

  2. Intratendinous layer of the rotator cuff

  3. Articular surface of the rotator cuff

  4. Holes in the greater tuberosity

  5. Bleeding from the decompressed acromion

PREFERRED RESPONSE:4

 

 

 

Question 8-Figure 94 is a sagittal knee MRI scan of an injured 25-year-old man. What is the most likely diagnosis?

 

  1. Osteochondritis dissecans

  2. Lateral patella dislocation

  3. Discoid lateral meniscus tear

  4. Anterior cruciate ligament tear

  5. Bucket-handle lateral meniscus tear

 

PREFERRED RESPONSE:4

 

Question 9-A 14-year-old girl reports a 6-week history of diffuse pain in both knees after attending cheerleading camp. There was no obvious traumatic event. She denies any symptoms of locking or significant swelling, but states her knees ‘give-way’ and ‘click’ occasionally. She has no other joint problems and denies any history of similar symptoms. Examination is unremarkable with the exception of global discomfort to palpation of both knees. Radiographs also are unremarkable. What is the most appropriate next treatment step?

 

  1. MRI scan of both knees

  2. Corticosteroid injection into both knees

  3. Bone scan with pinhole views of both knees

  4. Bloodwork to rule-out a rheumatologic condition

  5. Physical therapy regimen to both lower extremities

 

PREFERRED RESPONSE:5

 

Question 10-What leads to muscle hypertrophy?

 

  1. Neural recruitment

  2. Active stretching

  3. Passive stretching

  4. Proprioceptive training

  5. Progressive overloading

PREFERRED RESPONSE:5

 

 

Question 11-The femoral insertion of the superficial medial collateral ligament is represented by which letter on Figure?

 

  1. A

  2. B

  3. C

  4. D

  5. E

 

PREFERRED RESPONSE:4

 

Question 12-A 23-year-old woman sustained an ankle inversion injury 1 week ago. She reports pain and difficulty returning to recreational basketball. Examination reveals tenderness of the anterior talofibular ligament,pain laterally with inversion stress, weakness without pain in eversion against resistance, and a negative

anterior drawer. What is the next treatment step?

 

  1. Physical therapy

  2. Cast immobilization

  3. Peroneal tendon repair

  4. Chrisman-Snook reconstruction

  5. Modified Broström reconstruction

PREFERRED RESPONSE:1

Question 13-A patient had pain and mild swelling 6 to 8 weeks following arthroscopic rotator cuff repair. Laboratory studies showed an erythrocyte sedimentation rate of 30 mm/h (reference range, 0-20 mm/h) and a C-reactive protein level of 1.8 mg/L (reference range 0.08-3.1 mg/L). Aspiration fluid from the subacromial space revealed a negative gram stain.

How long must bacterial cultures be held?

  1. 48 hours

  2. 72 hours

  3. 96 hours

  4. 10 days

  5. 21 days

 

Question 14-What is the most common associated finding of overuse syndromes for the patellofemoral joint in children or adolescents?

  1. Muscle imbalance

  2. Bipartite patella

  3. Ligamentous laxity

  4. Hypoplastic trochlea

  5. Recurrent patellar dislocation

PREFERRED RESPONSE:1

Question 15-A 13-year-old boy has had medial-sided elbow pain while pitching for 3 weeks. The pain started after several innings and occurred during the late cocking and acceleration phase. Examination reveals elbow range of motion is full and symmetric. He is tender over the medial epicondyle to palpation and during valgus stress testing. A dynamic ultrasound of his elbow shows no medial widening with valgus stress.What is the recommended treatment?

 

  1. Ulnar nerve transposition

  2. Cessation of pitching for 4 weeks

  3. Medial epicondylitis debridement

  4. Medial collateral ligament repair

  5. Medial collateral ligament reconstruction

PREFERRED RESPONSE:2

 

Question 16-A 21-year-old Division I collegiate football player sustained the injury shown in Figures a throughc. Which of the following interventions is the optimal treatment for return to play?

 

 

 

  1. Injection of platelet-rich plasma

  2. Surgical treatment with cannulated screw fixation

  3. Open reduction and internal fixation with modular locking plate

  4. Short-leg casting for 6 weeks followed by conversion to a walker boot and return to play

  5. Short-leg casting for 6 weeks followed by conversion to a total-contact insert with a carbon fiber footplate

 

Question 17- Excessive resection of a posterior olecranon osteophyte in an overhead-throwing athlete with medial ulnar collateral insufficiency may result in

  1. loss of flexion.

  2. loss of extension.

  3. varus instability.

  4. valgus instability.

  5. excessive lateral ulnar collateral ligament strain.

PREFERRED RESPONSE:4

 

Question 18-The clinical diagnosis of an injury to the posterior branch of the axillary nerve is best determined with assessment of sensation over the

  1. lateral deltoid and the Jobe test.

  2. lateral deltoid and the Hornblower’s test.

  3. anterior deltoid and the lift-off test.

  4. anterior deltoid and the abdominal compression test.

  5. posterior deltoid and the active compression test.

PREFERRED RESPONSE:2

 

Question 19-Figures a and b are the plain radiographs of a 14-year-old healthy, active boy who sustained an anterior cruciate ligament injury while playing football. Which of the following reconstruction techniques is associated with the highest likelihood of growth disturbance?

 

 

  1. Iliotibial band tenodesis with over-the-top femoral fixation

  2. Tibialis anterior allograft reconstruction with cross-pin fixation

  3. Bone-patellar tendon-bone autograft with interference screw fixation

  4. Hamstring reconstruction with over-the-top staple fixation

  5. Hamstring reconstruction with suspensory femoral and tibial post-and-washer fixation

 

Question 20-The primary function of the posterior oblique ligament of the knee is to resist

 

  1. internal tibial rotation in full extension.

  2. external tibial rotation in full extension.

  3. external tibial rotation at 90 degrees of flexion.

  4. anterior tibial translation in full extension.

  5. posterior tibial translation at 90 degrees of knee flexion.

 

PREFERRED RESPONSE:1

 

Question 21-A 31-year-old right-hand-dominant man fell from a ladder and had persistent shoulder pain and a feeling that his shoulder was ‘popping out’ several times a day. Video a and the MRI scan seen in Figure b reflect the pertinent examination and diagnostic imaging findings. His symptoms persisted after 8 weeks of nonsurgical treatment with immobilization and physiotherapy. Which of the following interventions is the most appropriate next treatment step?

 

 

 

 

  1. Open rotator cuff repair

  2. Arthroscopic Bankart repair

  3. Arthroscopic rotator cuff repair

  4. Arthroscopic posterior labral repair

  5. Open anterior stabilization with capsular shift

 

 

 

 

 

Question1 -An 85-year-old woman with a history of poorly controlled hypertension, orally controlled diabetes, and atrial fibrillation with controlled rate is seen on a Thursday with an unstable intertrochanteric fracture.Evaluation reveals she is slightly hypernatremic (sodium level 155 mEq/L) (reference range, 136-142mEq/L). What is the most appropriate treatment option?

  1. Traction and hydration because surgical intervention puts this patient at high risk

  2. One liter of normal saline and immediate (Thursday) open reduction and internal fixation with a plate

  3. Rehydration, medical evaluation, and open reduction and internal fixation with a nail within 48 hours

  4. Rehydration, cardiac stress testing, endocrine evaluation, and open reduction and internal fixation with a plate on Monday

  5. Immediate open reduction and internal fixation with a nail followed by admission to medicine for treatment after surgery

PREFERRED RESPONSE: 3

 

Question 2 -What is the most important determinant of the energy imparted to the soft tissues as a result of a gunshot

wound?

  1. Yaw

  2. Mass

  3. Range

  4. Caliber

  5. Velocity

 

 

PREFERRED RESPONSE: 5

 

Question 3 -Figure 10 is the radiograph of an 18-year-old man who sustained an isolated gunshot wound to his right thigh. After appropriate evaluation and resuscitation, the fracture is repaired with a reamed intramedullary nail. What is the most commonly encountered complication in this scenario?

 

  1. Infection

  2. Malunion

  3. Nonunion

  4. Fat embolism

  5. Pulmonary embolism

 

PREFERRED RESPONSE: 2

 

Question4 -A 25-year-old thin man sustained a bimalleolar left ankle fracture, a comminuted spiral midshaft left humeral fracture, and a grade IV splenic laceration during a motor vehicle collision. His left radial nerve function is intact. He underwent splenectomy immediately and his fractures were splinted. In counseling the patient regarding surgical vs nonsurgical treatment of the humerus fracture, you would advise that

  1. the risk for radial nerve palsy is higher in spiral humeral shaft fractures that are treated nonsurgically.

  2. the patient may bear weight through the plated humeral fracture for the purpose of using ambulatory aids.

3. a functional fracture brace will not adequately maintain humeral shaft fracture alignment during the healing process.

  1. surgical fixation of the humeral fracture will allow for earlier fracture union than nonsurgical

    treatment with a functional fracture brace.

  2. long-term outcomes for plated humeral shaft fractures are better than for fractures treated nonsurgically.

PREFERRED RESPONSE: 2

 

Question5 -Figures 32a through 32c are the radiographs of a 31-year-old man who was involved in a motor vehicle collision. He has severe foot pain, marked swelling, and is unable to ambulate. What is the most appropriate definitive treatment step?

 

 

  1. External fixation

  2. Closed reduction and casting

  3. Closed reduction and percutaneous pinning

  4. Open reduction and internal fixation with rigid fixation of the first to fifth tarsometatarsal joints

  5. Open reduction and internal fixation with rigid fixation of the first to third tarsometatarsal joints and Kirschner wire fixation of the fourth and fifth tarsometatarsal joints

 

PREFERRED RESPONSE: 5

 

Question 6-Advantages of a locking plate implant over a 95-degree angled blade plate for fixation of supracondylar femur fractures include

  1. a higher union rate.

  2. a lower implant cost.

  3. a lower overall complication rate.

  4. a lower rate of prominent hardware requiring removal.

  5. improved ability to use with associated coronal fractures.

PREFERRED RESPONSE: 5

 

Question7 -A 55-year-old man has a draining wound at the end of his transfemoral amputation residual limb. He reports that he sustained a “compound fracture” of his thigh bone approximately 30 years ago, requiring amputation and rodding of a fracture near his hip. His wound drains intermittently and has done so since his amputation. Intermittent administration of oral antibiotics temporarily ceases wound drainage, but the drainage returns after antibiotics are stopped. Wound culture reveals Pseudomonas aeruginosa, which is sensitive to fluoroquinolones, carbapenems, aminoglycosides, and cephalosporins.

Radiographs of the residual limb are seen in Figures 63a and 63b. What is the recommended treatment?

 

 

  1. Administration of oral ciprofloxacin for 3 months

  2. Administration of oral ciprofloxacin for the rest of his life

  3. Surgical debridement and irrigation with implant removal and postsurgical ciprofloxacin for 3 months

  4. Surgical debridement and irrigation with implant removal, placement of a gentamicin-impregnated polymethylmethacrylate medullary rod, and postsurgical ciprofloxacin for 3 months

  5. Surgical debridement and irrigation with implant removal, sinus tract biopsy, placement of a gentamicin-impregnated polymethylmethacrylate medullary rod, and postsurgical iprofloxacin for 3 months

PREFERRED RESPONSE: 5

 

Question 8-Figures 73a through 73c are the current radiographs of a 35-year-old woman who fractured her ankle 3 years ago. Her course after surgery was complicated by wound dehiscence over her fibula plate. She had hardware removed and saucerization of her fibula at 9 months. She is now experiencing pain reproduced with dorsiflexion/plantar flexion that limits all of her daily living activities. She is unable to obtain a plantigrade foot with knee extension, has no pain with inversion/eversion, and has well healed wounds.Laboratory studies show that her erythrocyte sedimentation rate and C-reactive protein levels are within defined limits. What is the best treatment option?

 

 

  1. Total ankle replacement

  2. Tibiotalar arthrodesis

  3. Tibiotalar and subtalar arthrodesis

  4. Tibiotalar arthrodesis with gastrocnemius recession

  5. Tibiotalar and subtalar arthrodesis with gastrocnemius recession

 

 

 

Question 9 -The fracture shown in Figure 82 is scheduled to be fixed with a retrograde nail. An arthrotomy should be performed during the procedure because it

 

  1. ensures proper nail depth.

  2. provides control of the distal fragment.

  3. allows assessment for occult infection.

  4. allows protection of the polyethylene liner.

  5. allows assessment of the loosening component requiring revision.

 

PREFERRED RESPONSE: 4

 

 

 

Question 10 -A 52-year-old woman sustained a closed bimalleolar ankle fracture. She was treated with open reduction and internal fixation. A syndesmotic screw was added; however, there is persistent asymmetry of the ankle mortise as shown in Figures 87a and 87b. What is the most likely reason for this finding?

 

  1. The syndesmosis is malreduced.

  2. The lateral malleolus is malreduced.

  3. The posterior tibial tendon is entrapped in the medial joint.

  4. The deltoid ligament is interposed in the medial joint space.

  5. An osteochondral fragment is entrapped in the joint.

 

PREFERRED RESPONSE: 2

 

Question 11 -The World Health Organization Fracture Risk Assessment Tool (FRAX) calculates which fracture risk?

  1. 5-year risk for hip fracture

  2. 5-year risk for distal radius fracture

  3. 5-year risk for any fragility fracture

  4. 10-year risk for hip fracture

  5. 10-year risk for distal radius fracture

PREFERRED RESPONSE: 4

 

 

 

Question 12 -Figure 102 is an intraoperative figure taken during fixation of a right lateral tibial plateau fracture luxation. Which structure is indicated by the arrow?

 

  1. Iliotibial band

  2. Popliteus tendon

  3. Medial meniscus

  4. Lateral meniscus

  5. Lateral collateral ligament

 

Question 13 -If a physician elects to shorten a femur by 4 cm for traumatic bone loss treatment and places an intramedullary nail for fixation, which deformity will be created in the lower extremity?

  1. Patella alta

  2. Medial mechanical axis deviation

  3. Lateral mechanical axis deviation

  4. Increased anatomic tibiofemoral angle

  5. Translation of the anatomical axis of the femur

PREFERRED RESPONSE: 2

 

 

 

Question 14 -The vessel ligated during the Stoppa approach for acetabular fracture fixation (Video 112) is an anastomosis of which structures?

 

  1. Femoral and obturator

  2. Internal iliac and obturator

  3. Internal and external obturator

  4. External iliac and obturator

  5. Superior gluteal and obturator

 

PREFERRED RESPONSE: 4

 

Question15 -The radiograph seen in Figure 117 reveals a submuscular plate placement with locking screws for fixation.The biomechanics of the construct can be best described as

 

 

 

  1. stiff and axially stable.

  2. stiff and axially unstable.

  3. flexible and axially stable.

  4. flexible and axially unstable.

  5. flexible and rotationally unstable.

 

Question16 -A 22-year-old man was an unrestrained driver who was ejected from his car during a rollover motor vehicle crash. He sustained a closed head injury, multiple closed right rib fractures with an ipsilateral pneumothorax, and an open midshaft right tibia fracture. The tibia wound measures approximately 3 mm in length and is free of gross contamination.

What is the most important factor shown to minimize risk for infection at the site of an open tibia fracture?

  1. Transfer to a Level I trauma center within 3 hours

  2. Intravenous antibiotic administration within 3 hours

  3. Irrigation and debridement of the open fracture wound within 6 hours

  4. Open reduction with plate-and-screw fixation at the index tip within 6 hours

  5. Tibia wound irrigation within 3 hours with a solution containing bacitracin

 

PREFERRED RESPONSE: 2

 

Question17 -A 68-year-old woman fell and sustained a displaced femoral neck fracture. She is a community ambulatory and enjoys playing tennis weekly. Which treatment will provide her with the best hip function?

  1. Hip resurfacing

  2. Hemiarthroplasty

  3. Total hip arthroplasty

  4. Internal fixation with cannulated screws

  5. Internal fixation with a sliding hip screw and an antirotation screw

 

PREFERRED RESPONSE: 3

 

Question18 -A 23-year-old man was tackled while playing football. He felt a “pop” in his knee and noted significant deformity. Examination reveals a closed posterior knee dislocation that is irreducible despite adequate sedation. He is unable to dorsiflex his toes or ankle. His ankle-brachial index is 0.6. What is the next most appropriate treatment step?

  1. Surgical intervention

  2. Splint and monitor peripheral pulse oximetry

  3. Magnetic resonance angiography

  4. Computed tomography angiography

  5. Standard angiography

PREFERRED RESPONSE: 1

 

Question 19 -A 24-year-old man sustained a medial tibial plateau fracture (Schatzker type IV) after being involved in a motor vehicle-pedestrian collision. What is the best next step?

  1. An MRI scan

  2. Ankle brachial index

  3. Immediate open reduction and internal fixation

  4. Closed reduction and percutaneous screw fixation

  5. Definitive treatment with a hybrid external fixator

 

Question 20 -A 45-year-old woman sustained a fall from height and has the injury shown in Figures 135a and 135b.A 3-dimensional reconstruction CT scan is shown in Figure 135c. Joint-spanning external fixation is applied on the day of injury. Ten days later, her skin is acceptable for definitive fixation. What is the most appropriate type of fixation for her fracture?

 

 

  1. Percutaneous screws and cast

  2. Conversion to a circular fixator

  3. Medial and anterolateral locked plates

  4. Medial and anterolateral nonlocked plates

  5. Lateral locked plate and medial malleolus screws

PREFERRED RESPONSE: 4

 

 

Question 21 -The fracture seen in Figure 137 was most likely caused by what type of mechanism?

 

  1. Direct impact to the fibula

  2. Abduction of the foot relative to the tibia

  3. Adduction of the foot relative to the tibia

  4. Internal rotation of the foot relative to the tibia

  5. External rotation of the foot relative to the tibia PREFERRED RESPONSE: 2

Question22 -The best way to avoid sentinel event errors is through better

  1. training.

  2. staffing ratios.

  3. communication.

  4. patient assessment.

  5. availability of information.

 

PREFERRED RESPONSE: 3

 

Question23 -A 22-year-old man wants a second opinion 3 weeks after intramedullary nailing of a comminuted diaphyseal femoral shaft fracture. Examination reveals his injured leg has 26 degrees’ more external rotation than the contralateral limb and is 3 cm shorter based on a block measurement. He should be advised to

 

  1. let the fracture unite as is because there is sufficient hip rotation to accommodate the external rotation deformity, and a small shoe lift can accommodate for the limb length discrepancy.

  2. let the fracture unite, and if he later finds it bothersome, consider a corrective osteotomy of the injured femur for correction of the deformity.

  3. let the fracture unite because there is sufficient hip rotation to accommodate the external rotation deformity; if he later finds the leg length discrepancy bothersome, he should consider contralateral closed femoral shortening.

  4. consider revision surgery to correct the rotational deformity but not alter length because this may impair fracture union if performed at this time.

  5. consider revision surgery to correct both the rotational deformity and leg length discrepancy.

 

PREFERRED RESPONSE: 5

Question24 -On an anteroposteriorly directed fluoroscopic radiograph, the appropriate entry point for an intramedullary tibia nail being used for fixation of a proximal third diaphyseal tibial fracture is ideally positioned

  1. centered between the medial and lateral tibial eminences.

  2. in line with the lateral border of the lateral tibial eminence.

  3. in line with the medial border of the lateral tibial eminence.

  4. in line with the lateral border of the medial tibial eminence.

  5. in line with the medial border of the medial tibial eminence.

PREFERRED RESPONSE: 3

Question25 -Figures 156a and 156b are the radiographs of a 38-year-old man with diabetes mellitus who fell 8 feet from a ladder and sustained an isolated closed injury of his leg.

 

 

Examination revealed swollen but soft compartments. His neurovascular examination was unremarkable. A damage-control fixator was initially applied, and his soft-tissue envelope is now amenable to further intervention. What is the most appropriate treatment?

  1. Conversion to a peri-articular hybrid frame

  2. Open reduction and internal fixation with a lateral locking plate

  3. Open reduction and internal fixation with a lateral nonlocking plate

  4. Open reduction and internal fixation with medial and lateral plates

  5. Open reduction and internal fixation with posteromedial and lateral plates

 

PREFERRED RESPONSE: 5

 

 

 

Question 26 -Figure 160 is the intrasurgical photo of a 35-year-old woman with an open tibial fracture. Examination reveals no Doppler signal of the peroneal artery or anterior tibial artery. However, flow in her posterior tibial artery is detected by Doppler. According to the Gustilo-Anderson classification system, the fracture

should be classified as

 

  1. type I.

  2. type II.

  3. type IIIA.

  4. type IIIB.

  5. type IIIC.

 

PREFERRED RESPONSE: 4

 

Question 27 -To minimize complications and to maximize the likelihood of successful outcomes after percutaneous fixation of displaced extension-type supracondylar humeral fractures in children, the physician should

 

  1. use a divergent wire technique with wires placed medially.

  2. use a divergent wire technique with wires placed laterally.

  3. use a crossed-wire technique with wires placed laterally and medially.

  4. apply a postsurgical circumferential cast with the elbow fully extended to prevent postsurgical displacement.

  5. apply a postsurgical circumferential cast with the elbow flexed past 90 degrees to prevent postsurgical displacement.

 

PREFERRED RESPONSE: 2

 

Question28 -A 24-year-old football player sustained an injury to his left foot when another player fell directly on his heel. He is unable to bear weight, but radiograph findings were negative. He is exquisitely tender at the midfoot. What is the best next diagnostic study?

 

  1. A CT scan

  2. A bone scan

  3. Weight-bearing views

  4. Contralateral foot radiographs

  5. Repeat radiograph in 2 weeks

PREFERRED RESPONSE: 3

 

Question29 -What is the most common nerve injury seen in Figures 172a and 172b?

 

 

 

  1. Ulnar

  2. Radial

  3. Median

  4. Anterior interosseous

  5. Lateral antebrachial cutaneous

 

PREFERRED RESPONSE: 4

 

Question 30 -The risk for developing complex regional pain syndrome after surgery to the foot and ankle or the wrist can be decreased through the use of

 

  1. capsaicin.

  2. vitamin C.

  3. vitamin D and calcium.

  4. dexamethasone block.

  5. multimodal pain therapy.

PREFERRED RESPONSE: 2

 

Question31 -What is the most common complication seen after patellar fracture open reduction and internal fixation?

 

  1. Loss of reduction

  2. Knee extensor lag

  3. Symptomatic implants

  4. Flexion contracture exceeding 5 degrees

  5. Extension contracture exceeding 15 degrees

PREFERRED RESPONSE: 3

 

Question 32 -An athletic 30-year-old sustained multiple injuries in a high-speed motor vehicle collision that resulted in a loss of approximately 30% of blood volume. On arrival to the emergency department, the heart rate is100 and blood pressure is 104/62. The best means with which to evaluate true hemodynamic status is

 

  1. hematocrit.

  2. serial heart rate.

  3. serial blood pressure with a manual cuff.

  4. serial blood pressure with an arterial line.

  5. lactate and base deficit levels.

PREFERRED RESPONSE: 5

 

 

 

Question 33 -Which virtual hinge shown in Figure 196 will gain the most length with the least amount of translation and angulation at the end of deformity correction?

 

  1. A

  2. B

  3. C

  4. D

  5. E

 

PREFERRED RESPONSE: 2

 

 

 

Question 34 -Figures 200a and 200b are the radiographs of an 82-year-old woman who fell on a flexed knee. She has no other injuries and was able to ambulate without assistance before her fall. The recommended treatment to optimize her quality of life consists of

 

  1. external fixation.

  2. revision arthroplasty.

  3. open reduction and internal fixation.

  4. closed reduction and casting.

  5. closed reduction and fracture bracing.

 

PREFERRED RESPONSE: 3

 

Question 35 -Figure 201a is the radiograph of a patient with an open femur fracture who had debridement and nailing with antibiotic beads as shown in Figure 201b. The patient notices leg deformity while lying in bed.Subsequent CT scans are shown in Figures 201c and 201d. In addition to being fixed short, what other malalignment, if any, is seen?

 

 

 

  1. Fixed with approximately 24 degrees’ internal rotation deformity

  2. Fixed with approximately 24 degrees’ external rotation deformity

  3. Fixed with approximately 31 degrees’ internal rotation deformity

  4. Fixed with approximately 31 degrees’ external rotation deformity

  5. No malalignment; deformity is attributable to postsurgical pain and reflex relaxation

PREFERRED RESPONSE: 1

Question36 -A 23-year-old man had a laparotomy and splenectomy with packing of the abdomen after a motorcycle collision. Laboratory studies show a hemoglobin level of 7.1 g/dL (reference range [rr], 14.0-17.5 g/dL) and a lactate level of 8.0 mmol/L (rr, 0.6-1.7 mmol/L). He also has a left humeral fracture,an anteroposterior compression I pelvic fracture, bilateral distal third femur fractures, and an open GustilotypeIIIA tibial diaphysis fracture with moderate contamination. What is the most appropriate treatment to administer before leaving the operating room?

  1. Saline lavage and splinting of the tibia and knee immobilizers of both femurs

  2. Betadine dressing and splinting of the tibia with unlocked retrograde nailing of both femurs

  3. Betadine dressing and external fixation of the tibia and knee immobilizers of both femurs

  4. Irrigation and debridement and external fixation of the tibia and external fixation of both femurs

  5. Irrigation and debridement and external fixation of the tibia and unlocked retrograde nailing ofboth femurs

 

PREFERRED RESPONSE: 4

 

 

Question 37 -Which nerve identified by the arrow seen in Figure 207 is encountered during fixation of a tibial pilon fracture?

 

  1. Sural

  2. Saphenous

  3. Lateral cutaneous

  4. Deep peroneal

  5. Superficial peroneal

 

PREFERRED RESPONSE: 5

 

Question 38 -What is the mechanism of action of tranexamic acid in controlling traumatic hemorrhage?

  1. Inhibition of vitamin K reductase

  2. Inhibition of topoisomerase II and IV

  3. Antithrombin-III selective inhibition of Factor Xa

  4. Competitive inhibition of plasminogen activation

  5. Stimulation of integrin-mediated platelet adhesion and activation

 

PREFERRED RESPONSE: 4

 

 

 

Question 39 -Figures 211a and 211b are the radiographs of a 41-year-old construction worker who sustained a twisting injury to his right leg. Which injury in the ipsilateral extremity is most commonly associated with this

type of fracture?

 

  1. Lisfranc injury

  2. Anterior cruciate ligament injury

  3. Proximal tibiofibular joint dislocation

  4. Tibial plateau fracture

  5. Posterior malleolus fracture

 

PREFERRED RESPONSE: 5

 

Question40 -A 25-year-old man sustained a closed right knee dislocation in a motor vehicle collision. His pedal pulses are symmetrical in the emergency department, both before and after reduction of the dislocation.Angiography can be avoided if

 

  1. his ipsilateral ankle-brachial index is 0.78.

  2. he had an absent ipsilateral pedal pulse in the field before arriving at the hospital.

  3. he has a slightly cool right foot that becomes warm again over the course of 3 hours.

  4. he has normal color and warmth of the right foot with normal pedal pulses for 48 hours.

  5. he has a large hematoma that has increased in size during the first 3 hours after admission.

 

PREFERRED RESPONSE: 4

 

Question 41 -A 24-year-old man had multisystem injuries, including an open left femoral shaft fracture he sustained after a motorcycle collision. He received 3 liters of crystalloid and 2 units of packed red blood cells.Urgent debridement and irrigation of his open left femur fracture is planned. Which finding would support proceeding with definitive fixation of the fracture at the time of debridement?

 

  1. Lactate level of 2.2 mg/dL

  2. Platelet count of 70,000

  3. Urine output of 20 cc/hour

  4. Systolic blood pressure of 90

  5. Body temperature of 34.5°C

 

PREFERRED RESPONSE: 1

 

Question 42 -The World Health Organization Safe Surgery Guidelines Checklist requires that when prophylactic antibiotics are indicated, they should be administered

 

  1. within 30 minutes prior to incision.

  2. within 60 minutes prior to incision.

  3. within 30 minutes prior to or after incision.

  4. within 60 minutes prior to or after incision.

  5. only in the operating room once the patient’s allergies, if any, have been confirmed by the anesthesiologist and circulating nurse.

 

PREFERRED RESPONSE: 2

 

Question 43 -A 55-year-old man sustained a right acetabular fracture after a fall from a ladder. Anteroposterior and Judet radiographs of the pelvis are shown in Figures 226a through 226c, and an axial CT scan of the pelvis is shown in Figure 226d. The acetabular fracture is best classified as

 

 

  1. associated T type.

  2. associated both column.

  3. associated transverse and posterior wall.

  4. associated posterior column and posterior wall.

  5. associated anterior and posterior hemitransverse.

PREFERRED RESPONSE: 3

 

Question 44-A 78-year-old woman sustained a periprosthetic supercondylar femoral fracture. What is the advantage of submuscular plating compared with an extensile lateral approach?

  1. Decreased rate of infection

  2. Decreased risk for nonunion

  3. Decreased risk for iatrogenic fracture

  4. Improved functional outcome

  5. Increased longevity of the component

PREFERRED RESPONSE: 2

 

Question 45 -Figure 235 is the radiograph of a 75-year-old woman who is seen in the emergency department following a low-energy fall. What is the most appropriate treatment based on her radiographic findings?

 

  1. Perform a biopsy of the lesion

  2. Stabilize with an intramedullary nail

  3. Initiate immediate bisphosphonate therapy

  4. Treat with chemotherapy followed by wide resection

  5. Obtain a chest CT scan, urine protein electrophoresis, and serum protein electrophoresis

PREFERRED RESPONSE: 2

 

 

Question 46 -Figures 238a and 238b are the radiographs of a 60-year-old woman who fell and sustained a right midshaft humeral fracture 1 year ago. She was treated in a functional brace for 6 months and has used an electrical bone stimulator for the past 6 months. She has arm pain and limited use of her left shoulder and elbow.What is the best treatment option?

 

 

  1. A reamed intramedullary nail

  2. A change to an ultrasound bone stimulator

  3. Continued nonsurgical treatment with both functional bracing and electrical bone stimulator

  4. Systemic administration of 1-34 teriparatide

  5. Compression plating with or without bone graft

 

PREFERRED RESPONSE: 5

 

Question47 -A 75-year-old woman fell at home and sustained the injury seen in Figures 249a through 249c. What is the most appropriate treatment option?

 

 

 

  1. Stand pivot transfer only

  2. Bed rest with bathroom privileges

  3. Partial weight bearing on the right

  4. Weight bearing only after surgical intervention

  5. Bilateral weight bearing as tolerated

PREFERRED RESPONSE: 5

 

 

 

Question 48 -A 30-year-old man sustained the injury seen in Figure 261. According to the Lauge-Hansen Classification System, the fracture should be classified as

  1. pronation-abduction.

  2. pronation-adduction.

  3. pronation-external rotation.

  4. supination-adduction.

  5. supination-external rotation.

 

PREFERRED RESPONSE: 4

 

 

 

Question49 -The condition shown in Figure 268 has been subject to 2 nailing attempts. The patient is seen 8 months after the second surgery. What is the most appropriate treatment method?

 

  1. Bone stimulator with vitamin D supplementation

  2. In situ noncompressive plating with a bone graft

  3. In situ repeat intramedullary nailing with a bone graft

  4. Corrective alignment with exchange nailing with a bone graft

  5. Corrective alignment and compression plating without a bone graft

 

PREFERRED RESPONSE: 5

 

Question 1-A 42-year-old patient with a right distal radius fracture underwent open reduction and internal fixation. To reduce the likelihood of complex regional pain syndrome, the most appropriate medication is

 

  1. Biotin.

  2. Tramadol.

  3. vitamin A.

  4. vitamin C.

  5. vitamin E.

PREFERRED RESPONSE:4

 

 

 

Question 2-The tibial fracture shown in Figure 6 has been temporized with an external fixator; the plan is to convert to intramedullary nailing. During nailing, the best

way to avoid a valgus deformity would be to use which surgical tactic?

 

  1. Starting point at A, no blocking screw

  2. Starting point at A, blocking screw at C

  3. Starting point at A, blocking screw at D

  4. Starting point at B, blocking screw at C

  5. Starting point at B, blocking screw at D

 

PREFERRED RESPONSE:5

 

Question 3-Figures 13a through 13d are the stress radiographs of a 17-year-old female gymnast who injured her left foot while landing after a back hand spring. She had diffuse midfoot tenderness. Treatment should consist of

 

 

 

  1. cast immobilization.

  2. closed reduction and percutaneous pin fixation.

  3. a walker boot and early range of motion exercises.

  4. open reduction and primary first and second tarsometatarsal joint arthrodesis.

  5. open reduction and internal screw fixation of the first and secondtarsometatarsal.

 

PREFERRED RESPONSE:5

 

Question 4-What is the best measure to indicate that a multiply-injured patient has been adequately resuscitated to allow early total care of bilateral femoral shaft fractures?

  1. Heart rate

  2. Hematocrit

  3. Systolic blood pressure

  4. Diastolic blood pressure

  5. Lactate and base deficit

PREFERRED RESPONSE:5

 

Question 5-Stability for a bicondylar tibial plateau fracture with a posteromedial coronal fragment is best obtained by using

  1. a lateral locking plate only.

  2. an anteromedial locking plate only.

  3. a posteromedial locking plate only.

  4. posteromedial and lateral nonlocked plates.

  5. medial anterior-to-posterior lag screws and a lateral nonlocked plate.

PREFERRED RESPONSE:4

 

Question 6-A 25-year-old man was struck by a car and sustained an open tibial shaft fracture treated with medullary nailing. Preoperatively, he had a heart rate of 92 beats/min and a blood pressure of 144/72 mm Hg. He was awake and alert and had isolated right leg pain. His pain was not exacerbated with passive range of motion of the right ankle or toes, and his leg compartments were soft. At the conclusion of the surgical procedure his leg felt tense.

Anterior compartment pressure was noted to be 25 mm Hg. The patient’s heart rate was 52 beats/min and his blood pressure was 100/50 mm Hg. The appropriate treatment is to

 

  1. perform immediate 4-compartment fasciotomy without checking the other 3 compartments of the leg.

  2. measure the pressures within the other 3 compartments and perform selective fasciotomies for pressures above 72 mm Hg.

  3. measure the pressures within the other 3 compartments and perform selective fasciotomies for pressures above 42 mm Hg.

  4. measure the pressures within the other 3 compartments and perform selective fasciotomies for pressures above 20 mm Hg.

  5. not perform fasciotomy because the fracture is open and the compartments have been decompressed.

PREFERRED RESPONSE:3

 

Question 7-Figures 33a through 33c are the radiographs of a 35-year-old woman with a fracture treated with firstgeneration locked plating. She began weight bearing at 3 months and at 4½ months had knee pain that was worse with activity-related stair and sit-to-stand maneuvers. What is the most likely cause of her pain?

 

 

 

  1. Nonunion

  2. Malunion

  3. Hardware irritation

  4. Missed Hoffa fracture

  5. Modulus mismatch between hardware and bone

PREFERRED RESPONSE:3

 

Question 8-A 25-year-old man has a comminuted and displaced fracture of the inferior pole of the patella with disruption of the extensor mechanism. During counseling regarding treatment options, he should be informed that as the result of severe comminution,

 

  1. partial patellectomy with patellar tendon advancement is the only reliable option for this fracture.

  2. partial patellectomy with allograft reconstruction of the patellar tendon is the only reliable option for this fracture.

  3. partial patellectomy may be necessary, but open reduction with internal fixation, if possible, is associated with better outcomes.

  4. total patellectomy with extensor mechanism reconstruction is the only reliable option for this fracture.

  5. surgical therapy unlikely will be successful, and 12 weeks of knee immobilization in a cast is the treatment of choice.

 

PREFERRED RESPONSE:3

 

Question 9-Which nerve is most likely to exhibit the worst functional recovery after repair of a gunshot wound?

  1. Ulnar

  2. Tibial

  3. Radial

  4. Femoral

  5. Musculocutaneous

PREFERRED RESPONSE:1

 

Question 10-A 29-year-old man sustained a comminuted right femoral shaft fracture and ipsilateral displaced femoral neck fracture (Garden IV). Following adequate resuscitation and surgical clearance, the optimal treatment and recommended order of fracture fixation is

 

  1. open reduction and screw fixation of the femoral neck followed by a retrograde femoral nail on a radiolucent table.

  2. open reduction of the femoral neck and treatment with a sliding hip screw with a long side plate sufficient to bypass the shaft fracture.

  3. closed reduction of both fractures on a fracture table and placement of a cephalomedullary nail.

  4. closed reduction of both fractures and treatment with a sliding hip screw with a long side plate sufficient to bypass the shaft fracture.

  5. retrograde femoral nail on a radiolucent table followed by closed reduction and screw fixation of the femoral neck on a fracture table.

PREFERRED RESPONSE:1

Question 11-What is the most commonly observed characteristic of coronal plane fractures (Hoffa fractures) of the distal femur?

 

  1. Occurrence of bicondylar injuries

  2. Involvement of the lateral condyle

  3. Involvement of the medial condyle

  4. An association with closed fractures

  5. Easy identification by plain radiographs

PREFERRED RESPONSE:2

Question 12-An 11-year-old boy is brought to the emergency department with left knee pain after being involved in a motor vehicle collision. He has no other injuries. His left knee is edematous and tender and his leg is noted to be in valgus alignment. Neurovascular status of the limb is normal. Radiographs reveal a displaced physeal fracture of the distal femur with substantial proximal metaphyseal extension of thefracture laterally (Salter-Harris II). Which of the following treatment options is most appropriate?

  1. Closed reduction and long-leg cast application

  2. Closed reduction and 1½ spica cast application

  3. Closed reduction and spanning external fixation

  4. Open reduction and internal fixation with a lateral locked plate

  5. Open reduction and internal fixation with lag screws in the metaphysis

PREFERRED RESPONSE:5

 

Question 13-Which nerve is found traveling with the round ligament or spermatic cord through the superficial inguinal ring?

  1. Ilioinguinal

  2. Iliohypogastric

  3. Genitofemoral

  4. Lateral femoral cutaneous

  5. Terminal branch of the subcostal

PREFERRED RESPONSE:1

 

Question 14-A 34-year-old man was thrown from a motorcycle at a high rate of speed and sustained a grade 2 open humeral shaft fracture with associated radial nerve injury and an ipsilateral closed both-bone forearm fracture. What is the most likely cause of his radial nerve palsy?

  1. Contusion

  2. Neurapraxia

  3. Neurotmesis

  4. Axonotmesis

  5. Root avulsion

PREFERRED RESPONSE:3

 

Question 15-The condition shown in Figure 73 reveals a purely coronal deformity with a shortening of 2 cm more than that associated with varus deformity alone. There is no sagittal plane deformity. An osteotomy has just been performed below location B, which is the center of rotation axis (CORA). Locations A and C represent other elements of the correcting strategy. What should occur at each location to correct varus and axial lengthening?

 

 

 

  1. Distraction of A, rotation through C

  2. Compression of A, rotation through C

  3. Rotation through A, distraction through C

  4. Rotation through A, compression through C

  5. Distraction through A, distraction through C

 

PREFERRED RESPONSE:3

 

Question 16-Ten or more years after severe polytrauma, premenopausal women, compared to men

  1. need less psychological support.

  2. are less likely to feel well rehabilitated.

  3. have a shorter duration of rehabilitation.

  4. show no difference in quality-of-life scores.

  5. show higher rates of posttraumatic stress disorder and take more sick leave time.

PREFERRED RESPONSE:5

 

 

 

Question 17-Figures a and b are the radiographs of a 20-year-old man who injured his finger after being hit with a baseball. Treatment should include

 

 

  1. early motion as tolerated.

  2. surgical reduction and pinning.

  3. distal interphalangeal joint fusion.

  4. splinting of the distal interphalangeal joint in extension.

  5. splinting of the proximal interphalangeal joint in extension.

 

PREFERRED RESPONSE:2

 

Question1 8-A 26-year-old man involved in a motor vehicle collision had an isolated injury to his right foot consisting of an open talar neck fracture with extrusion of the talar body. The talar head remained reduced at the talonavicular joint and was split. The extruded talar body was placed in a plastic bag and brought in with the patient. Treatment of the injury complex should include irrigation, debridement, and

 

  1. open reduction and internal fixation of the talar head fracture and implantation of a prosthetic talar body.

  2. open reduction and internal fixation of the talar head fracture without reimplantation of the talar body.

  3. completion of talectomy by resection of the fractured talar head segment.

  4. completion of talectomy followed by immediate implantation of a prosthetic talus.

  5. reimplantation of the talar body and open reduction and internal fixation of the talus fractures.

 

Question 19-Figure a is the radiograph of a 16-year-old girl who was an unrestrained passenger in a motor vehicle collision. She sustained a right hip dislocation as an isolated injury. An uneventful closed reduction was performed as seen in Figure b. A postreduction CT scan is seen in Figure c. What is the best next treatment step?

 

 

 

  1. Open treatment within 24 hours

  2. Closed treatment with touchdown weight bearing on crutches

  3. Closed treatment with hip precautions and nonweight-bearing activity

  4. Examination under anesthesia to determine the need for fixation

  5. Skeletal traction, bed rest for 3 days, and then open treatment

PREFERRED RESPONSE:1

Question 20-Figures a and b are the radiographs of a patient seen for follow-up care after open reduction and internal fixation of an ankle fracture-dislocation. After reviewing the radiographs, the physician should recommend

 

 

  1. revision of the ankle syndesmosis fixation.

  2. removal of the syndesmosis screw at 6 weeks.

  3. removal of the syndesmosis screw at 12 weeks.

  4. insertion of a second syndesmosis screw proximal to the current fixation.

  5. continued nonweight-bearing activity until 12 weeks, then begin weight bearing without removal of the syndesmosis screw.

 

Question 21-One week after closed reduction of a distal radius fracture, an 11-year-old re-develops the initial deformity of 25 degrees apex volar angulation and 8 mm of dorsal displacement. What is the most appropriate treatment?

 

  1. Cast change and follow up in 3 weeks

  2. Removable splint and follow up in 3 weeks

  3. Open reduction and internal fixation with plates

  4. Closed reduction and percutaneous pinning

  5. Closed reduction and intramedullary fixation through the radial styloid

PREFERRED RESPONSE:4

 

Question 22-A 70-year-old woman sustains a lateral tibial plateau fracture when she is struck from the side while crossing the street. Compared to a 20-year-old man sustaining a lateral tibial plateau fracture by the same mechanism, she is less likely to have sustained a

  1. lateral meniscus tear.

  2. medial meniscus tear.

  3. lateral collateral ligament tear.

  4. medial collateral ligament tear.

  5. posterolateral corner injury.

PREFERRED RESPONSE:4

Question 23-Figure is the anteroposterior radiograph of a 56-year-old woman who was thrown from the back of a motorcycle at a high rate of speed. Advanced Trauma Life Support protocols are followed, but she remains hemodynamically unstable. Which of the following interventions is the best next treatment step?

 

 

  1. Emergent pelvic angiography for possible embolization

  2. Emergent trip to the operating room for pelvic stabilization

  3. Placement of a pelvic C-clamp in the emergency department

  4. More resuscitation and a search for another source of bleeding

  5. Application of a pelvic binder or circumferential pelvic antishock sheet

 

 

 

Question 24-A 28-year-old man sustained the injury seen in Figures 147a and 147b. During surgery, after radius fixation, the distal radioulnar joint was evaluated and instability was noted. The next step should be

 

  1. casting in pronation.

  2. casting in supination.

  3. early range of motion.

  4. ligament reconstruction using a tendon graft.

  5. percutaneous fixation of the distal radioulnar joint.

 

PREFERRED RESPONSE:5

 

Question 25-Figures 153a and 153b are the MRI scans of a 75-year-old man with a 30-year history of swelling and drainage through a thigh wound. He reported no fewer than 3 attempts at surgical treatment. The drainage persisted; brief periods of antibiotic use were the exception. The appearance of the resected skin around the sinus tract is shown in Figure 153c. In addition to surgical drainage and debridement, what would be the most appropriate best next step in treatment?

 

 

 

 

  1. CT scan of the chest

  2. Serum electrophoresis

  3. Resection of the involucrum

  4. Use of phenol gauze into the cavity after resection

  5. Histologic examination of the skin at the sinus tract

 

PREFERRED RESPONSE:5

 

Question 26-What is the main difference between a Gustilo-Anderson grade IIIA and IIIB tibial fracture?

  1. Degree of comminution

  2. Amount of contamination

  3. Presence of a vascular injury

  4. Need for free tissue transfer

  5. Length of the traumatic wound

PREFERRED RESPONSE:4

 

Question 27-Figures a and b are the radiographs of a 55-year-old man involved in an airplane crash who is seen in the emergency department with a closed injury to his left ankle. Trauma workup findings are negative for any other injuries. What is the appropriate next step?

 

 

 

  1. Closed reduction with cast application

  2. Closed reduction with orthotic application

  3. Placement of an ankle-spanning external fixation device

  4. Open reduction and internal fixation of both tibia and fibula the next day

  5. Calcaneal pin traction, elevation, and open reduction and internal fixation between days 3 and 5

PREFERRED RESPONSE:3

 

Question 28-An absolute contraindication for closed management of a humeral shaft fracture in a fracture brace is

  1. severe fracture comminution.

  2. associated brachial plexus injury.

  3. presence of radial nerve palsy from time of injury.

  4. development of radial nerve palsy following closed reduction.

  5. associated femoral shaft fracture treated with intramedullary nail.

 

PREFERRED RESPONSE:2

 

 

 

Question 29-Figures a and b are the plain radiographs of an 86-year-old woman with a 2-month history of hip pain that worsens with activity. Which condition is the most likely cause of her pain?

 

  1. Paget disease

  2. Stress fracture

  3. Femoral neck fracture

  4. Degenerative hip arthritis

  5. Metastatic disease to the pelvis

 

PREFERRED RESPONSE:2

 

Question 30-A patient was seen 3 months after undergoing reamed intramedullary nailing of a closed tibia fracture.Examination reveals motor strength of 5/5 throughout except for the extensor hallicus longus, which is 2/5. Sensation is intact to light touch throughout with the exception of the first web space. Review of the medical record does not reveal prior documentation of the abnormality. What is the most likely explanation for these findings?

 

  1. Transient peroneal nerve neurapraxia

  2. Missed lateral compartment syndrome

  3. Missed anterior compartment syndrome

  4. Laceration or injury to the peroneal nerve during placement of a distal medial-to-lateral interlocking screw

  5. Laceration or injury to the peroneal nerve during placement of a distal anterior-to-posterior interlocking screw

 

PREFERRED RESPONSE:1

 

Question 31-A 71-year-old patient with type II diabetes has a 2-month history of draining plantar ulceration at the forefoot. Which clinical finding is most indicative of underlying osteomyelitis?

 

  1. Probes to bone

  2. Erythema and warmth

  3. Fluctuance of fat pad

  4. Ascending lymphangitis

  5. Subcutaneous soft-tissue gas

 

PREFERRED RESPONSE:1

 

Question 32-A 32-year-old man was struck in the pelvis by a low-velocity gunshot. The risk for a subsequent infection would most likely be attributable to the

 

  1. caliber of the bullet.

  2. presence of a pelvic ring injury.

  3. penetration of the gastrointestinal tract.

  4. performance of a formal surgical debridement.

  5. passage of the bullet through the urinary bladder.

PREFERRED RESPONSE:4

 

Question 33-An 83-year-old right-hand-dominant patient reported a fall on an outstretched left arm with the injury noted in Figures 195a and 195b. A CT scan of the elbow revealed that the capitellar and trochlear fractures were highly comminuted and that the numerous articular fragments had very little bone. Optimal functional outcome can be obtained with

 

 

  1. the ‘bag of bones’ technique (ie, nonsurgical management with activity as tolerated).

  2. excision of the olecranon fracture fragment followed by total elbow arthroplasty.

  3. fixation of the olecranon fracture and total elbow arthroplasty.

  4. fixation of the olecranon fracture and excision of the capitellar and trochlear fracture fragments.

  5. open reduction and internal fixation of the distal humerus and olecranon fractures.

PREFERRED RESPONSE:3

 

Question 34-Figures a through c are the radiographs and CT scan of a 25-year-old man with a grade II open distal tibia-fibula fracture. The wound is directly medial with well-demarcated edges. There is no other lesion to the surrounding soft-tissue envelope and there is minimal additional swelling. Other comorbidities include a 1-pack-per-day smoking habit and posttraumatic stress disorder as the result of combat military service. He underwent surgical debridement and irrigation and was placed into a 2-pin external fixator. During the subsequent surgical procedure 48 hours later, the wound and bone were cleaned and closure was possible with mild tension. At this time, what is the most appropriate definitive osseous treatment?

 

 

 

  1. Hybrid external fixation

  2. Immediate ankle arthrodesis

  3. Resection of the distal tibia and initiation of the tibiotalar transport

  4. Open reduction and internal fixation of both the fibula and tibia

  5. Open reduction and internal fixation of the fibula and external fixation of the tibia

PREFERRED RESPONSE:5

 

Question 35-Figures a through c are the initial radiograph and CT scans of a 45-year-old man who fell from a roof. Two months later, he has persistent left shoulder pain, especially with shoulder range of motion.Treatment should consist of

 

 

 

  1. bone stimulator.

  2. percutaneous fixation.

  3. distal fragment excision.

  4. open repair of the nonunion.

  5. sling and early range of motion.

 

PREFERRED RESPONSE:4

 

Question 36-Figure is the MRI scan of a 42-year-old man who has had elbow pain on the right dominant side after lifting a boat trailer 3 days ago. He felt pain and a ‘pop’ in his antecubital fossa. Examination revealed abnormal hook test findings. Recommended treatment should consist of

 

 

  1. observation.

  2. physical therapy.

  3. dynamic bracing of the elbow.

  4. acute surgical repair of the distal biceps tendon.

  5. delayed surgical reconstruction of the distal biceps tendon.

 

PREFERRED RESPONSE:4

 

Question 37-Open reduction and internal fixation of a bicondylar right tibial plateau fracture is performed on a 42-yearold woman whose radiographs are seen in Figure a. An axial CT scan is seen in Figure b. Your surgical tactic will include

 

 

 

  1. a direct posterior approach to the proximal tibia and fixation with two posterior buttress plates.

  2. a posteromedial approach to the proximal tibia and locked buttress plate fixation posteromedially.

  3. a transfibular approach to the proximal tibia and fixation with a posterolaterally based locked construct.

  4. an anterolateral approach to the proximal tibia and fixation with a laterally based locked construct.

  5. anterolateral and posteromedial approaches to the proximal tibia and fixation with an anterolateral locking plate and a posteromedial nonlocking plate.

 

PREFERRED RESPONSE:1

Question 38-The structure(s) best shown by the obturator oblique view of the pelvis is (are)

 

  1. tear drop.

  2. anterior column and tear drop.

  3. anterior column and posterior wall.

  4. posterior column.

  5. posterior column and anterior wall.

 

PREFERRED RESPONSE:3

 

Question 39-Figures 214a through 214c are the clinical photograph and radiographs of a man with a crooked arm that was 1.5 cm shorter than the contralateral side and had 25 degrees of varus and 5 degrees of procurvatum.He is left-hand dominant and has full range of motion with no pain or strength deficits. He does not like the cosmetic appearance of his arm while wearing long-sleeved shirts and he questions the previous orthopaedic surgeon’s treatment. He should be told that he has an

 

 

  1. unhealed fracture and should have corrective surgery for the nonunion error.

  2. acceptable deformity and that the previous treatment was reasonable.

  3. unacceptable deformity and should have had surgical treatment.

  4. unacceptable alignment and should have corrective surgery for alignment only.

  5. unacceptable alignment and length and should have corrective surgery for alignment and lengthening.

 

Question 40-A 62-year-old man sustained the fracture seen in Figure 217. The knee and hip arthroplasty components were felt to be stable and were not causing problems up to this point. The previous distal femur fracture had healed. Stabilization of the new fracture should include

 

 

 

  1. retention of all implants and fixation of the fracture with an anteriorly-based locking plate.

  2. retention of all implants, application of an anteriorly-based cortical strut allograft, and cerclage cabling of the fracture.

  3. removal of all implants and total femoral endoprosthetic replacement.

  4. removal of the distal femur plate and revision to a longer distal femur locking plate that bypasses the stem of the proximal femur component.

  5. removal of the hip arthroplasty component and distal femur plate and revision of the proximal femoral component to a long, porous-coated component that bypasses the fracture site.

PREFERRED RESPONSE:4

 

Question 41-What is the recommended antibiotic coverage for a patient sustaining a type II open tibia fracture in a motor vehicle collision?

  1. Fluoroquinolone

  2. First-generation cephalosporin

  3. First-generation cephalosporin and penicillin

  4. First-generation cephalosporin and fluoroquinolone

  5. Third-generation cephalosporin

 

Question 42-Figures a and b are the anteroposterior and lateral radiographs of a 14-year-old boy who is brought to the emergency department with right ankle pain. Radiographs reveal an irregularity of the articular surface of the tibial plafond and a Salter-Harris III fracture.

What is the next appropriate treatment step?

 

 

  1. CT scan of the right ankle

  2. Advance weight bearing as tolerated

  3. Open reduction and internal fixation of the fracture

  4. Weight-bearing restriction for 4-6 weeks with conversion to a short-leg cast

  5. Weight-bearing restriction for 4-6 weeks with conversion to a long-leg cast

PREFERRED RESPONSE:1

 

Question 43-Figures a and b are the radiograph and CT scan of an 85-year-old woman who fell and sustained a fracture of the dominant distal humerus. What is the most appropriate treatment?

 

 

  1. Total elbow arthroplasty followed by supervised therapy

  2. External fixation for 4 weeks followed by an articulated dynamic brace

  3. Hanging arm cast for 4 weeks followed by an articulated dynamic brace

  4. Closed reduction and limited internal fixation, followed by an articulated dynamic brace

  5. Open reduction and internal fixation and 4 weeks of immobilization followed by supervised therapy

 

PREFERRED RESPONSE:1

 

Question 44-A patient underwent fixation of a transverse patella fracture with a modified tension band construct. What is the most likely patient complaint?

 

  1. Extensor lag

  2. Painful neuroma

  3. Painful implants

  4. Painful posttraumatic arthritis

  5. Loss of knee range of motion

PREFERRED RESPONSE:3

 

Question 45-Figures 232a and 232b are the anteroposterior and lateral radiographs of a 32-year-old man with a midshaft femur fracture. When performing reamed medullary nail fixation of the fracture, the interlocking technique should be

 

 

 

  1. static locking above and static locking below the fracture.

  2. static locking above and no interlocking below the fracture.

  3. dynamic locking above and static locking below the fracture.

  4. no interlocking above and static locking below the fracture.

  5. no interlocking is necessary because the femoral nail geometry should maintain rotational control.

 

PREFERRED RESPONSE:1

 

Question 46-A 32-year-old woman has ongoing pain 2 years after a motor vehicle collision in which she sustained a pelvic fracture. Chronic instability of the pelvis as the source of the ongoing pain can be best assessed with

 

  1. a CT scan.

  2. a supine anteroposterior (AP) pelvis radiograph.

  3. a standing AP pelvis radiograph.

  4. inlet and outlet radiographs.

  5. alternating single-leg-stance radiographs.

 

PREFERRED RESPONSE:5

 

Question 47-After fixation of an intra-articular distal humerus fracture through a posterior transolecranon approach, a surgeon contemplates transposition of the ulnar nerve. Which statement best summarizes the fate of the ulnar nerve with regard to transposition?

 

  1. There is no benefit from transposition.

  2. There is less subsequent ulnar neuritis.

  3. There is higher-risk cubital tunnel with transposition.

  4. There is risk for failure if there are medial implants; transposition will protect against neuritis.

  5. The reoperation rate is higher if the nerve is left in its native position.

 

PREFERRED RESPONSE:1

 

Question 48-What is the most likely deficit in elbow function resulting from an isolated lesion of the ulnar nerve above the elbow?

 

  1. No elbow deficit

  2. Weakness of elbow flexion

  3. Weakness of elbow extension

  4. Weakness of forearm pronation

  5. Weakness of forearm supination

 

PREFERRED RESPONSE:1

 

 

 

 

 

Question 49-Figures 244a and 244b are the postreduction radiographs of a 50-year-old patient with a distal radius fracture. According to the AAOS Clinical Practice Guidelines, the strength of the recommendation

for surgical fixation is

 

  1. weak.

  2. negative.

  3. moderate.

  4. consensus.

  5. inconclusive.

 

PREFERRED RESPONSE:3

 

Question 50-A 26-year-old man has a grade I open midshaft femoral fracture and a closed-head injury (Glasgow Coma Scale score of 6). After initial resuscitation and neurosurgical evaluation, he is considered hemodynamically stable. The head injury is a subdural hematoma that requires monitoring and maintenance of cerebral perfusion pressure. The most appropriate treatment is debridement and irrigation of the fracture and

 

  1. external fixation.

  2. unreamed retrograde intramedullary nailing.

  3. reamed retrograde intramedullary nailing with reamer irrigator aspirator.

  4. open reduction and internal fixation with a lateral plate.

  5. open reduction and internal fixation with a submuscular plate.

 

PREFERRED RESPONSE:1

 

Question 51-A 27-year-old man sustained a fracture-dislocation of his right knee during a motor vehicle collision.Plain radiographs and a CT scan revealed a displaced medial right tibial plateau fracture. An MRI scan of the knee revealed a peripheral detachment of the lateral meniscus, complete tears of the popliteofibular and fibular collateral ligaments, posterolateral capsular disruption, and partial tearing of the popliteus muscle. The anterior and posterior cruciate ligaments remained intact and attached to the medial tibial plateau articular segment. Surgical repair of this injury complex should include open reduction and internal fixation of the tibial plateau fracture and which of the following?

 

  1. Lateral meniscal repair only

  2. Lateral meniscal repair and allograft reconstruction of the popliteofibular ligament only

  3. Lateral meniscal repair and allograft reconstruction of the fibular collateral ligament only

  4. Lateral meniscal repair and allograft reconstruction of the fibular collateral and popliteofibular ligaments

  5. Lateral meniscal repair and allograft reconstruction of the fibular collateral and popliteofibular ligaments and the popliteus muscle

 

PREFERRED RESPONSE:4

 

Question 52-Which treatment approach for acetabular fractures carries the highest risk for heterotopic ossification?

 

  1. Ilioinguinal

  2. Watson-Jones

  3. Extended iliofemoral

  4. Extended ilioinguinal

  5. Modified Rives-Stoppa

 

PREFERRED RESPONSE:3